Sunteți pe pagina 1din 54

EXPLANATORY NOTES FOR VERBAL ABILITY

9. WORD USAGE: 10. ‘Laidback’ means ‘easy-going’ or ‘casual’. Options A,


B and D are synonymous with it.
1. The word ‘mean’ can be used in sentences 1, 3 and 4. Choice (C) ‘upright’ is the opposite of ‘laidback’.
(3 times) Choice (C)
2. The word ‘point’ can be used in sentences 1, 2 and 4. The 11. ‘Boorish’ refers to a behaviour which is unpleasant and rude.
word ‘view’ can fill in the blanks in sentences 1, 3 and 4. Uncouth, unseemly and awkward imply the same idea.
(3 times) ‘Pompous’ is behaving in a self-important manner.
So it is inappropriate. Choice (D)
3. The word ‘good’ can be used in all the four sentences.
(4 times) 12. ‘Serene’ implies peace. Composed, unperturbed and calm
4. The word ‘catch’ can be used in sentences 1 and 2. are synonymous with it. ‘Queer’ which means peculiar or
(2 times) strange is inappropriate here. Choice (A)

5. The word ‘claim’ can be used in sentence 4, the word ‘call’ 13. ‘Cursory’ means to give a quick and fleeting glance.
in sentence 1, the word ‘visit’ in sentence 2 and the word ‘Attentive’ is an inappropriate substitute. Choice (B)
‘figure’ in sentence 3. So, each word can be used in one 14. ‘Entice’ is to attract or lure. Woo and draw can be used in its
blank only. (1 time) place, but not help. Choice (A)
10. APPROPRIATE SUBSTITUTE: 15. ‘Ominous’ suggests that something bad is likely to happen
and is not a welcome signal. Menacing, sinister and baleful
1. ‘Obdurate’ means stubborn, (obstinate or rigid) and is the are also unpleasant, whereas propitious indicates fortunate.
appropriate choice. ‘Extravagant’ is a tendency to spend a Choice (C)
lot. ‘Mean’ and ‘petty’ suggest narrow-minded.
Choice (A) 12. ANALOGIES:
2. A punitive measure is one which serves as a punishment. Only 1. Contemplate is to ponder (synonymous) as captivate is to
disciplinary gives this idea. ‘Retaliatory’ is to retort (reaction to capture. The words fascinate (charm) and wonder, mitigate
harm), ‘Vindictive’ is trying to harm someone whom you think (ease; lessen) and reject, fabricate (invent) and feign
has harmed you. ‘Conciliatory’ is trying to make angry people (pretend to feel or have) are not synonymous.
calm. Choice (C) Choice (D)
3. ‘Factitious’ is being too artificial or contrived. ‘Flippant’ is 2. The depth of water and the strength of colour can be found
being casual, ‘facetious’ is being amusing at a time when it out. The other options are not related in this manner.
is not warranted. ‘Factual’ is providing facts. Choice (C)
Choice (B)
3. Blood can clot on its own just as milk can curdle on its own.
4. ‘Pensive’ is to be engrossed in thought because you are sad We don’t need an agent or doer for this activity. The other
or worried. Choice (A) three activities need an agent. Choice (B)
5. Assiduity means to be diligent and hardworking. Options A 4. Piston is a part of a power engine. Wings are part of an
and C can be eliminated because of their negative aeroplane. The remaining options are unrelated.
connotation. Choice (B) Choice (C)
11. INAPPROPRIATE SUBSTITUTE: 5. We recite a poem just as we chant a prayer. We cannot
express a speech. Choice (C) can be eliminated because all
1. “Utiopian’ world is an imaginary or visionary world. ‘Stark’ is though dance is choreographed, the order of the words is
the odd-word. Choice (D) reversed in this option. Choice (B)
2. Dry, droop and shrivel can replace wither (= because 6. Radical (extreme) and moderate are antonyms, similarly
shrunken or wrinkled). Blossom (bloom) is inappropriate. revolutionary (rebellious) and conservative (moderate) are
Choice (B) not antonyms but synonyms. Epicurean (devoted to
pleasure) and extravagant (exaggerated) grotesque
3. Pinnacle, acme and prime convey the same meaning as (absurd) and massive (large and heavy) are not antonyms.
zenith (peak). Perigee, meaning the point in the orbit of the Choice (D)
moon, is inappropriate in the given context. Choice (D)
7. Death occurs in the gallows, love is felt in the heart.
4. Banter, jest, chaff and badinage indicate humorous, teasing. Choice (B)
Choice (C)
8. An edifice has a foundation. A nation has a constitution.
5. Unsnarled means not twisted. Options A, C and D convey Choice (C)
the same meaning as the word disfigure. Choice (B)
9. A botanist studies plants while a sociologist’s area of study
is society. Choice (A)
6. ‘Eclectic’ means ‘wide’ or broad–ranging while ‘unique’
means distinctive or special. Choice (C) 10. Opthalmia affects the eye while rickets is a disease that
7. ‘Hallowed’ means holy or sacred or worshipped. Choice (A) affects the bones. Choice (B)
‘famous’ means well known. Choice (A)
13. REVERSE ANALOGIES:
8. ‘Desecrated’ means treated with disrespect, violated,
vandalized or deface. 1. Despotism is a quality of a tyrant. So, among the choices
Choice (D) ‘impaired’ means lessen or weaken. only option C is not similar. Irony does not indicate a person.
Choice (D) All other choices define a person (misanthrope, traitor and
debonair). Choice (C)
9. ‘Opalescent’ means ‘many coloured’ while choice (B)
‘glorious’ means ‘wonderful’ or ‘magnificent’. The word 2. A mendicant is a beggar. So, option A is the odd pair.
‘opalescent’ deals with colours. Choice (B) Choice (A)
Triumphant Institute of Management Education Pvt. Ltd. (T.I.M.E.) HO: 95B, 2nd Floor, Siddamsetty Complex, Secunderabad – 500 003.
Tel : 040–27898195 Fax : 040–27847334 email : info@time4education.com website : www.time4education.com SM1001941/138
3. Carpenter, butcher, farmer and surgeon use a saw, a knife, 2. Only 'negate' can fit in the second blank since the 'claim'
an axe and a scalpel respectively. Option D is the answer. made is nullified by facts. Choice (C)
Choice (D)
3. The 'strain' can be 'unprecedented' or 'tremendous'. This can
4. A sentence has some words just like a poem has some be 'baffling' or 'intimidating'. Choice (B)
verses, a forest has trees and an organ has cells.
Choice (B) 4. The sentence talks about 'General Anxiety Disorder' − hence
the tension experienced cannot be understandable or
5. The young one of a cat is called a kitten. The young one of acceptable. It has to be exaggerated or unreasonable. Worrying
an elephant is a calf and not a cub. Choice (D) over 'stressful life-events' is normal (it is not a disorder), hence it
must be 'routine life-events. Choice (C)
6. Wig is used for baldness. Options A, C and D share the same
analogy. You can’t use exercise for obesity, you need to do 5. Choice D can be ruled out since forsaking the match will not
it. Choice (C) lead to the dream being raised. Of the other three 'shattered'
is better than 'stumped' or 'quashed'. Choice (A)
7. We retrench employees to cut cost. We cut flab to remove
excess fat. We dispose waste which we don’t need. We 6. The word which best describes spreading of idea or
prune trees to remove extra–growth. All these relate to experiences is disseminate. So, circulated and distributed
elimination of what is not necessary. Redressal of can be eliminated. Dissipated means to disappear or
grievances does not share the same relationship. disperse, which is ruled out here. For the second blank, the
Choice (B) word repudiate which means to disown is inappropriate.
Experience cannot be promulgated. Replicate fits in best.
8. Cricket is a sport, squash is a vegetable, cricket is also an Choice (C)
insect and maroon is a colour. Spout is not a container. It is
a part of a container. Choice (A) 7. Although the first word given in the first option ‘shortage’ can
be chosen, the second word ‘potential’ is not suitable.
9. Reading a map is similar to the relationship suggested in Scarcity and shortcoming do not fit in the first blank. Dearth
choices A, B and C. We can’t blemish a scar. Blemish is to and market are apt. Choice (B)
cause a damage. Choice (D)
8. Options A and C contain a combination of words which are
10. Absence of malice is goodness. Absence of light is darkness contradictory. A person who is ‘relegated’ or ‘uncomfortable’
and not blackhole. So that is the answer. The other three can’t be working to ‘improve’ or ‘build’ the image of the
options are similar to the one in the question. government. The word affiliated is usually followed by ‘to’
Choice (C) and can be ruled out. Hence, option D ‘isolated’ and
‘tarnishing’ is correct. Choice (D)
11. In the given pair, the relationship is of the type MEMBER :
9. All the words given as options for the first blank seem
GROUP. A similar relationship is there in A, B and C. Hence,
suitable. However, for the second blanks only ‘unlikely’ can
the odd-pair is option D. Choice (D)
be chosen. Choice (A)
12. A pebble is a small stone, a brook is a small stream, a rock 10. The first two options can be eliminated because ‘provides’
– a small boulder, a shrub – a small tree. The same and ‘encourages’ cannot go with appetite. Between options
relationship is not there between tea and drink. C and D, ‘inviting’ sight is better than ‘pleasant’ sight in the
Choice (D) given context. Choice (D)
13. Here option C is the odd-pair. A restaurant is not used 20. CLOZE TEST:
primarily for recreation. Choice (C)
1. Only anonymity, meaning a state of not being known or
14. Warmth is a quality of the hearth. The same relationship is recognized, can give people a sense of freedom. It is absurd
exhibited by choices B, C and D. However, Igloo is a type of to say that animosity (hatred or dislike), anomaly (something
house and hence is the odd man out. Choice (A) that departs from normal) and vicissitude (changes of
circumstances or fortune) can give people a sense of
15. Flax is a type of fibre, not a costume. Choice (B) freedom. Choice (B)
2. Small town attitudes are jettisoned (rejected, thrown away)
14. CONFUSING WORDS:
in a big city like Mumbai. Jettisoning small town attitudes can
give people a sense of freedom. Hence only option C is in
1. (i) ‘Immanent’ means inherent (built-in, in this context),
line with what is stated in the para. The words exacerbated
while ‘imminent’ means expected in the near future.
(make something worse) camouflaged (covered, concealed)
(ii) ‘Ensure’ means to make sure of something, while and permeated (spread throughout) are inappropriate in the
‘insure’ is to provide for loss. given context. Choice (C)

(iii) ‘Deprived’ means lacking in something (lacking in 3. ‘Dwell on’ means spend time on or brood. Only option B is
means, in this case), while ‘depraved’ means of low apt in this sentence. The words endure, invest and spurn
moral standards. (reject) do not make sense in this context. Choice (B)
4. The word ‘Solitude’, meaning the state of being alone gels
(iv) ‘Crass’ means crude or disregardful of others’ feelings, with the description given in the sentence. The word solace
and is used to describe behaviour. ‘Cross’ means (comfort) is inappropriate in comparison. People don’t go to
annoyed or angry and is used to describe persons. the Himalayas to seek oblivion (anonymity) or transition
(v) ‘Rebuke’ means to scold harshly, while ‘rebuff’ means (change). Hence choices B and C can be eliminated.
to turn down or disregard help or suggestions offered. Choice (A)
Choice (C)
5. The word which fits into the blank is linked to the preceding
15 to 19: SENTENCE COMPLETION: word ‘privacy’ through the conjunction ‘and’ thereby
indicating that the two words convey the same idea. Hence
1. The word in the second blank qualifies 'manoeuvres'. It can only ‘quietude’ is appropriate in the given blank. The words
be electoral or political not secular or economical (in the trepidation (nervousness), pugnacity (quick to fight) and
context of the sentence). The word in the first blank should subtlety (shrewdness) don’t make sense in this context.
be bane not boon. Choice (A) Choice (B)
Triumphant Institute of Management Education Pvt. Ltd. (T.I.M.E.) HO: 95B, 2nd Floor, Siddamsetty Complex, Secunderabad – 500 003.
Tel : 040–27898195 Fax : 040–27847334 email : info@time4education.com website : www.time4education.com SM1001941/139
6. Manifest means to demonstrate or show - the darkness has 6. After matching the two columns, we know that ag and be are
manifested itself. This is the most appropriate word here. the correct combinations. Considering this, we can eliminate
Declare means to announce, authenticate means to prove, choices A, C and D. Choice (B)
substantiate means to provide evidence. Choice (C)
7. The correct combinations are ah and be, and we find this
7. ‘Dark ages’ suggests moving back in time – hence regressed only in choice A. Choice (A)
(return to a former state) is apt. Denigrate (criticize unfairly),
renege (to go back on a promise) and detract (to take away 8. The correct match is ag and bh and this is given in choice A
the value of) are unsuitable in the context. Choice (A) and B. ce go together. Choice (B)

8. The imbalance is most pronounced (very noticeable). It is not 9. af and bg are the correct combinations, so choice C is the
prophetic (predicting the future) or propitious (favourable) or correct combination. Choice (C)
protracted (drawn out). Choice (D)
10. af and bh are the correct combinations and this is found in
9. Indians have a preference (a greater liking) for the male choice D. Choice (D)
child. It is not a fuss (a display of excessive excitement) or
fervour (a passionate feeling) or selection (the action of 22. INCORRECT / INAPPROPRIATE USAGE:
choosing). Choice (B)
1. In sentence C, ‘touch too loud’ is incorrect. ‘a touch’ means
10. The word ‘despite’ at the beginning of the sentence suggests slightly (a little) so, the sentence should read ‘a touch too
that the operation has become brazen (bold and shameless) loud for my taste’. Choice (C)
not troublesome (giving trouble) or reserved (slow to reveal
emotions or opinions) or insolent (rude). Choice (D) 2. The correct expression to suggest that ‘enough time has
passed’ is ‘in the course of time’ not ‘in course of time’.
11. Semantically, any of the four words given as options can take ‘Course’ in sentence A means direction. In sentence D,
the first blank. Saving the elephant seems to be at the cost ‘course’ refers to the path a river takes. Choice (B)
of targeting the hippopotamus is what the passage suggests.
So contextually, we can conclude that option C is the best 3. In the first sentence ‘school’ is used as a collective noun. We
choice. Choice (C) say, ‘school of dolphins’. School for dolphins is absurd.
Choice (A)
12. Nations ban trade is the correct usage, not stop. Usually, we
say there is an ‘embargo on trade’ not ‘embargo trade’, 4. The phrasal verb to be used in sentence C is ‘read through’
‘sanction’ used as a verb means allow or permit. This is and not ‘read up’. ‘Read through’ means to ‘read carefully’.
illogical in this context. Choice (B) ‘Read up’ is to ‘read a lot’ on the subject. ‘Read between the
lines’ is an idiomatic expression which means to look for a
13. The hippopotamus does not have horn, antlers or mane. meaning which is not openly stated. Choice (C)
Ivory is obtained from the teeth of the hippo. Choice (A)
5. The second sentence should read ‘due for’ release and not
14. We use the word lament when we want to express a sense ‘due to’ release… ‘Due to’ means ‘because of’ which is
of loss, and unhappiness. This word is better than all the incorrect here. Choice (B)
other words provided as options. Choice (B)
6. In sentence A ‘heave’ means to haul or pull. In the second
15. Expansion ultimately leads to explosion. So, expand is the sentence it refers to the rise and fall of the sea. In sentence
apt word. When a body decomposes, methane gas is four it means to let out. In the third sentence it is
produced. The body does not decompose, decay or pollute meaningless. Choice (C)
with gas. Choice (C)
7. In the first sentence ‘inner life’ means how he feels mentally.
21. MEANING AND USAGE TABLE: In the second ‘inner Paris’ refers to the central part of the
city. In the third ‘inner circle’ refers to the privileged
1. The meaning implied in ‘a’ has been used in g. Hence, we members. In the fourth ‘inner efforts’ in not apt.
look at options A and D only. b − h is common to both these Choice (D)
choices. Hence, we look at c. c − e is the right combination.
Choice (A) 8. ‘A plastic smile’ refers to an artificial smile, ‘the plastic minds’
are the impressionable minds while being plastic is being
2. AG is the match in all the four choices and hence it can be malleable or flexible. But there is no such usage as ‘plastic
ignored. f can match only with c. This match is there in choice climate’. Choice (C)
b and d. 'Summer grinds on ….' in h suggests continuing for
a long time (d) while ,,,,, ‘a real grind’ …… in e suggests hard 9. ‘Sales pitch’ refers to sales talks, ‘to make a pitch’ is to make
work (b). Choice (B) a determined effort, while ‘reserving its pitch’ refers to the
sites chosen. But you can’t coat with a pitch of gold since
pitch as a means tar or bitumen. Choice (D)
3. c − e is the right match. This is found in choices B and D. So
we look at the next sentence. f matches b. So choice D is the 10. In sentence B ‘mean’ refers to a middle course between
right answer. Checking further, g matches d and extremes. In C ‘mean’ is to signify. In D, it refers to an
h − a go together. Choice (D) average. There is no such phrase as ‘men of mean’ (means
or mean birth would be appropriate). Choice (A)
4. The flash that blinds you can only be the flash of a camera
bulb (or lightning which is not mentioned). Hence c − e go 23. SENTENCE WITH AN ERROR:
together. This is found only in choice B. To confirm further
we look at the other sentences. ….. catch a flash of …… in f 1. .……respite is followed by with and not from……..
means a glimpse (a) …… a flash of a moment ….. in g Choice (D)
means very quickly (d). Finally h and b match with each
other. Choice (B) 2. .…….different from…….. Choice (B)

5. e gels with c. This is found only in choice B. So we examine 3. ……as her sister’s (..hair). The comparison is between
it further. f matches a. g can match d only, which h Mina’s hair and her sister’s hair. Choice (D)
matches b. Choice (B)
4. …….risen considerably…….. Choice (C)
nd
Triumphant Institute of Management Education Pvt. Ltd. (T.I.M.E.) HO: 95B, 2 Floor, Siddamsetty Complex, Secunderabad – 500 003.
Tel : 040–27898195 Fax : 040–27847334 email : info@time4education.com website : www.time4education.com SM1001941/140
5. ……twice as much as what…. Choice (C) 10. ‘To see through something’ means to understand
somebody’s real intentions. Choice (B)
6. “……..conventional taboos…….”. Choice (B)
7. “……..blotting paper………”. Choice (B) 26. ERROR IDENTIFICATION:
DOUBLE SENTENCE TYPE:
8. “………. the…..changes that occur…….” Choice (B)
9. Replace “but also” with ‘and”. Choice (C) 1. When treated as an independent sentence, the second
sentence is incomplete. It is a sub-ordinate clause, which
10. .....engulfed is followed by with and not in……. begins with ‘because’. Choice (B)
Choice (D)
2. Both the sentences are grammatically wrong. In the first
11. ……….. an infallible. Since the word begins with a vowel it sentence, the definite article ‘the’ has to be replaced by ‘a’.
should be followed by ‘an’ because population is a singular In the second sentence, ‘struck down’ should be used and
subject. Choice (C) not ’struck off’ because a court strikes down. Besides, names
are ‘struck off’, orders are struck down. Choice (C)
12. ………. population ….... is ………... Choice (C)
3. Both sentences are grammatically correct.
13. …. not only absorb …….. but also to discharge …….. Choice (D)
Choice (D)
4. There is no error in both the sentences. Choice (D)
14. …….. resemblance is followed by to ……….
Choice (B) 5. Bombs explode when someone causes them to explode.
15. ……… who lives in Florida. The relative pronoun ‘who’ is The verb ‘explode’ does not take the passive form (were
appropriate here. Choice (D) exploded). So, sentence I is incorrect. Choice (A)

16. ………. I have lived in ……... Choice (A) 27. FILL IN THE BLANK:
17. ……… that he has no time to spare ………. or, ……….. little 1. The sentence denotes an unfulfilled condition. The given
time to spare ………….. Few is used for countable nouns, verb ‘were’ is in the past tense, so we should use ‘would
little for uncountable nouns. Choice (C) swear’ to make the sentence right. Choice (C)

24. GRAMMATICALLY CORRECT STATEMENT: 2. Choice A, B and D do not make the sentence meaningful, so
choice C is apt. Choice (C)
1. .......entrust is followed by……to. Choice (D)
3. The idiom ‘a bee in the bonnet’ means an obsession, and
2. “from the beginning to the end”. Choice (A) this is correct in the context. Choice (D)
3. “......enter into an agreement”. Choice (D) 4. ‘Nemesis’ means retributive justice. As this is meant in the
context, this is the correct choice. Choice (C)
4. “……to him or his brother” is correct. Choice (A)
5. Since the first part of the sentence says corruption is
5. Doubt is best expressed by whether. Choice (C) common, Tehelka. com (which made an issue of corruption)
is aptly described in B (to make a mountain out of a mole hill
25. REPLACING THE UNDERLINED PART: means making a lot of fuss about something trivial).
Choice (B)
1. ‘The ease with which he got through his work’ refers to the
6. Between the conjunction ‘if’ and ‘whether’ after the word
manner of working which appears to be without any strain.
ponder, whether is preferable. Although ‘if’ or ‘whether’ can
Choice (B)
be used whenever a choice is offered, verbs such as
2. The underlined words means that though we have eyes we discuss, consider, ponder and decide are usually followed by
donot observe the things in front of us. Choice (C) whether. In option B, the Prime Minister asked (reported
speech) should be followed by the verb in the past tense
3. The underlined words refer to the animals in forests and (was) conducive and not ‘is’. ‘In the name of developing
plains. Choice (D) backwardness’ in option C makes the meaning absurd. In
the fourth option ‘for the sake of” is unacceptable in the given
4. The underlined words mean having the ability or capacity. context. Choice (A)
Choice (B) 7. Sentence A is wrong because of the incorrect conjunction
‘as’ in the beginning. Inappropriate word storm ‘stuck’
5. The words ‘… being written down …’ mean being considered instead of ‘struck’ in sentence B makes it wrong. The verb
or regarded as. Choice (A) evacuated cannot be used in the active voice in the given
context. So sentence C is also incorrect. Choice (D)
6. The expression “entirely of one’s doing” mean that whatever
has happened, the person concerned has done it and he 8. Incorrect positioning of the phrase ‘as part of an international
alone is responsible for it. Choice (B) agreement’ makes the first option incorrect. “Known as
Kyoto protocol” in the concluding part refers to the
7. “Hard pressed for money” means short of money. agreement and not carbon credit coming into vogue.
Choice (D) ‘Increased awareness’ in sentence B is wrong (error in
parallelism). Besides, the omission of comma after the word
8. ‘To leave no stone unturned’ means to work very hard, to do control in A and B also make them grammatically wrong.
every possible thing. Choice (A) Sentence D is a clear distortion. Choice (C)
9. The word ‘so’ indicates a comparison. He is such a meek 9. Brahmaputra with its tributaries is a single unit. Hence
person that…..or ‘floods’, should be used to ensure subject-verb agreement.
He is such a coward that….we need a clause in place of the So, option A is the obvious choice. Option B contains an
underlined part and the replacement should also function as error in parallelism (blocking and pushes). Option C blocking
the main clause and also convey the same idea as in the has been followed by pushing, but the sense is incomplete.
original sentence. Choice (A) Option D is also an incomplete structure. Choice (A)
Triumphant Institute of Management Education Pvt. Ltd. (T.I.M.E.) HO: 95B, 2nd Floor, Siddamsetty Complex, Secunderabad – 500 003.
Tel : 040–27898195 Fax : 040–27847334 email : info@time4education.com website : www.time4education.com SM1001941/141
10. In this question, options B and C are blatantly incorrect. 23. ‘Market’ is singular, therefore, plural verbs like ‘have’ and
(hosting long, mega carnivals thrice a month in option B and ‘are’ are ruled out. Choice (C)
positions shopping paradises in option C).
The incorrect positioning of the adverb ‘now’ makes 24. The words ‘syndrome’ in option A, ‘problem’ in option C are
option D wrong. Option A is the best choice. Choice (A) awkward. Also, China cannot start any ‘bilateral’ agreement
‘among’ other countries. Hence options A, C and D are
11. In sentence A, the word ‘together’ has been positioned incorrect. Choice (B)
incorrectly. It is used in this sentence as an adverb qualifying
the verb ‘work’ and should follow it. In sentence C, peasant 25. Options A, B and C have too definite a tone (surely, you will).
and intellectual must work for ‘their’ country, not ‘his’ country Option D is perfect because of ‘probable’ in it.
(subject – verb agreement). The expression ‘must have won’ Choice (D)
in sentence D distorts the meaning. Sentence B is
grammatically correct. Choice (B) 28. IDENTIFY THE INCORRECT SENTENCES:
1. Sentence (a) refers to more than one person. So ‘seats’ is
12. A list of articles is singular and hence has to be followed
correct, not seat. Sentence (d) contains a blatant
by ‘has been published’. So, options B and D are
error– ‘meeting deceased brother’. Choice (A)
unacceptable. In sentence C, astonished cannot be used in
the passive form. Option A is grammatically correct. 2. ‘To be made of sterner stuff’ is the correct idiom. It means to
Choice (A) have a stronger character than others while handling
problems. In sentence (c), ‘hasn’t affected her at all’ is in the
13. Omission of the article ‘an’ before idealistic makes choice A
present perfect tense. So ‘happen’ should also be used in
wrong. The absence of verb ‘is’ after idealistic in choice B
the same tense (has happened), not just ‘whatever
makes it grammatically incorrect. Positioning of the word
happened’, which is simple past. (b) and (c) are incorrect,
‘also’ in choice C alters the meaning. Choice D is correct.
making option A the answer. Choice (A)
Choice (D)
3. In this question, the errors are in sentences (b) and (c).
14. Sentences B and C distort the meaning completely and are
‘Ones’ personality’ in sentence (b) is an example of
unacceptable. Mix-up of tenses in sentence D makes it incorrect
punctuation error. The apostrophe should be before ‘s’
‘was the frequency’ is in past tense. ‘Bangladesh is now
(one’s). In sentence (d), ‘the other’ can be used if we are
mentioned’ is in present tense. Choice (A)
referring to someone specific. Since, it is a general
15. The correct option is C. In sentence A, ‘heritage sites’ which reference, ‘not another’ is appropriate. Choice (B)
are languished is incorrect, (passive form is not acceptable
4. Sentence (a) has an error in the usage of preposition. ‘Victim
here). In sentence B, ‘thanks for’ is wrong. The comma after
of political turmoil’ not ‘in’. Sentence d is incorrect because
languishing in sentence D conveys an absurd meaning and
‘flee his country’ is the correct usage not ‘flee from’.
is wrong. Choice (C)
Choice (B)
16. The conjunction ‘only’ in the second half of the sentence
5. Sentence c is incomplete and hence incorrect. ‘prohibition
suggests that a contrast is highlighted. So, option C which
gave a huge boost’ has to take an object (here bootlegging).
begins with ‘Against’ is the best choice. ‘For’ and ‘under’ are
Omission of article before bootlegger in sentence d makes it
not suitable beginnings. Besides, the ‘importing of’ in option
grammatically incorrect. Choice (D)
A is unacceptable. Choice (C)
6. We do not enter a platform, we usually reach a platform. So
17. Here, the phrasal verb ‘served up’ has to be used to convey the
sentence b is incorrect. Sentence (c) is incorrect because of
intended meaning. So, option A is eliminated. Options C and D are
the use of the conjunction ‘and’ in place of the ‘to infinitive’.
ruled out because of the incorrect usage of the word ‘help’ and the
The sentence should read ‘I had to wade through the crowd
sentence structure which reads ‘a film on his life has been featured’
to reach my compartment.’ Choice (C)
instead of ‘feature film’. Choice (B)
7. Statement (d) is incorrect. The expression ‘invade into’ his
18. Option A is the best choice because the word ‘wronged’
privacy is erroneous. He fears both “will invade his privacy”
which implies the victim is correct here. Option D is incorrect
is correct. Choice (D)
because of the inappropriate conjunction ‘only’ which distorts
the meaning. Options B and C are eliminated because of the 8. Sentence (a) is erroneous because the adverbial form is
incorrect usage of the word ‘wrong’. Choice (A) predominantly not predominant. Sentence (b) speaks of
monks (plural) and should end with ‘lives’ or it should be a
19. Option A is incorrect. The sentence refers to the past and we
modest …….. In sentence (c), monks are charged is
can’t say it ‘can involve’ as if we are talking about a person.
incorrect because the simple present tense implies it is a
‘Involving’ in sentence C is a blatant error, ‘will involve’
regular feature. ‘In recent times’ indicates that the present
makes D wrong. Choice (B)
perfect tense (have been charged) has to be used.
Choice (A)
20. The idea that Attenborough read Gandhi’s biography in 1962
and for 20 years from then (that time), he was obsessed with 9. In sentence (d), preposition ‘in’ has to be replaced by ‘on’
the idea of retelling the story has been correctly conveyed (people on the streets). Choice (B)
only in option C.
The incorrect use of since, for and from and their 10. Forts and castles are marvels (plural) that remind one and
inappropriate positioning distorts the meaning in all the other not ‘reminds’ (error in subject – verb agreement).
options. Choice (C) Choice (C)

21. The sentence conveys the idea that something sudden 29. IDENTIFY THE CORRECT SENTENCES:
happened after a lull. This idea of sudden change after a
calm is best expressed by ‘when’. Choice (B) 1. Sentence (b) is wrong because it should be ‘a philosophical
bent’ (notice ‘a quirk of personality in the next part of the
22. Option A is ruled out since 'calm' and 'frantic' cannot go sentence). In sentence (c) it should be ways of seeing since
together. In option B, 'but' is inappropriate because firmness it talks of methods, tools (plural). Sentences (a) and (d) are
need not preclude calmness. Option C is also eliminated right. Choice (C)
because 'although' implies that a proud man is incapable of 2. Sentence (a) must have the definite article ‘the’ before ‘law’.
lofty thoughts. This need not be true. Option D is the most Sentence (c) must be ‘standards’ (plural). Sentences (b) and
appropriate. Choice (D) (d) are right. Choice (D)
Triumphant Institute of Management Education Pvt. Ltd. (T.I.M.E.) HO: 95B, 2nd Floor, Siddamsetty Complex, Secunderabad – 500 003.
Tel : 040–27898195 Fax : 040–27847334 email : info@time4education.com website : www.time4education.com SM1001941/142
3. In sentence (a), the adverb ‘actually’ must be placed 4. Option A is the most concise way of conveying the central
between the auxiliary and main verb – is actually under way. idea that more has been written about the Emergency while
In sentence (c) it should be ‘turns to’. Sentences (b) and (D) almost no attention has been paid to the Indian Railways
are right. Choice (B) strike. Choice (A)
4. In sentence (c), it should be ‘a broad spectrum’. In sentence 5. The whole idea is neatly conveyed in option B. In option C,
(d), it should be ‘in a number of Muslim countries….’ some key details like the name of the President, state funds
Sentences (a) and (b) are right. Choice (A) and the year are missing. Choice (B)
5. In sentence (b), it should be ‘to save’ (parallelism requires
that two phrases or clauses having the same relationship to 6. Sentence A is incorrect because of the mix-up in tenses. The
the verb must have the same structure). Sentence (d) must main clause (price revision has become) is in the present
have ‘than’ (comparison) not instead of ; Sentences (a) and perfect tense and should take a present tense verb.
(c) are right. Choice (D) ‘Continued’ is in the past tense in the sub-ordinate clause.
Incorrect use of prepositions (in the higher side) in sentence
B and (over a wide band) in sentence D eliminate these
30. RESTATING A SENTENCE:
choices. Choice (C)
1. Here the original sentence is about ‘all good writing’. The
author wants to stress on C things – it comes from the heart, 7. Sentence A is wrong because of the error in parallelism – to
is powerful, and touches the reader. These key elements are stay ahead and carving a niche is incorrect. It should either
there in the sentence in option A. Choice (A) be ‘staying ahead and carving a niche’ or ‘to stay ahead and
carve a niche’. Improper phrasing (definition of India’s
2. Key words in the original sentence ‘letters to fashion instead of Indian fashion) makes sentence B
cousins’….’fiction added to mundane activities’ and the unacceptable. The phrasal verb ‘brought about’ is used
purpose of the writer was to ‘entertain’. The essence is when we refer to changes introduced. So, sentence C is also
captured in option A. Choice (A) wrong. Sentence D has no grammatical error. Besides, the
issues have been listed in the most appropriate order. The
3. The idea intended in the original sentence is “most men do logical flow of ideas is maintained and hence this is the best
not marry the kind of women they otherwise admire”. Option choice. Choice (D)
C is closest to this idea. Choice (C)
4. ‘All is fair…….’ conveys the idea that the means do not 8. ‘Alternatively’ is used to suggest a second choice or
matter. Only the end result matters. Choice (D) possibility. ‘Alternately’ suggests two things happening one
after the other regularly. Options B and D can be eliminated
5. The original sentence states that gentlemanliness stems on that score. The comma positioned after alternately in
from conscious choice and is not symptomatic of a lack of sentence C distorts the meaning. Choice (A)
courage. This idea is conveyed in option C. Choice (C)
9. The adverb ‘allegedly’ has been positioned at the incorrect
6. The maximum time mentioned in the original sentence for place in sentence B. In sentence C ‘alleged disappearance’
keeping a test tube on the Bunsen flame is ten minutes. of tigers is inconsistent with his having a major role. In
Option A is ruled out because it says that the test tube should sentence D, ‘absconding of tigers’ is an absurd idea. People
be kept on the flame for more than ten minutes. Option C abscond, not animals. Choice (A)
says that we should remove the test tube for just ten minutes.
Hence, the actual meaning gets distorted in this option too. 10. In sentence A, the article ‘a’ is unnecessary before relatively
Similarly, option D talks about ‘waiting for ten minutes….’. backward Orissa. In sentence B, the adverb ‘by far’ should
Option B is the answer because the main idea is conveyed be placed after ‘as’. The deal must rank as and not ‘must be
accurately. Choice (B) ranked’. So, sentence C is also incorrect. Choice (D)
7. The given sentence states that this year in the budget, more
money was allocated to the agricultural sector than to any 11. In neither …. nor construction, the verb agrees with the
other sector. Only choice (3) conveys this very meaning. subject nearest to it. Hence intends. Further, the verb ‘intend’
Choice (C) should be followed by ‘to’ infinitive. Choice (C)
8. The theme of the given sentence is that we know much less
12. ‘Despite’ means ‘inspite of’, hence it cannot be followed by
about the common cold than we know of many other
‘of’. Inspite cannot be used without ‘of’. Choice (D)
diseases. Option D conveys this very idea. Choice (D)
9. The sentence says that aluminium is a better conductor of 13. ‘Several of the people’ and ‘several people was’ are
heat and electricity than other metals. Then the latter half incorrect. ‘Several people were’ is the correct form.
says “except silver, copper and gold”. So, it means that Choice (B)
silver, copper and gold are the only metals that are better
conductors of heat and electricity than aluminium. This is 14. Choice ‘B’ is wrong, as the pronoun ‘you’ is used instead of
what option C says. Choice (C) ‘yours’. Choice ‘C’ is wrong, as it is the colours that are
compared, not the watches. Choice ‘D’ is wrong as the
10. Here, the given sentence conveys the idea that a minimum
preposition ‘with’ is not used with ‘similar’. Hence, choice A
balance savings accounts earns more interest. This is what
is the correct statement. Choice (A)
option A also states. Choice (A)
31. CONCISE EXPRESSION OR BEST WAY OF 15. The cumulative conjunctions ‘as well as’, ‘and’, ‘both-and’,
WRITING OR CONFORMS TO STANDARD ‘not only-but also’ are used to join two positive ideas. In
choices A, B and C, wrong combinations of the conjunctions
ENGLISH USAGE:
are used. (both-as well as; both-also; and also) Hence,
1. Here, B is better than D because D gives the wrong impression of choice D is correct. Choice (D)
a universal truth. B is the best because it is the assertion of a
particular person about himself. Choice (B) 32. SENTENCE ANAGRAM:
2. Option C is the most concise. Choice (C) 1. The last word in part b ‘serious’ can be followed by the first
3. Choice A is ruled out because 'to square a circle ....... word ‘attempt’ in part d. Only options C & D have this
productive assets' is verbose. Choice B is also ruled out sequence. Between c & d, only part c immediately states
because 'anomaly of iniquitous distribution' is not concise. what the attempt was made for. Part a which begins with the
Choice D does not capture the idea that is being expressed preposition ‘with’ can be placed at the end. So choice C is
adequately. Choice C is the best. Choice (C) the best. Choice (C)

Triumphant Institute of Management Education Pvt. Ltd. (T.I.M.E.) HO: 95B, 2nd Floor, Siddamsetty Complex, Secunderabad – 500 003.
Tel : 040–27898195 Fax : 040–27847334 email : info@time4education.com website : www.time4education.com SM1001941/143
2. Between part b and d, the options which begin with d can be 33. PARAGRAPH ANAGRAM (Type – I):
the answer because part d refers to the main subject
(a team of doctors). Option D which places c after d gives an 1. 'a' cannot begin the para because it begins with the word 'it'
absurd meaning (one heart transplanted to four patients). and the word has not been explained in a, hence a is a
So, option B (dacb) is the answer. Choice (B) continuation of a previous statement. (Choice B is ruled out).
'c' cannot start the para because it begins with 'now' and
3. In this question, the key lies in linking ba together. This link therefore does not make complete sense unless preceded by
is provided only in choice C. The sequence provided in this a previous statement. Hence choice C is ruled out. Between
option badc makes a logical sentence. Choice (C) choices A and D, choice D seems to be more appropriate
because it begins with 'b' which gives an introduction to what
4. It is quite clear that bd should go together (political –dogma). the para is about. Further 'this year' in d and 'Now' in c link the
This sequence is found in options A, B and D. part a (when our two and they must go together. Choice (D)
action is guided) has to be followed by part b
(either by . . . ). This can be found only in option D. 2. Statement d cannot begin the para because it states that
Choice (D) there are other professions affiliated to conservation – this
implies that the word 'conservation' has already been
5. In this question, it is clear that db (realised that she looked mentioned in a previous statement. Therefore choices A and
familiar) and ca (not remember when or where he had seen D are ruled out. Between choices B and C which begin with
her) should go together. This sequencing is in options B and a, only B is correct because in choice C there is no
D. b at the end of the sentence makes it incomplete. So, connection between statements a and b. Choice (B)
dbca is the correct order. Choice (D)
3. Choice A is ruled out because there is no clear link between
6. Option B is ruled out because c cannot follow a. The 'd' and 'b'. Statements 'a' and 'c' may start the paragraph, but
sentence is not complete in this case. Also b cannot come 'c' is a better choice as it introduces the topic of stress,
after c. Option B is also ruled out because a cannot succeed whereas 'a' sounds like the continuation of a previous
b. Option C will also not be the answer as b coming after c is statement. Hence choice C is ruled out. Between choices B
illogical. a follows d. Here the sentence is complete with the and D, D is a better option because 'a' follows, 'b' better than
subject being “the power of morality” followed by the verb, 'd' does. 'cb' talks about how stress used to be the disease
which is followed by “of all intellectual powers”. of people racing down the fast lane of success, and 'a'
c precedes b. Choice (D) continues in the same vein, stating that today, stress has
become much more widespread than before. 'd' is an apt
7. The subject matter of the sentence is “the primary …method of concluding statement as it deals with the likely cause of stress.
cognition : music….”. Hence, b would follow a. And what does Hence choice (D) is the most appropriate. Choice (D)
music do? “(It)…..offers….automatic integration of sense data
into ….meaningful entity”. Choice (C) 4. 'b' cannot start the paragraph because of the presence of the
word 'its' which refers to something mentioned earlier. Hence
8. “….Competitiveness has reached a peak…” So, c would choice b is ruled out. 'd' cannot start the paragraph either
follow a. Also, d would come after c and b would come in the because of the word 'also' which implies that an alternative
end. Choice (B) use of the ecosystem is mentioned in a previous statement.
Hence choice d is also eliminated. Between choices A and
9. The subject matter of the sentence is “The presence of
C, C is better since 'a' follows 'e' better than 'd' does. 'a'
corporates bodies.” b would be the first line. a should follow
elaborates on why medicinal plants are important. This is an
b. c would come after d. c would be the last part of the
explanation for what is mentioned in 'e'. Hence eacbd is the
sentence. Choice (D)
best option. Choice (C)
10. b cannot follow d. Hence, option A is ruled out. b cannot
5. Since all answer choices start with 'b', we must choose
precede d. Hence, option C also cannot be the answer. a
between 'a', 'c' and 'e' to follow 'b'. 'e' is the best option as
cannot follow c. Therefore, we rule out option D. “A biotech
'fumigation' can be logically linked up to 'cleaned up' in 'b'.
revolution….” is the subject matter of the given sentence.
Choices, C and D are eliminated. The difference between
Hence, c comes first followed by b. Choice (B)
choices A and B is in the order of 'c' and 'd'. The sequence
11. We cannot start the sentence with parts a, b, d because all 'adc' makes more sense because 'a' mentions the reason for
of them have prepositions in the beginning. ‘c’ is the 'd' and 'c' mentions the likely impact of the actions mentioned
beginning part, followed by ‘d’ (‘set up’ should be followed by in 'd'. Choice (B)
‘in Tamilnadu’). The reasons for the set up are given in ‘b’
6. ‘a’ cannot be the opening line because of the word ‘its’ in it.
and ‘a’. b cannot be the final segment as it ends with ‘and’.
b follows d. (This in b refers to ragging becoming a norm)
So, the order is cdba. Choice (C)
Also, c is a better concluding remark than a.
12. The subject of the sentence is part c and that is followed by Choice (D)
a (committee set up by…..). The phrase ‘to study’ (what ?)
7. The subject matter of the passage is ‘language’ and not
leads us to d (takeover) and the ‘conclusion’ is given in b.
‘mime’. Hence, a will be the first line. Also, d can’t come after
So, the order is cadb. Choice (C)
a. Hence, we rule out options A and B. Also, ‘this method’ (in c)
13. On observing the parts of the sentence and the given refers to mime. Hence, c should come after b.
choices, it is clear that a is the beginning and the word ‘busy’ Choice (C)
in ‘a’ leads us to ‘c’. Then the subordinate clause given in b
8. 'd' opens the paragraph in all four options. It is clear that both
‘that they …….’ follows and d completes the sentence. So,
'b' and 'c' can follow 'd'. However, 'b' should follow 'c' because
the order is acbd. Though b can follow a, c cannot follow b
'such an approach' mentioned in 'c' refers to the militant
(shouting and demonstrating tells us about the M.Ps not how
approach mentioned in 'c'. Hence 'dcb' is a more logical
they deliberate). Similarly d follows b
sequence than 'dbc'. 'a' closes the paragraph. Choice (C)
(b as the last part of the sentence sounds incomplete)
Choice (D) 9. ‘a’ cannot start the passage as the word ‘also’ indicates that
some other statement has preceded it. Hence, option A is
14. It is clear the cd go together (when ….. decided) and ba go
ruled out. Also, b can’t be the first line because of the word
together (he would never enter). Between choice A and B,
‘it’ in b. a following c is awkward. Choice (D)
the latter is better because it gives the cause first and then
the effect. Choice (B) 10. ‘a’ cannot start the passage because ‘so’ indicates a
conclusion. Hence, options A and D are ruled out. b is the
15. Here bd go together (when ….. died) and ca go together opening sentence. Option C is ruled out because a cannot
(people suspected …. pollution …). Choice (A) succeed d. Choice (B)
Triumphant Institute of Management Education Pvt. Ltd. (T.I.M.E.) HO: 95B, 2nd Floor, Siddamsetty Complex, Secunderabad – 500 003.
Tel : 040–27898195 Fax : 040–27847334 email : info@time4education.com website : www.time4education.com SM1001941/144
11. The paragraph talks about the damage caused to heritage 22. After reading all the statements quickly, we decide that ‘d’
sites by the Red Guards and not particularly about the palace should be the concluding statement, as it begins with
mentioned in b. So d begins the para and is followed by b. ‘hence’. So, we can rule out choices A and C. Between
‘Was not so lucky’ in sentence a refers to the idea of choices B and D, statement a is the introductory sentence as
escaping destruction in b. So dba. In sentence c, ‘the statue’, it has the phrase ‘raises questions’ and the controversy is in
refers to ‘the Buddha statue’ mentioned in e and the ec ‘b’. So, the order is abcd. Choice (B)
sequence is clear. The fourth option dbaec is the best
choice. Choice (D) 23. c is the first sentence – from ‘a currency’ we come to ‘the
rupee’. This is followed by a – ‘the manic existence’ in c is
12. In this paragraph, b can be the opening sentence, but not the continued in the ‘rupee ruckus’ in a. d follows giving specific
concluding sentence. As a concluding sentence, it is too instance of the ‘rupee ruckus’. b is the concluding statement.
abrupt. So, choice A can, be eliminated. The “trade – off” in b Choice (C)
is explained in sentence d. Between a and e, it is evident that
24. d is the opening sentence (in all the choices) and we find that c
a follows d because it introduces the ‘lobbyists’. Knowing
follows it (the proverbial wisdom of d is applied to the market
people on Capitol Hill is easier for a former legislator. This link
place in c). a follows b – ‘do so’ in a refers to the action mentioned
can be established best only if e follows c and not vice versa.
in b (pour good money after bad). Choice (B)
bdaec is the correct order. Choice (B)
25. The opening statement b (in all the choices) is followed by d,
13. The opening sentence is the same in all the options. ‘Mild ‘these two views’ in d refers to ‘… short and long-term out
dose’ in d refers to the ‘herbal remedies’ mentioned in b. So look’ in b. a follows d ‘this’ in a refers to the ‘dicey situation’
d has to follow b, which is the case in options A, B and C. in d. Choice (D)
The opening sentence talks about harmony within the
system and its link with the external. Since diagnosis has to 26. Only in statement c the word ‘Shangri La’ is explained
precede remedy, e should precede b. ‘This was in keeping’ whereas in statements a and b the word is mentioned
in sentence a refers to the idea of surgery being considered casually. As such c is better as an opening statement. In
a last resort in d. So, the sequencing is cebda. Choice A, which begins with c, the statements are
Choice (C) sequentially arranged to form a logically coherent paragraph.
Choice (A)
14. The first clue to this question lies in linking sentences d and
a. ‘one of them’ in d has to follow the ‘three sleep difficulties’ 27. Choice A cannot be the right answer because statement a
in a. This sequence is found only in option A and the solution should follow statement d because a reiterates the fact
becomes easy. Between c and e, either can be the opening stated in Statement d – statement d states that Africa is more
sentence But the ad sequence in option A makes (ceadb) affected by global warming than other continents and this
the best choice. Choice (A) fact is emphasized by the findings of the study mentioned in
statement a. The same reason is attributed to choice d for
15. The para is about the difference in attitudes of the Indian and not being the right answer. Between choices B and C we
the European to clouds and d introduces this. The structure must decide whether b follows c or the other way round. b
of the first sentence makes it clear that the Indian’s attitude follows c because c talks of industrial pollutions that offer
has to be mentioned first. So e follows d. Cloud followed by protection and b says Africa not being industrialised lacks
rain is logical. So a follows d. ‘For him’ in c refers to the those. Choice (C)
European in b. Clouds depress him and the sun is a symbol
28. While b and a seem suitable as the opening sentence of a
of happiness and cheer to him. Hence, sunny smile.
para, a follows b - b ends with 'a new study shows' and a
Choice (D)
begins 'The study ……' d follows a, the link being 'forest'.
16. ‘But’ in a rules out a as the first statement. Hence, option A c concludes the para. Choice (B)
can be rejected. Option D is also ruled out because b cannot 29. Since ba are the opening sentences in all the choices, we
start the passage because of the word ‘it’ in it. Now, we need
must decide on what follows. c is right − 'The reason for this
to decide whether b should come before or after a. b would
painful paradox' …… as it links the situation presented in ba
come before a. The key indicator words are ‘originally’ in b
to the situation/solution in de. Choice (D)
and ‘recent’ in a. Choice (B)
30. c can be the opening sentence but d cannot follow it
17. a cannot start the passage because of the pronoun ‘they’ in it. (…. He should not stoop down …. in d cannot follow c which
Hence, option C can be rejected. c also cannot start the passage says …… vices rule the world …..). b seems appropriate as
because of the words ‘of these’ in it. b is a good opening the opening sentence. Now be (in choices C and D) should be
statement. Since b talks of two different things – science and followed by d − '…. remains in the human level' in e is followed
religion, statement d with ‘its’ cannot come after b. ‘It’ would by 'That means …..' in d. Choice (C)
relate to only one thing. a can come after b because of the word
‘they’ in a. Also, ‘these’ indicates that c would come after a. The 31. The words `these influences', in statement `a' indicate that
last line would be d. Choice (D) `a' is a continuation of an idea expressed in a previous
statement. `c' can begin the para and `c' should be followed
18. The words “it, moreover, finally” suggest that b, c or d cannot by `b' as `b' is a continuation of the idea expressed in `c'. a
start the passage. Hence, either C or D would be the answer. follows e − the link being ‘influences’. 'They' in d stands for
c cannot follow a. b logically follows a. Choice (C) '….. clarity of thought and perception' in a. Choice (B)
19. d cannot start the passage because of ‘most of them’ in it.
34. PARAGRAPH ANAGRAM (Type – II):
Hence, option C is rejected. We have to now decide whether a
or c is better as the opening line. c seems to be better. c would
1. ‘a’ cannot follow 1 because there is no connecting word that
be followed by b. a would precede d. Choice (B)
links 1 and a. Hence, options A and C are ruled out. Hence,
20. a, b or c cannot start the passage because of the pronoun either B or D is the answer. However, a cannot follow c
‘he’ in a and b and the definite article “the” in c. Hence, either because there is no logical connection between them. b
B or D is the answer. b cannot follow c because dcb is an follows c as it talks about the stones mentioned in c. d should
awkward sequence. Choice (D) follow b. a should follow d as it logically precedes 6.
Choice (D)
21. b is the opening sentence (in all the choices) and it is
followed by d (…. ‘lengthy documents’ in b is followed by ‘All 2. Here, the subject of 1 is the time period between 1206 A.D.
of them ….’ in d). This is followed by c (‘Yet…’ gives the and 1526 A.D. Hence, a cannot follow 1 where the reference
exception to ‘All of them’) and then a (which adds one more is to ’dynasties’. For the same reason d too cannot come
point to what is said in c) Choice (C) after 1. Hence, by the method of elimination we arrive at the
Triumphant Institute of Management Education Pvt. Ltd. (T.I.M.E.) HO: 95B, 2nd Floor, Siddamsetty Complex, Secunderabad – 500 003.
Tel : 040–27898195 Fax : 040–27847334 email : info@time4education.com website : www.time4education.com SM1001941/145
conclusion that b would come after 1. The answer would be 14. The pronoun “They” in sentence b following ‘human beings”
option A. You may do a quick check. b follows 1 because b in the opening sentence is logical. Also, b explains the
discusses the time period between 1206 A.D. and 1526 A.D. ‘strange activity’ mentioned in the opening sentence. “These
In b there is reference to five dynasties that ruled. c says, texts” in sentence d following sentence b is also clear. The
“Out of these five dynasties…”. Hence, c comes after a. reference to ‘crisis’ in sentence a is an extension of
Also, d would precede 6. Choice (B) “problematic situations” in sentence d. ‘Great deal of
scriptural activity’ in sentence c is described as being ironic
3. ‘His’ in a refers to Gandhi. Hence, a follows 1. Also, d would in the last sentence. The correct order is bdac.
precede 6. Such a combination is present in option A. Choice (A)
Choice (A)
15. ‘Others call them’ in sentence a has to follow sentence b (MD
4. b would precede 6 as b refers to a device invented by Pascal
…… calls them). So ba link is clear, and based on this clue,
while 6 is a logical sequel to b. Only option B has a
only option C can be eliminated. Sentence d, which conveys
combination with b in the end. One can read this option for
the idea of the generation dealing with problems too soon
further confirmation. Also, d, a or c cannot come before 6.
and meeting with a quarter life crisis explains why d has to
Choice (B)
precede 6. Besides, the words “impatient aspirers” in
5. c should precede b. Look at the words, “on one occasion….” sentence a are linked to “way too soon” in d. So, cbad is the
and on yet another occasion…” Such a combination is correct order. Choice (B)
present in option D. Hence, we scrutinize this option further.
a can follow 1, because the subject is Gandhi. Also, the 16. The paragraph is about a group fighting for the rights of the
words ‘all-pervasive” are there in b and d. Hence, d can Palestinians. ‘This group’ in sentence c refers to the
follow b. Choice (D) Palestinian campaign for Freedom and Peace which has
been introduced in the opening sentence. So c follows the
6. After reading all the statements, we can decide that a comes first statement. The sixth sentence beginning with the
before statement 6. As per the order of the words (planning, conjunction ‘But’ gives a clue that it is linked to sentence b
ingenuity, cunning) in statement a, dcb is the correct (group staying away form militant outfits). ‘Involvement of
sequence. Hence, the order is dcba. Choice (C) women’ in sentence c is followed by violent action not
attracting women in sentence d. So, the sequence is cdab.
7. After reading all the given statements, we know that a should Choice (A)
precede statement 6 (… pinball machines …. in a and ….
three shots …. in 6). b should follow d (Though he lost 17. In this para, it is very clear that sentence d will be followed
money, he was in high spirits) c follows b. Hence, the order by sentence b – the word ‘some’ followed by ‘others’.
is dbca. Choice (C) Similarly, sentence a following sentence c is also logical.
Sentence c refers to ‘a study’ and sentence a specifies the
8. c follows 1 – ‘windy and very cold’ in 1 makes you wish you study. This order can be seen only in option C. Also note that
had ‘gloves and muffler’. ‘But’ in d links it to c (the alternative the sixth sentence is linked to the idea expressed in
he was forced to choose). The ‘brisk walk’ in d leads him to sentence a. Choice (C)
‘the theatre half-an-hour before….’ in a.
b precedes 6 – ‘Hello Dolly’ is the name of the movie in 18. ‘No proper exit route’ in sentence c is linked to the ‘disaster
matinee show. Choice (D) to’ a small entrepreneur in the opening sentence. The
pronoun ‘He’ in sentence b refers to ‘An entrepreneur in
9. After reading the statements we know that a precedes sentence a. b follows a. So, the correct sequence in cabd.
statement 6, and d follows b – d gives an example for what Choice (B)
is stated in b. c follows statement 1 (power is the clue).
So, the correct order is cbda. Choice (A) 19. The words ‘suggests otherwise’ in sentence d are regarding
the impression that ‘Mao has been shelved’ in the opening
10. After reading the given statements, we decide that d sentence. So d follows 1. Sentence c following sentence b is
precedes statement 6 and a precedes d. c follows statement clear (rough hands and clothes revealing their rural
1 and b follows c, as both the statements describe the background). Reference to ‘mausoleum’ in sentence a and
restaurant. So, the order is cbad. Choice (C) details about its location in the concluding sentence makes
a the penultimate sentence. The sequence is dbca.
11. The words “more than 90% of land” in sentence d and “That Choice (A)
figure climbing to 100% in sentence a provide the clue to a
following d. So, option B can be eliminated on that score. 20. c is a continuation of the first sentence. ‘Subsequently the
The concluding sentence talks about some factors causing growth rate halved’ in sentence d makes it clear that b has
the damage (as doing the rest). This is linked to sentence b to precede it ‘Slump’ in a is linked to ‘slide’ in the sixth
(some damage). So, option C which puts b at the penultimate sentence. So, the correct order in cbda. Choice (D)
position is right.The opening sentence talks about Spain.
Sentence c which begins with “A third of the country” follows 21. ‘a’ would follow 1 and d would precede 6. c has another
the opening sentence. The sequence is cdab. example, while b has the first example (of Arjuna
Choice (C) Ranatunga). Choice (D)

12. The para deals with reasons for Africa’s poverty. The idea 22. ‘b’ follows ‘1’ because it gives an example of the “best efforts
that West is not going to help, in the opening sentence is to be good-looking” – mentioned in ‘1’. It is followed by ‘a’.
followed by “It is Africa” which must force the change in c. The key words are “Later I came to understand ….”. Also ‘c’
“The answer is” in sentence a answers the question raised precedes ‘6’. Hence (D) is the appropriate choice.
in sentence d. ‘They’ in 6 refers to Africans in b. So b Choice (D)
precedes 6. cdab is the correct sequence. Choice (B)
35. PARAGRAPH WITH A MISSING SENTENCE:
13. The opening sentence talks of a business group’s plan of 1. The tone of the paragraph is optimistic throughout. Hence
becoming a multi-activity conglomerate. This is followed by choice B that talks of auguring well fits well. All the other
sentence d, the wealth it is likely to generate if Mittal’s plan options are pessimistic, which is not in keeping with the
succeeds. ‘Changi airport’ and “Rothschild” in sentence b paragraph. Choice (B)
are the “world’s best names” mentioned in c. So, b follows c.
“Retail project” in a is linked to the “supermarket giant” in the 2. Since the paragraph begins by saying you are the architect
concluding sentence. The sequence is dcba. of your own future, the concluding line has to reinforce this.
Choice (D) Both choices C and D appear to be suitable. But if we look
Triumphant Institute of Management Education Pvt. Ltd. (T.I.M.E.) HO: 95B, 2nd Floor, Siddamsetty Complex, Secunderabad – 500 003.
Tel : 040–27898195 Fax : 040–27847334 email : info@time4education.com website : www.time4education.com SM1001941/146
at the sentence before the blank it talks of preparation for the 4. The main points in this extract are (1) Charlie Chaplin’s
future. Hence choice D, which also talks of preparation and portrayal of tramp and its universal appeal (2) qualities of the
carries the idea forward, is the most suitable. tramp (3) Chaplin’s skill in miming and (4) language being no
Choice (D) barrier. All these have been effectively conveyed in option C.
Option D has omitted the ‘tramp’. Option B – the last
3. The paragraph talks of the old and the new coexisting side sentence is a distortion. Option A is not brief.
by side. Choice C also has this idea and hence is most Choice (C)
appropriate to end the para with. Choice D can be ruled out
because it suggests that the new sticks out awkwardly. 5. All the options other than option A are distorted. Democracy
Choice (C) fails due to intelligence failure, is the distortion in option B. It
4. The paragraph begins with a ‘breakthrough’ and goes on to ‘is the perfect form of government’ is the distortion in option
explain what it is. Choice B can be ruled out because it is too C, and the last sentence is a distortion in option D. Option A
certain in tone. Choice C is appropriate. Choice (C) is correct and concise. Choice (A)

6. The last sentence in option A which states that God-like


36. ODD MAN OUT:
qualities neutralize the devil is a distortion. Choice C omits
the idea of the god-like and the devil-like qualities in man.
1. ‘Lawyer’ is the odd man out. The others watch the
Choice D lists out the ideals which man has held dear. This
proceedings and can give a verdict. Choice (D)
is not done in a summary. Besides the last sentence here is
also a distortion. Choice B captures all the ideas in the text
2. Reading, spelling and speaking are language abilities,
in the most concise manner. These include
whereas ‘counting’ deals with numerical ability.
(1) Man’s spirit of sacrifice to uphold ideals.
Choice (C)
(2) This spirit making up for all his failings.
(3) His challenging Nature, despite his limitations.
3. ‘Coliseum’ is an arena where sports events are held. The
(4) The divine and the devil in him. Choice (B)
others are places of worship. Choice (D)
7. The main ideas in this extract are
4. The process of saturation does not involve change of state
(1) Change in people’s attitude to jobs.
from one form to the other (liquid, gas, or solid).
(2) Companies adopting new recruitment procedures.
Choice (C)
(3) Shift from the ‘life time’ to the contractual model.
5. Slip, skid and trip are actions which are accidental. ‘Skip’ is (4) use of scientific testing tools and competency-based
intentional. Choice (C) interviews.
All these points have been covered only in option 4 without
6. Breathing is an involuntary action. The others are voluntary. any distortion. Option A and C seem to suggest that written
Choice (D) tests and interviews are no longer in use. The extract talks
about making the test and interview more scientific and
7. While the others are winds, ‘Tsunami’ is a tidal wave. comprehensive. Option B is incorrect because the passage
Choice (B) does not talk about employees’ preference for contractual
jobs. It states that employers are shifting the model. Hence,
8. Volleyball, Soccer and football are played with a ball only. option D is the best choice. Choice (D)
Baseball is played with a bat and a ball. Choice (C)
8. In this question, note that there is a hypothetical situation
9. While the other three are machines or gadgets ‘electricity’ is presented. In a summary, the details of the hypothesis
a form of energy. Choice (D) should not be mentioned. On that score, option D can be
eliminated. Options B and C convey the idea that god did not
10. Afforestation is the odd man. The others highlight threats to intend to have a perfect world. The passage only states that
the environment. Choice (B) god intended to introduce the suspense element in
everyone’s life. Option A is the correct choice.
37. BEST SUMMARY Choice (A)
1. The extract talks about the
(1) factors which are tourist attractions (cultural heritage 9. The last sentence in option A covers only a partial idea of
and beach resorts). what researchers claim about tea. We cannot conclude that
(2) how tourism helps local art and craft industry and tea drinking originated in Asia from the facts provided.
promoters access to global culture. So, option B is also eliminated. Option D fails to mention
(3) threat tourism poses to local culture and environment. Europe. Choice C is the best choice. Choice (C)
All these points have been covered best in option B and
hence is the best choice. Thailand has been cited as an 10. The ideal habitat for elephants has not been mentioned in
example and should not be included in a summary. So, option A. Both options B and C are distorted. All elephants
option A is eliminated. Option C has mentioned tropical other than Terrai Bhabhar live in vast forest lands is not true.
climate as a tourist attraction. This is a distortion. Biologists have not called for measures to save the elephant
Besides, it has omitted the threat to culture. Option D as stated in option C. Option D is the best choice.
has not specified beach resorts and the craft industry. Choice (D)
Choice (B)
38. FACTS, INFERENCE AND JUDGMENT:
2. The first option is not suitable, because ‘ironically’ conveys
an opinion which is not part of the original extract. Peace and 1. (a) is a fact because it can be proved.
happiness are examples and can be omitted. The last (b) is somebody’s opinion.
sentence of option B is an empty statement conveying no (c) This too can be ascertained and hence is a fact.
idea. Between C and D, D is more concise and is preferred. (d) is a statement which is of ‘CAUSE and EFFECT’ type.
Choice (D) If this (cause)….. then this (effect). d is an inference.
Choice (A)
3. Options C and D distort the ideas. In option C “when a
fashion becomes outdated it is a fad”. is a distortion. 2. (a) is a verifiable fact.
Fashions do not get replaced by fads as stated in option D. (b) There was a loss (cause) and it could be in billions of
Option A has omitted the idea that fashion includes rupees (inference).
behaviour. It also fails to mention that fashion keeps (c) This is the opinion of a person (personal judgement).
changing. Option B is the best answer. Choice (B) (d) This again is a scientific fact. Choice (B)
Triumphant Institute of Management Education Pvt. Ltd. (T.I.M.E.) HO: 95B, 2nd Floor, Siddamsetty Complex, Secunderabad – 500 003.
Tel : 040–27898195 Fax : 040–27847334 email : info@time4education.com website : www.time4education.com SM1001941/147
3. (a) is the judgement of a person. It is how he feels. 13. ‘Obfuscation’ is lack of clarity. All the pairs except Proclivity:
(b) Anyone can verify this statement. Hence, it is a fact. Penchant share an antonymous relationship as the
(c) One can find out whether industries are also being capitalized pair of words. ‘Proclivity’ is the tendency to
subjected to powercuts or not. This can be verified. behave in a particular way or to like a particular thing and
Hence, this statement is also a fact. ‘penchant’ also means having a special liking for something
(d) If X happens then the effect will be Y. Hence, the given or a tendency to do something. Choice (B)
statement is an inference. Choice (C)
14. The words that can fit into the first blank are proxy, substitute
4. (a) Ascertainable fact. and surrogate; betrayal cannot go into the blank because it
(b) If “objects lie scattered” there may have been means the act of cheating. And the word that can go into the
“a struggle preceding the murder”. Hence, inference. second blank is entrenched (deep-rooted). ‘Inveterate’ is
(c) is the author’s personal opinion. Hence, judgement. ruled out because it often goes with habits or feelings and
(d) (If) the safe was broken (then) the motive could not symbols. ‘Cantonal’ means regional; hence ruled out.
be……… inference. Choice (C) Choice (B)
5. (a) Ascertainable fact (one can find out whether this is right
or wrong). 15. The words ‘foreign invaders conquered and successively
(b) Author’s perception (Judgement). ruled’ and ‘while’ indicate that the blank requires an
(c) More workload, little social life (Inference). equivalent of ‘ruled by the natives’, which is ‘indigenous’.
(d) This is again the author’s opinion. (somebody else may Inborn means “existing from birth” – it is generally used for
disagree). Hence this statement is a judgement. natural qualities.
Choice (D) The Inherent qualities of something are the necessary and
natural parts of it. ‘Intrinsic’ means belonging naturally,
PRACTICE EXERCISE – 1 essential. Hence (B), (C), and (D) do not suit the above
context. Choice (A)
1. ‘Bucolic’ means connected with country or village life and the
word that means the same is ‘rustic’. Choice (C) 16. ‘Dominated’ goes into the blank as it means “having a
commanding or controlling influence over”
2. While some dictionaries provide the meaning as choice (A), To ‘tyrannise’ is to use power over someone, to threaten
other provide it as choice (D). Choice (A) is closest in them cruelly or unfairly.
meaning to the original French phrase, while choice (D) is To ‘overrule‘ means to officially decide that a decision is
the way the phrase is normally used in modern day English, incorrect. It is generally done by someone in authority.
in the context of economic policy. Choice (A) To ‘bully’ is to frighten or intimidate Hence (B), (C), and (D)
do not suit the above context. Choice (A)
3. ‘Gridlocked’ in the context means becoming powerless to act
in a way that the E.U likes to. The word that means the same 17. Passive goes into the third blank as it means “accepting or
is ‘straitjacketed’. Choice (B) allowing what happens without active response resistance.”
‘untouched’ means unaffected ‘unstirred’ also means
4. The word that can replace the given phrase is ‘laconic’, unmoved .
‘prolix’ and ‘voluble’ mean wordy and ‘succinct’ means ‘docile’ means ‘ready to accept control’, but is generally used
expressed clearly and in a few words, hence it has a positive for living things. Hence (A), (B), and (C) do not suit the above
connotation. Choice (A) context. Choice (D)

5. ‘Precipitate’ means to make something happen faster and 18. The word ‘repeat’ is wrongly used in Choice (B). It should
the opposite would be ‘retard’. If something retards a process be….. repeat after her…. that means to say what she has
or the development of something, it makes it happen at a said. Choice (B)
slower pace. Choice (B)
19. Explanation
6. The antonymous pair is b – d. Pliant means easy to influence In 1 'seemed' is appropriate since the sentence is about the
or convince while ‘obstinate’ means difficult to convince. way someone feels. 'Appeared' is used when we wish to
Choice (D) present what someone understands. In 2 'stirring' is
appropriate since the context indicates (using the words
7. The idiom ‘pull oneself by the bootstraps’ means to improve sleepiest of sleepy) that there was not even the slightest
one’s position by one’s efforts. Choice (B) movement. In 3, things are 'faint' in visibility, not 'light'. In 4,
a bird floating on lazy wing is one that is staying aloft by
8. The word ‘matter’ can be used in all 4 sentences. gliding, wings widespread but motionless. In 5 'but' is
4 times appropriate. We use 'but for' in the sense of reason
presented.
9. ‘Promulgate’ means to spread an idea widely. The word that Thus 'abbab'. Choice (B)
can substitute it is ‘promote’. Choice (C)
20. ‘So’ is followed by ‘that’ and not ‘so’.
10. The fact that capuchins manage to find ways to live in almost ‘too’ is not followed by ‘that’ and the same is the case with
any kind of habitat shows that they are adaptable (ability to ‘very’. Choice (C)
adjust to changing conditions). Choice (B)
21. There are errors in both the sentences.
11. The word ‘terminal’ means certain to come to an end. The Ι. make fun ‘of’ someone and not ‘at’ someone.
word that cannot replace it is ‘lethal’ which means ‘able to ΙΙ. Japanese should be preceded by ‘the’. The reference
cause death’. The decline is not causing danger, rather it is is to the natives of a particular country.
the habitat which is causing the decline. The remaining Choice (C)
words can replace terminal. Choice (D)
22. Option (D) is the best as there are errors in the rest of the
12. The word that can go with all the four words is ‘party’. ‘Party options
favours’ means small gifts given to children at a party; ‘party (A) the article ‘a’ is missing before ‘devotee’
line’ refers to the official opinions and policies of a political (B) the adverb ‘incessantly’ should be placed after the
party; ‘party spirit’ refers to the sort of mood in which you can object.
enjoy a party and someone’s ‘party piece’ is something that (C) definite article ‘the’ should be placed before ‘Almighty’
they often do to entertain people at parties, for example as it refers to something that is unique.
singing a particular song. etc. Choice (D) Choice (D)
Triumphant Institute of Management Education Pvt. Ltd. (T.I.M.E.) HO: 95B, 2nd Floor, Siddamsetty Complex, Secunderabad – 500 003.
Tel : 040–27898195 Fax : 040–27847334 email : info@time4education.com website : www.time4education.com SM1001941/148
23. There are errors in 30. The essence of the text is best encapsulated in choice (B)
(b) ‘beneficial ‘to’ mankind’ and not ‘for’ mankind. option (A) provides an example because of which it is not as
’Beneficial to’ and beneficial for’ are used as follows: concise, option (C) talks about the same thing as option (B)
Exercise is beneficial to you. but the word ‘imposed’ means forcing on something and
Exercise is beneficial to your health. hence it does not go with the word ‘unconsciously’. Also, ‘our
(d) ‘is’ should follow ‘suffering’. Choice (B) past experiences tend to ….’ is a repetition of the first
sentence and hence there is also a problem of redundancy
24. Explanation in option (C) Choice (B)
Line a has no error.
In line b the adverb 'diligently' is wrongly placed. It obstructs PRACTICE EXERCISE – 2
the continuity of the thought "...train ourselves to lie
judiciously...". 'Diligently' should be placed before 'train'. Explanatory notes for questions 1 to 10:
Line c has no error.
In line d the auxiliary verb 'will' is wrongly placed. Since the 1. The pronoun 'they' in ‘A’ and the opening words - 'on the
statement specifies a pre-requisite (only then), it should read other hand' and 'on the one hand' of sentences ‘C’ and ‘D’,
"...only then will we be worthy dwellers...". respectively, suggest that statements ‘A’, ‘C’ and ‘D’ cannot
Line e has an error. It needs a comma after 'lies', since the open the paragraph. ‘B’ is the only sentence that can open
following portion represents additional information (of the paragraph. Ans : BDCA
exception to the habit).
The corrected sentences are: 2. The words 'the atrocities', 'the all-Dalit village', 'the caste
"Therefore, the wise thing is for us to diligently train related clashes' are the references to 'black chapter' and 'a
ourselves to lie judiciously; ......" remote village' mentioned in ‘C’. Hence the opening
"Only then will we be worthy dwellers ...." sentence is ‘C’ not ‘B’. 'It' in D refers to the ‘writ petition’
"....where even benign Nature habitually lies, except mentioned in A. Hence CBAD is appropriate.
when....". Choice (C) Ans : CBAD
25. Option (C) expresses the idea clearly and briefly. 3. Sentence E introduces us to the idea of technological
Option (B) is unnecessarily elaborate. Option (D) does not progress and economic growth. The result of this is given in
provide all the points clearly. It is superficial. Sentence A. The effects of this growth are given in
Option (A) is very close to option (C), but is not as concise. sentences B and C. Sentence D concludes the passage.
Choice (C) While B could draw attention as a possible opening
sentence, B and A have no link. Ans : EABCD
26. (c) is the beginning of the sentence as it introduces the
idea/view. 4. B can be the opening statement. A can follow B because the
(a) follows as the word “would be taking a long theme remains ‘Russia’ and the ‘space programs’. Also, the
step”…….is a logical connection. idea of the collapse of the Soviet Union is further explained
(d) and (b) follow. in D. Hence, D comes after A and C follows D. Hence BADC
(b) has a concluding tone. is the logical sequence. Ans : BADC
Hence cadb Choice (C)
5. Statements C and D go together, because the ‘them’ in D
27. (b) is the opening statement as it introduces the topic. refers to the Britons who are not going out and spending
(d) takes the idea of ‘attacks via web’ forward and money. D and B go together because both of them deal with
explains how it happens. the government’s take on the situation. A is the concluding
(a) follows as the term ‘malicious’ that is used in (d) is sentence. Hence CDBA is the correct sequence.
explained here. Ans : CDBA
(c) gives more insight into the topic by providing more
details. 6. In E, the word ‘they’ refers to people. Hence, B which is the
(e) ‘These’ devices clearly refers to ‘laptops and mobile first sentence of the paragraph showed precede E. The idea
devices’ mentioned in (c) of moving slowly is further explained in C. A is a perfect
Hence bdace Choice (C) sequel to C. And D can be the closing statement. Therefore
BECAD is the appropriate sequence. Ans : BEACD
28. The option that best concludes the paragraph is option (C).
Option (A) takes the idea forward but doesn’t conclude, even 7. A is the opening statement since it mentions an event that is
though it seems like a conclusion. discussed in the other sentences. C follows A – ‘it’ in C refers
Option (B) strengthens the idea that light is harmful to to the broken arm in A. E explains the effect of the injury. D
animals but it does not conclude the para. further explains it and B is a good closing statement. The
Option (D) talks about advancements which are related to appropriate sequence is ACEDB. Ans : ACEDB
the issue being discussed. It does not conclude the
paragraph. 8. It is only statement ‘C’ that begins the passage by discussing
Option (C) has a concluding tone and is in line with ‘. . . grave 'Eco-tourism industry' in India. The possessive pronoun 'its' in
. . . environmental threat . . .’. Choice (C) D refers to the 'Eco-tourism' of ‘C’ and so D follows C. A,
29. The paragraph, appropriately sequenced is as follows: which explains why its track record is low, follows D. E
(D) The tortoise is a ground-living creature that lives as carries forward the idea. The demonstrative pronoun 'this' in
close as possible to the ground without being under it. ‘B’ refers to the percentage mentioned in ‘E’. Hence B is a
(A) It has about as good a turn of speed as a creature continuation of E. Ans : CDAEB
needs to hunt down a lettuce.
(C) It has survived while the rest of evolution flowed past it 9. Sentence ‘C’ which speaks of the 'rich variety of wetland
by being, on the whole, no threat to anyone and too habitats' introduces the topic. Statement B which refers to
much trouble to eat. the total area of wetlands follows C. The words “realising the
We need (D) since it presents the primary subject, the importance” in A continues the idea further. Hence CBA go
tortoise. (A) follows by telling us about its very slow pace in together. Further, ED goes together. Ans : CBAED
movement and this is the reason that it poses no threat to
anyone, the thought in (C). 10. ‘B’ is the most appropriate opening statement, since it
The statement that is to be omitted is (B): "Its horizons are a presents an analysis of presidential and cabinet systems of
few inches away". While this does tell us something about government. The words ‘To begin with’ suggest that C follows
the tortoise, it does not contribute to the purpose of the B. A continues the idea further. ‘Unlike this’ in E suggests that
paragraph. Choice (B) E follows and finally D concludes. Ans : BCAED

Triumphant Institute of Management Education Pvt. Ltd. (T.I.M.E.) HO: 95B, 2nd Floor, Siddamsetty Complex, Secunderabad – 500 003.
Tel : 040–27898195 Fax : 040–27847334 email : info@time4education.com website : www.time4education.com SM1001941/149
Explanatory notes for questions 11 to 13: Explanatory notes for questions 19 to 25:
11. One way of explaining a scientific term is to start with the 19. Clearly, C opens the author's discussion on what he dislikes.
‘known’ or ‘familiar’ and move on to the’ unknown’ or Now, 'this' in A refers to 'others' lack of planning' in C.
‘unfamiliar’ . The passage begins by introducing cloning as Similarly, 'this' in D also refers to the same. So we have a
‘making identical copy.....,’ a simple concept that the reader logical sequence in CAD. B talks about 'Indian time' which
may know. Then the concept is further explained: ‘Clones may not necessarily refer to 'lack of planning and so is the
are exact replica....’ This idea is further explained in the next odd one out. Choice (B)
sentence: ‘they represent a population of genetically
identical organism...’ A more unknown area is dealt with in 20. The topic of the passage is the fact that football produces
the next sentence: ‘this term is also applied to a population saints sometimes with the example of Manna. This means
of viruses...’ And in conclusion, the ability to clone DNA ......is that A, C and D are linked. D introduces the contrasting fact.
an important technique…. Choice (C) C brings in the main point. A concludes the discussion. B
which makes a general irrelevant statement about football is
12. The paragraph opens by mentioning the lessons to be drawn the odd one out. Choice (B)
from Steve Jobs. The next sentence adds to what is stated
in the first sentence: he asked a question expecting a 21. Clearly, the para is about why Romney triumphed over his
particular answer. The next sentence states the reply. The rivals. This is introduced in B. A explains that and D presents
sentence that follows further explains the reply. The last a contrast to that. C, by talking about 'blaming',
sentence is the concluding comment from the author: becomes the odd one as the discussion is about taking
business schools will be studying the significance of his credit. Choice (C)
accomplishment. Choice (A)
22. Clearly, A follows D ('but' connects then). B concludes the
13. d begins the passage and it talks of a present condition. It discussion. C, with an irrelevant idea is the odd one.
states that man has been ‘mutilated’. How? We go back to Choice (A)
the past to statement (a). The past tense ‘believed’ in (a) and
23. The topic is 'silly books on Success'. Clearly, BAC is the
‘was considered’ in c sum up the consequence mentioned in
logical link. D makes a general statement about success
(e). We now come back to the present and the end result is
which is irrelevant to the topic. Choice (D)
mentioned in (b) and it is the concluding statement. So the
right order is daceb. Choice (B)
24. Options B, C and D are about India’s relations with China,
but option A talks about India’s intervention in Maldives.
Explanatory notes for questions 14 to 18:
Choice (A)
14. From the choices either ‘d’ or ‘b’ may follow statement 1. ‘d’
fails to follow 1 logically. Hence choices (A) and (D) can be 25. Options B, C and D are about coaching given to employees
ruled out. The words "a late spring night" in statement 1 finds and executives and how it is taken by them. But option A
a continuation in ‘b’. a follows describing the scene further. talks about its evaluation. Choice (A)
Hence badc Choice (C)
Explanatory notes for questions 26 to 35:
15. Indeed it is ‘c’ alone that elaborates on the words 'the
26. Statement D beings the paragraph by introducing the topic
watchword in Singapore’ found in the opening sentence.
on which the paragraph is based – Aditya - 11 the Indian sun
Moreover 'the findings' (of ‘d’) are the result of an 'action
mission due after three years. Statement B follows D by
committee' referred to in ‘b’. Hence ‘d’ should follow ‘b’. But
stating a condition that the said mission would be possible
in choices (A) and (B), ‘d’ follows ‘c’ and ‘d’ cannot be the
only if the Advisory Committee on Space Sciences, has its
continuation of the opening sentence. Hence cabd.
way. Statement E follows B by stating that discussions to this
Choice (D)
effect are under way. Statement A follows E by further
16. As there is no reference to 'most of the mines' of ‘a’ in the describing Aditya and mentions the brain behind conceiving
opening sentence, it cannot be the continuation of statement and designing it. Hence statements DBEA form a sequence.
1. ‘b’ is fit to be the second sentence as it focuses 'mining' Statement C can begin another paragraph but it does not
as one of the reasons for the threats to tiger, in continuation form a logical sequence with DBEA. Ans : (C)
to the first sentence. Then ‘c’ explains why 'mines' have 27. Statement A begins the paragraph by speaking about the
become a threat and ‘a’ and d elaborate on it. Hence bcad avalanche on the Siachen glacier which buried ten soldiers
Choice (C) of the Indian army. ‘This was not an isolated incident’, in C
17. Though d and a seem to be the continuation of the opening refers to the incident mentioned in A. Hence C follows A. E
sentence at first glance, Choice A can be ruled out statement corroborates what is stated in C by citing the instance of
‘c’ cannot fit in follow B, because neither can it follow d another tragic incident. Statement D sums up what is stated
(Choice C). For, in choice (A), b says that 'a few in ACE by stating that avalanches are equally dangerous to
announcements are implemented ….. then c which follows b, both Indian and Pakistan troops. Hence ACED form a
says that the country looks forward to the budget speech of sequence. Statement B which speaks about demilitarizing
the Finance Minister. It is the minister who has to announce, the glacier, does not form a part of this sequence and is
so minister cannot follow the announcements. So also in (C), therefore the odd man out. Ans : (B)
statement d which speaks of the 'sops for weaker sections', 28. Statement B, which begins the paragraph with what
and 'concessions in income tax' should follow the Finance Mahatma Gandhi opined about the Indian cities, introduces
Minister's speech in (C). It is the minister who announces the us to main idea on which the paragraph is based. Statement
concessions or anything. Indeed, the opening words, D follows B by describing about the squalid urban
'Followed by a pre-budget economic survey' refer to the 'lot landscapes of the 21st century. ‘Since then’ in
of hype ….. presentation'. Then between (B) and (D), which D refers to the situation a century ago during the time of
start with ‘c’, answer Choice (D) is apt as the last sentence Mahatma Gandhi. Statement A is a continuation of D
(6) of the passage refers to the 'announcements' talked and C carries forward what is stated in D. Hence
about in statement b which is the last sentence in the BDAC is the logical sequence. E which brings in another idea
sequence in (D). Choice (D) – environmental governance is the odd one out.
Ans : (E)
18. ‘b’ cannot follow ‘1' because there is nothing in ‘1’ to connect
with the word ‘Then'. ‘d’ justifies what is stated in ‘1', thus ‘d’ 29. Statement E states the main topic – i.e. the Munich
should follow ‘1'. ‘b’ connects with ‘Then', by elaborating agreement which is intended to end hostilities in Syria.
further. ‘a’ then states a fact and ‘c’ connects with the word Statement C follows E by pointing to the indifference which
‘But'. Thus choice C is the answer. Choice (C) the World displayed, for many years, to the war situation in
Triumphant Institute of Management Education Pvt. Ltd. (T.I.M.E.) HO: 95B, 2nd Floor, Siddamsetty Complex, Secunderabad – 500 003.
Tel : 040–27898195 Fax : 040–27847334 email : info@time4education.com website : www.time4education.com SM1001941/150
Syria. A and D, in that order, bring out the gravity of the 2. Only statement (b) can begin the para and it mentions “two
situation in Syria. Hence ECAD is a sequence, Statement B reforms” which are detailed in (d) and (a) respectively. What
could start another paragraph but does not form a sequence happens if rural areas are supplied power at night? This is
with ECAD. Ans : (B) mentioned in statements (e) and (c) in that order. The
practice in (e) is the result of ‘a notion’ in (a). Also price-
30. Statement B begins the para by stating how the sky was realization by the farmer is the last step leading to place (c)
being observed with optical telescopes, during the time of as the conclusion. Choice (A)
Galileo. E follows B by mentioning the different wavelengths
of the electromagnetic spectrum which were a result of the Explanatory notes for question 3:
advances in technology. A carries forward the paragraph by
talking about other advancements which enabled scientists 3. B holes being revealed is unlikely to begin the paragraph,
to gain new insights into the working of the universe. given that the cause for the craters have to be discussed first
Statement D concludes by mentioning about neutrinos which as in D. EA makes a mandatory pair of the drunk and the
have been discovered in the last few decades. Hence BEAD sober. D describes the current scenario in Chicago. BC also
make a paragraph. Statement C does not form a part of this makes a mandatory pair with discussions of budgets and
sequence. Ans : (C) states with and without money. Ans : EADBC
31. Statement D brings out the main idea i.e. the double
Explanatory notes for question 4:
standards faced by men and women. ‘This’ in B refers to the
double standards mentioned in D, hence B follows D.
4. Options A, C and D talk about the effects of receding glaciers
Statements A and E, in that order, go on to explain how the
and a step taken to counter one of them while option B is not
double standards show up in language. Therefore DBAE
directly connected to that topic. It talks about deforestation.
form a logically coherent paragraph. Statement C which
Choice (B)
brings in another idea is the odd man out. Ans : (C)
32. Statement A opens the paragraph by mentioning a well Explanatory notes for question 5:
known fact. C, which states how alcohol destroys the liver of
habitual drinkers is a mystery, presents a contrast with what 5. Statement C which sets the tone of the paragraph is ideally
is stated in A, E is a continuation of C. B follows C by stating the opening sentence. Statement E which explains how
that there is another cause for liver damage. Statement D, Africa, captures the romance of the early days of flight is a
which does not form a part of the sequence ACEB, is the odd continuation of E. Statements D and B, in that order, give
man out. Ans : (D) examples of the charming anachronisms discussed in E.
Hence statements CEDB form a logically coherent
33. Statement C, which gives an introduction to the subsequent paragraph. Statement A, which speaks about airlines flying
sentences, is the opening sentence. Statement B follows C new routes, does not form a sequence with CEDB and is
by describing the present situation, stating that now things therefore the odd man out. Ans : (A)
are not as bad (as they were in the 1940s) but Los Angeles
is still the worst city in the U.S. with reference to pollution. PRACTICE EXERCISE – 3
Hence C and B form a pair. Statement E follows B by
corroborating what is stated in B as it describes the situation Explanatory notes for questions 1 to 35:
in the southern part of the state. A follows E as it is an
extension of what is stated in A. Hence statements CBEA 1. Choice (C) is the most appropriate as the concluding
form a logical sequence. Statement D which brings in statement because it sums up the para. The first three
another idea is clearly the odd man out. Ans : (D) sentences state that industrial societies are obsessed with
economic growth because they think it is a sign of progress
34. Only statement A can begin the paragraph as each of the
and they believe that globalization is an effective way to achieve
other sentences have to be preceded by another sentence.
economic growth. Hence mass production and mass
Statement D follows A by giving the reason why Sri Lanka
transportation of food has become a normal practice.
seems to be sitting pretty. Statement C carries forward what
Therefore choice (C) is the most logical when compared to
is stated in D. Statement E follows C by giving one more
the other options. Choice (C)
reason why Sri Lanka seems to be sitting pretty. Therefore
statements ADCE form a paragraph. Statement B which 2. Option (A) is the most appropriate statement to follow the
presents an idea which totally contradicts what is stated in second statement because the second statement states that
ADCE is the odd man out. Ans : (B) the inefficiency of modern industry surpasses one’s ordinary
35. Statement C talks about the genes, which most of us have, powers of imagination, and Choice (A), which says
that make us as hardy as dandelions. Statement A follows C “therefore the inefficiency of modern industry remains
as it carries forward the same idea by comparing a few unnoticed”, sums up the para. The remaining options are not
people to orchids. ‘So holds a provocative theory of genetics’ related. Choice (A)
in B, throws light on what has been discussed in C and A. 3. The sentences which precede the blank show that modern
Statement E explains the ‘adaptability and evolutionary communication techniques made communication easier and
success discussed in B, by giving the example of orchid helped in developing relationships that do not require face-to-
children, who, with the right environment and good parenting face contact. Choice (B) sums up the paragraph by saying that
can become successful and happy people. Hence community is being stretched over a wider area, thanks to
statements CABE form a logically coherent paragraph and modern communication techniques. The remaining options
statement D talks about something which is entirely different are not proper concluding statements. Choice (B)
from what is being discussed in CABE. Therefore D is the
odd sentence. Ans : (D) 4. The sentence which precedes the blank states that employers
favour long working hours. The sentence which follows the
EXPLANATORY NOTES FOR ADDITIONAL QUESTIONS
blank states that long working hours have a detrimental effect
Explanatory notes for questions 1 and 2: on employees. Option (B) which says that employees may not
work with required concentration due to the long working
1. The paragraph narrates an incident. Statement c begins the hours, ideally follows the second statement and precedes the
narrative. Statement a comes next in the sequence as it last statement which says that employees find it difficult to
carries forward the narrative. ‘Their suggestions’ in b refers maintain a work-life balance due to long working hours.
to the suggestions which people made. Hence b is a Although option (A) may follow the second statement, it cannot
continuation of a. ‘them’ in d refers to the suggestion made logically precede the last statement and thus does not serve
by people. e is a continuation of d. Hence cabde is the logical as a proper link between the two statements. The remaining
sequence. Choice (D) options are not connected. Choice (B)
Triumphant Institute of Management Education Pvt. Ltd. (T.I.M.E.) HO: 95B, 2nd Floor, Siddamsetty Complex, Secunderabad – 500 003.
Tel : 040–27898195 Fax : 040–27847334 email : info@time4education.com website : www.time4education.com SM1001941/151
5. The first sentence points out to the increasing focus on well- point; a society obsessed to control the future in (B) and
being and longevity in India, the subsequent sentences science and technology generating new uncertainties in
illustrate this with examples. Choice (C) sums up the para by (D) are unrelated to the paragraph. Choice (C)
saying that these signs point to a society that is growing
healthier by the day. The remaining options are not related. 12. Blood pressure medication used by people over 70, causes
Choice (C) dizziness, which is often a culprit in falls. A recent study has
found that serious fall injuries were higher for people who
6. The significant idea here is that of 'paradoxical questions' took hypertension drugs. What can logically conclude this
followed by : (a colon), towards the end of the paragraph. paragraph? Of the choices given, A is the ideal one – Given
One of these questions is in place, and would therefore be these findings, it is advisable that elderly patients discuss
followed by another - paradoxical, at that. (A) is inappropriate their drugs with their primary care physician. This statement
because neither is it paradoxical, nor is it in tune with the is well related to the preceding sentence. B can be ruled out
earlier thought that writers put in 'immense effort' to help the since diuretics (substances that tend to increase the flow of
readers 'have access'. (B) is inappropriate because it is not urine) is not related to the argument. Option C cannot be
a paradoxical question. (C) is appropriate – here's the related to the preceding sentence. ‘Activity’ is a new point
paradox – in trying to convey what their imagination and that has not been referred to earlier in the paragraph, so D
creative impulse have generated, are writers actually failing can be eliminated. Choice (A)
('betraying') these elements? (D) is inappropriate as this is
not in tune with the earlier thought, that indicates that writers 13. The paragraph in general is critical about disgruntled
really want to make the connections with the readers. workers and argues that one must put some energy and some
Choice (C) enthusiasm into one's work. (A) repeats the paragraph. (B)
oversimplifies the paragraph as it stands. (D) is important but
7. The significant words here are 'those individuals that
it introduces a case rather than close the paragraph. (C) with
survive....', which implies that those not included in the
‘but while’ matches the tone of the passage with ‘Of course’,
thought of this sentence, do not survive.
apart from reinforcing the message of the paragraph it's a
(A) which talks of surviving, one way or the other is not
crying shame when it, is viewed as just a way to make a living
appropriate. (B) is appropriate – refers to the extinction
and nothing more. Choice (C)
of those not included in the previous thought. (C) is
inappropriate for the same reason.
14. The paragraph talks about the importance of transparency
(D) is inappropriate because the paragraph does not
and reproducibility in science. Option A concludes the
evaluate or compare species on their chances of
paragraph stressing the importance of these things in
survival, it only lays down a very basic difference
counting a scientific observation as a discovery. Option A
between those that suvive and those that don't.
goes well with the rest of the paragraph. Option B is an
Choice (B)
entirely different point. B talks about ‘policies’ which are not
8. The paragraph focuses on the sense of illusion that glamour mentioned in the paragraph. ‘Its’ in D has no clear
exudes and points to the requirement to sustain the illusion antecedent. Choice (A)
– mystery. The paragraph would best conclude with a
continuation from the idea of mystery – how it is brought 15. Pay attention to the sentence preceding the blank. It says that
about, or what role it plays. (A) is inappropriate because it in an inter-connected world, actions in one place trigger
explains glamour in the aspect of the focus it draws, rather consequences elsewhere. Option C mentions the effect of this
than its mystery. (B) is inappropriate as there would need to situation, so C is the best concluding sentence. Option A can
be an intermediate thought to explain how mystery probably come after C, not before it. The paragraph talks about
contributes to amelioration. (C) is inappropriate – also needs a trend seen worldwide, so confining the situation to India alone
an intermediate thought to explain how mystery contributes doesn’t appear quite logical, so option B is not as suitable an
to pleasure or inspiration. (D) is appropriate because it option as C. Option D is an easy elimination because it is a
explains what contributes to the sense of mystery, and how repetition of a point in the paragraph. Choice (C)
this this impacts on the viewer. Choice (D)
16. Options A and C have no relation to the sentence before the
9. The theme of the paragraph is that corruption in Russia has blank, and these options mention the same point in different
brought the public to the point where they fear the very words. Option D mentions a new point that is not related to
institutions they should have faith in. (A) which sums up is those stated in the paragraph. Option B is continuation of the
appropriate – the irony of the situation is summed up in the preceding sentence. Ans : (B)
example of corrupt law enforcement agencies hence it is the
appropriate conclusion of the paragraph (B) is inappropriate – 17. What logical conclusion can be supported by the paragraph?
the statement only sums up a police state and does not Option A is a premise given. The paragraph says that in the
necessarily relate to corruption. (C) is not as good a conclusion past selection played a major role in the survival of the
as A though it does draw from the irony of the situation. (D) is human species. But today with changing lifestyles it is not
inappropriate because this is an example of a criminal the case. Option B sums up the argument by saying that
organisation functioning in criminal ways. Choice (A) extreme selection is a thing of the past. C is a general
statement that has little relevance here. Option D goes
10. From the way the paragraph starts, we understand that the against the grain. Ans : (B)
event was of considerable significance and meaning for the
entire French nation. From the words 'thus began the 18. The passage begins by expressing the general decadence
Dreyfus affair...' we understand that the context would then in all fields during Charles’ rule. But as we proceed, it tapers
move on to how the affair developed, or ended. (A) is down to decadence in ‘art’ and ‘noonday’ in America, which
inappropriate because it does not follow the train of thought means the peak period of art in America. Hence only choice
discussed above. (B) is inappropriate because it does not (C) follows as the last sentence and it is understood that
focus on the French nation, and discusses anti-Semitism with American artists excelled in ‘Baroque art’. Statements (A)
relation to all of Europe. (C) is appropriate – tells us in what and (B) are generalized and (D) is negative and does not go
manner the affair continued, and with what impact on the with ‘noonday in America’. Ans : (C)
nation. (D) is inappropriate as it talks only about Dreyfus and
not about the nation affected by the affair. Choice (C) 19. The crux of the paragraph is that things are likely to get
11. The paragraph says that pandemics have always existed, worse before getting better. Then look at the penultimate
but what is remarkable is that today the attitude of people sentence which begins with ‘Though the central bank has
towards them is one of panic. The short statement ‘Panic tempered its warning with the observation that risks…’. This is
itself is a pandemic’ (option C) is a perfect fit as the a clear indication that bad loans should be tackled
concluding statement. Option A talks about an unrelated immediately. Option B, which talks about the necessity of

Triumphant Institute of Management Education Pvt. Ltd. (T.I.M.E.) HO: 95B, 2nd Floor, Siddamsetty Complex, Secunderabad – 500 003.
Tel : 040–27898195 Fax : 040–27847334 email : info@time4education.com website : www.time4education.com SM1001941/152
tackling NPAs (Non Performing Assets), is the right option. 28. The paragraph ends with the observation that something is
The paragraph doesn’t suggest any ‘multi-pronged action’, so odd. What is odd? That only 3 craters are found on Earth.
A is ruled out. Options C and D talk about points not related to Why is it odd? The answer is given in (D), if many big
the ones mentioned in the paragraph. Ans : (B) asteroids would have hit the Earth in the distant past, where
have all the other craters gone? (A) arrives at a conclusion
20. Based on the facts given, the ideal conclusion is that a lot of without a proper argument as such. (B) does not follow as
retirement money will be invested in stock markets. So an oddity. It is rather an inference. (C) does not clarify which
option D is the answer. Option A talks about a related point; craters are being referred to. Ans. D
B has no relation to the paragraph; and it is not clear what
the precedent of ‘this’ in C is. Ans : (D) 29. (C) and (D) are poorly substantiated, the specific reasons for
Vietnamese ire are not clear yet. (B) mentions a
21. The only option that explains the sentence preceding the geographical imperative that can, at best, leave the
blank is A. If the words in the message were the same that paragraph dangling. (A) emphasizes the Chinese point of
he would have uttered as finance minister, it means the view with reinforcements announced that …. To make that
message was not different, but only the author of the perfectly clear. Ans. A
message. Option B is irrelevant to the paragraph – it is just
an observation. C too is irrelevant to the points in the 30. The second half of the penultimate sentence explains a
paragraph. Option D is unwarranted in the context of the retrospective word formation ‘tobacco cigarette.’ Option A
passage. Ans : (A) completes the explanation, so A concludes the paragraph.
Option B is irrelevant as ‘technology enhanced drug-delivery’
22. The sentence before the blank talks about a person being in is not referred to in the paragraph. The precedent of ‘this’ in
control of his life as the foundation of stress management. C is not clear – vamping or tobacco cigarette. Option D is
This point is illustrated in option D, so D is the right choice. totally unrelated to the paragraph. Ans : (A)
Option A doesn’t go with the preceding sentence as it is
confined to just recognizing stress. B talks about locating EXPLANATORY NOTES FOR ADDITIONAL QUESTIONS:
stressors; C mentions avoiding stressful situations as the
solution. Both are not related to ‘being in control of your life’ 1. What can be the best concluding sentence out of the four
stated in the penultimate sentence of the paragraph. options? The penultimate sentence says that anthropogenic
Ans : (D) (relating to the origin and development of human beings)
causes are the sole contributor. Option B concludes the
23. The author states that empathy is not unwillingness to take paragraph asserting the effect of human actions. All other
tough decisions for fear of hurting others. The idea that can options talk about general points having no relevance to the
go with this is C which states that it simply means that they penultimate sentence. Choice (B)
are aware of, and take into consideration the impact on
others. Option A talks about ‘assessing’ co-workers, which is 2. The paragraph is about a trait of people with ADHD.
not directly related to taking tough decisions. Options B and Compared with others, they have sluggish brain reward
D do not go with the paragraph – ‘empathy’ neither agrees circuits, so they find everyday life experience dull and are
with co-workers nor brings people together. Ans : (C) impatient and restless with daily routine. Option C
summarises the argument and concludes it; it says their
24. The paragraph says that though the queen is above politics, problem is that they have a set of behavioural traits that do
when it comes to important issues facing the UK she isn’t. not match others’. A is too general and talks only about lack
Before the Scottish referendum, she urged voters to think of focus. The focus of B is on things – a thing that stimulates
very carefully before casting their vote, and the strategy one may appear dull to another. Option D is rather advisory
worked. Now which sentence can complete the paragraph? and does not go with the paragraph. Choice (C)
‘The code worked’ should be followed by a positive
statement about the UK. So, option C is the answer. All other 3. The paragraph is about three surging forces simultaneously
options cannot go with ‘The code worked’. Ans : (C) affecting our planet. These forces exert tremendous
pressure on our atmosphere and environment. Going by the
25. The passage says that conditions are totally unfavourable to writer’s style and nature of argument, option D is the right
life in many parts of the cosmos. Look at the figure given in concluding sentence. This sentence sums up the argument
the penultimate sentence of the paragraph - one millionth of precisely and beautifully - we’re in the middle of three
one billionth of 1 percent of the material of the universe exists “climate changes” at once: one digital, one ecological, one
in living form. What can we conclude from this? Of the given geo-economical. Option A is irrelevant as it talks about the
options, A is the only conclusion that can be drawn: we and effects of the forces in various countries; Americans’
all life are the exception to the rule. B and C are too general anxieties have no relation to the argument. ‘Immigration
to go with the remaining paragraph. As for D, though we can policies’ is not referred to in the paragraph, so B is
be considered lucky considering the condition in other parts inappropriate. ‘Resilience’ has no place in the argument;
of the universe, the word ‘luck’ doesn’t suit ‘a cosmic hence C can be eliminated. Choice (D)
perspective.’ Option A is the best choice.
Choice (A) 4. Option B is the correct answer choice because it mentions
the connection between ‘warmer summers and higher ozone
26. The paragraph ends with a comparison of heritage and reading’ mentioned in the penultimate sentence. A is inapt
novelty and heritage gains the upper hand. Hence, it is best as it talks about keeping ozone pollution down, which has no
to play it safe, and go along with (A). The caution in (B) and connection with the preceding sentence. C is an entirely
(D) do not seem to be warranted if the more room there will different point. Option D also has no direct relevance to the
be for old-fashioned industries. (C) essentially repeats the point discussed in the previous sentence.
argument. Ans. A Choice (B)

27. (A) is overly optimistic as there is no indication in the 5. The penultimate sentence of the paragraph talks about more
paragraph to this effect. (B) is an unwarranted conclusion. than one sense getting involved when we look at food.
The passage only mentions predicting the performance of The option that completes this point is D, which mentions
stock markets and that sunshine is good for stock markets. taste buds and tummy. Nothing in the paragraph points
(C) also cannot be concluded from the paragraph. The to confusion, so option A is ruled out. Option B is quite
penultimate sentence states that Economists have long irrelevant to the paragraph. Option C talks about two senses
known that sunshine is good for stock markets. Hence, it is getting smashed up, but nothing in the paragraph
only apt to conclude that the economists in question have suggests this, Ans : (D)
been blessed with new scholarship as in (D). Ans. D

Triumphant Institute of Management Education Pvt. Ltd. (T.I.M.E.) HO: 95B, 2nd Floor, Siddamsetty Complex, Secunderabad – 500 003.
Tel : 040–27898195 Fax : 040–27847334 email : info@time4education.com website : www.time4education.com SM1001941/153
PRACTICE EXERCISE – 4 8. The paragraph states two points: jump in the sale of mobile
phones and their misuse. Option A covers only one point,
Explanatory Notes for questions 1 to 10: and that too with a slight distortion. The paragraph doesn’t
support option B; ‘only country’ is not suggested in the
1. The paragraph is about the laidback attitude of the
paragraph. Option C covers both the points in the paragraph.
governments towards senior appointments in regulatory
The projection in option D can be an inference, not
bodies and apex institutions. Appointments are made at the
a summary. Choice (C)
eleventh hour. This causes inconvenience to the appointee
and can affect the functioning of the bodies. Option B 9. The paragraph states that biologists have been warning
summarises the idea contained in the paragraph succinctly. about diseases like Zika for years. Climate change has
Option A is too general as it is silent about who does the forced people to move around and their pathogens have
appointments and where. ‘Creating a vacuum’ in option C come into contact with species with no resistance. Increased
finds no support in the passage. Option D is too general and movement of people and climate change have had great
has no relevance to the paragraph. Choice (B) influence on the spread of infectious diseases. The ideas in
the paragraph are best summarised in option B. The second
2. All the information given in the paragraph points to the part of option A is not supported by the paragraph. Option C
distinctive nature of the National Park: it is a hotspot of has no relevance to the contents of the passage. Option D is
biological diversity; it is home to many endangered species a partial summary. Choice (B)
etc. All these point to option D - The Kudremukh National
Park is unique in its services to biodiversity and ecosystems. 10. The paragraph says that the Syrian implosion could have
Option A distorts the central idea of the paragraph, been avoided if the government had not used force to
suggesting there has been an attempt to underestimate the supress popular political demands, which in any other
Park’s contribution. Option B covers only one point stated in democratic country would have been considered legitimate.
the paragraph. Option C can also be eliminated on the same Option A summarises this idea concisely. India is cited as an
grounds as in B. Choice (D) example to suggest a popular movement that took place
there, so a comparison is not intended. This eliminates
3. The paragraph describes the position of the rich in America. option B. Option C is distortion of the contents of the
Clearly, unfairness exists there, as per the instances stated paragraph as there is no reference to bloodshed. Option D
in the passage. Option A is ruled out as ‘exploitation’ is not is not supported by the passage. Choice (A)
suggested in the paragraph. ‘Money exerting influence’ in B
is also beyond the paragraph. Nothing about the past status Explanatory Notes for questions 11 to 20:
is indicated in the paragraph, so ‘remarkable rise in
11. The passage essentially mentions that about 270m people
inequality’ in option C cannot be considered. Option D
from China are moving from one area to another in search of
summarises the paragraph; ‘structural unfairness’ in the first
work and are leaving their children under someone's care. Only
sentence and the examples cited in the paragraph point
choice (C) brings out all the features in the apt manner. In (A),
to this. Choice (D)
the movement of people from rural areas is not mentioned.
4. The paragraph attempts to trace the reason why so many Also, the number of people that have moved has not been
people feel the need to be constantly recognised and specified. These two points make the choice incorrect. In (B),
flattered. It is likely that this behavioural trait has developed the entire list of guardians is not mentioned, making it incorrect.
in them due to their childhood experience of excessive love In (D), the use of Chinese people is incorrect. There could be
followed by neglect. Answer option B summarises this – a people of different nationalities in China who have moved from
personality of narcissism (self-love) is caused by childhood one place to another. Choice (C)
experience of excessive attention followed by neglect by 12. The paragraph essentially implies that George W. Bush
unregulated parents. Option A is advisory in nature, and cracking jokes and being witty about his lack of intelligence,
doesn’t make an apt summary. Option C is too general as it reminded people about his presidential stand up act. This
talks about psychological disorders. Option D is not indicated imposed a question on the kind of work he had done for the
in the paragraph. Choice (B) eight years that he was a President and how it never seemed
5. According to the paragraph, people today spend a lot more like it was fruitful during that tenure. Only choice (B) brings
time sitting than our ancestors. They spend huge sums to out all of these points clearly. Choice (A) is incorrect as the
ensure that they don’t get damaged due to excessive sitting. years did not go unnoticed. Rather, they were in the limelight
Still, there are chances of fatal injuries due to long sitting. as they were a question on the execution of his work. Choice
Option A summarises the contents of the passage concisely. (C) is incorrect as the choice mentions his absence. No
Option B focuses on moving, which is contrary to the where in the question does it mention or can be inferred that
argument. Regular ambulation (walking) is not suggested by he was absent from his duties. It only says that there was a
the paragraph. ‘Movement being integral to human survival’ reminiscence of how his duties were executed during his
is also not related to the paragraph. Choice (A) tenure. Choice (D) is incorrect as we do not know if he was
inefficient as a President overall or if he was lacking in the
6. The paragraph focuses on accidents involving selfie-takers. execution of his duties. Choice (B)
The reason for the accidents, according to the paragraph, is 13. The passage talks about a meeting that was held in
people’s craze for photography, triggered by the development Bengaluru in which innovative ways of developing new
of mobile phones with cameras. Option C best summarises the products was discussed. In addition to this, ways and
contents of the paragraph. Option A is a very general methods to combine traditional business with startups was
statement. Option B is a distortion because the paragraph also the theme. These are the only two main points of the
doesn’t point to ‘accidents involving photographers on the meeting. Choice (D) is the correct choice as it covers both
rise’. Option D is inapt as a summary as it is a very vague the points aptly. Choice (D)
comment on photography. Choice (C)
14. The important points in the passage are that the Central
7. The paragraph says that simple mutations in cell division Goverment reached a conclusion without proper analysis
cannot be the solitary cause of cancers. External causes like about a certain issue. Only choice (A) brings out all the points
diet, tobacco and alcohol are also responsible for many in the apt manner. Choice (B) is incorrect as the Central
cancers. These ideas are best summarised in option B: Government was not inconclusive but the floor test was
although some rare cancers can be driven by genetic inconclusive. Choice (C) is incorrect as the passage does
mutations, the most prevalent diseases are down to extrinsic not suggest if the Congress government had lost or won. The
factors. A distorts the ideas in the passage. Option C is not passage only suggests that the Congress Government had
entirely true; ‘bad luck’ (simple mutation) also causes lost its majority. Choice (D) is incorrect as it does not indicate
cancer. D is ruled out on the same grounds as in C. who or what Nabam Tuki is or on what grounds the
Choice (B) President's rule was established. Choice (A)

Triumphant Institute of Management Education Pvt. Ltd. (T.I.M.E.) HO: 95B, 2nd Floor, Siddamsetty Complex, Secunderabad – 500 003.
Tel : 040–27898195 Fax : 040–27847334 email : info@time4education.com website : www.time4education.com SM1001941/154
15. The main points of the passage are that Make in India and of the chemicals released. Choice (C) cannot be a
Digital India may require manpower for research or any other summary because it is verbose; Choice (D) is also ruled
job type. Also, Sri Lanka would benefit from this opportunity out as it leaves out the aspect that smile makes one
as they can provide the manpower, but it may not be only feel good. Choice (A)
monetary benefit. Only choice (A) highlights these points
well. In choice (B), the words “which may vary” are incorrect 22. The important points that the passage deals with are:
as they imply that the jobs can change, and not that there 1. Realty is thriving in Singapore.
were different kind of jobs. In choice (C), only monetary 2. Millionaires are queuing up to buy high-end
benefits are highlighted. This is not correct as we cannot properties.
conclude this from the passage. In choice (D), “all the 3. Of them, Indian millionaires are becoming dominant
manpower” is incorrect. Choice (A) with rising contribution.
16. The passage mentions that BlackBerry's move to software Choice (B) has these points. Choice (A) is wrong because it
has borne fruit as the company has given some positive says Indian millionaires are “the most dominant” but the
pointers indicating growth. Only choice (C) mentions all passage says that they are among the most dominant.
these points correctly. In choice (A) the use of “when the Choices (C) and (D) are ruled out because it is not clear
company reported” is wrong. This gives an idea as if the whether 'contrary to…' refers to queuing up or acquiring
move to software was because of the good stocks, which is high-end properties. Choice (B)
incorrect. In choice (B), the switching to software point has
not been mentioned, making it incorrect. In choice (D) “its 23. The important points in the passage are:
first quarter-to-quarter revenue which was due to higher 1. Production and distribution are basic economic
stocks” is incorrect as this was not the reason for the “first activities.
quarter-to-quarter revenue”. Choice (C) 2. They change their institutional features under the
changing social conditions.
17. The passage mentions that Arun Jaitley argued that privacy 3. Under feudalism, agriculture was the most dominant
is not a fundamental right and the Lok Sabha legislature's productive activity and distribution played more
primary objective was the delivery of benefits, subsidies and important role than ownership over land.
services to the people. He also said that gaps or flaws in law 4. Under capitalism, a small section of the people capture
can be improved at any time. Only choice (C) highlights all means of production and control others.
these points effectively. Choice (A) is incorrect as it does not Only Choice (A) has these points. Choice (B) is wrong
mention any of the primary objectives. Choice (B) is incorrect because it says capitalism aids a minority, which distorts the
as Lok Sabha or House is not mentioned at all. Choice (D) is meaning of the passage because capitalism as a concept
incorrect as the use of “primary objectives towards people” cannot do that. Choice (C) is ruled out as it leaves out
is incorrect as per the passage. Choice (C) important points. Choice (D) is also wrong because it says
the means of production help a minority to control others but
18. The passage mentions that while most animals hate cold and
the passage says it is minority's capture of the means of
find ways to escape from it, some animals can bear the cold.
production that helps. Choice (A)
Only choice (B) brings out these ideas effectively. Choice (A)
is incorrect as it does not mention the methods and ways in
24. The most important points expressed in the passage are:
which some animals escape the cold. Choice (C) is incorrect
(1) Real India does not reside in villages any more, it
as the 'them’ refers to animals who embrace cold. If the other
resides in cities.
animals are mentioned, the pronoun “themselves” should be
(2) Cities offer many opportunities and a variety of
used. Choice (D) is incorrect as the use of “some animals”
attractions.
and “most animals” has been interchanged, thereby
(3) Urban India contributes most to the nations’s GDP. The
changing the idea of the passage. Choice (B)
focus of the passage is essentially on real India residing in
19. The passage talks about one of the objectives of “Make in cities now. Only choices (C) and (D) reflect this idea. Again
India” and the steps which could be implemented to realize choice (D) can be eliminated because it has repetitive
this objective. Only choice (A) covers all the points. Choice (B) statements. All the above mentioned points are focused in
is incorrect as it talks about “major objective” and not one of option (C). Choice (C)
the objectives. Choice (C) is incorrect as “major improvement”
is incorrect as per the passage. Also, the reason for doing any 25. The passage essentially says that for an effective change in
of this is not clear. Choice (D) is incorrect as it is too far fetched the format of a newspaper the publisher should:
a conclusion. Choice (A) (1) identify its core reader group
(2) cater to the needs of the group
20. The passage mentions that there is not only a requirement (3) keep them informed about the benefits of the format.
to return indigenous artworks by ethnic groups and cultures These three are expressed only in Choice (C). Choice (A)
to their home countries by the West, there is also a growing is long-winded. Choice (B) is eliminated because it
consensus from different people to “share” global heritage includes the example which is not important. Choice (D)
more fairly. Only choice (B) brings this out well. Choice (A) is is ruled out because it does not give complete
incorrect as it does not mention that the people who want the information. Choice (C)
artworks back are countries and ethnic groups. It also
incorrectly mentions that the same people want to share art. 26. The focus of the passage is on the manufacturing ability of
Choice (C) mentions “countries world over” and “rest of the China, and how it gets affected by US economy. Choice (B)
people”, both of which are incorrect. Choice (D) is incorrect does not convey the idea properly. Choice (C) is long-
as it mentions the artworks to be returned to West and not winding. Choice (D) conveys the idea that the U.S. controls
returned by West and it also mentions expansive manner China’s economy. Choice (A)
which is not present in the original passage. Choice (B)
27. Option (B) does not qualify as a best summary because it
Explanatory Notes for questions 21 to 30: mentions examples of major scientific breakthroughs like the
21. The given passage has the following important points: ‘human genome project’. Option (C) is incorrect because it
1. Dacher Keltner has specialized in the study of facial distorts the idea of the original passage by mentioning that it
expressions. is actually the personal attributes of the team leader which
2. A smile is one of the most useful tools of human are responsible for the discovery. Option (D) cannot be
behaviour because it helps build kinship, called the best summary because it does not mention that
strengthen social relations and makes one feel good. the junior members in the scientific team are excluded from
Only Choice (A) has these points. Choice (B) leaves out laurels and rewards. Thus Choice (D) does not capture all
the positive effects that a smile produces and only talks the significant points in the passage. Choice (A)

Triumphant Institute of Management Education Pvt. Ltd. (T.I.M.E.) HO: 95B, 2nd Floor, Siddamsetty Complex, Secunderabad – 500 003.
Tel : 040–27898195 Fax : 040–27847334 email : info@time4education.com website : www.time4education.com SM1001941/155
28. Option (A) does not qualify as the best summary because it 3. The passage is about the role played by SPINK1 gene
mentions trivial details and examples. Choice (B) cannot be mutation and PRSS1 gene mutation in the development of
the best summary because it does not mention that doctors pancreatitis. It was found that the onset of the disease was
are unable to extend the best treatment to all because earlier in individuals having only SPINK1 gene as compared
modern medicine is expensive and best treatment cannot be to those having only PRSS1 though mutation of PRSS1 gene
available to all due to paucity of funds. Option (C) is not a was found to be playing a role in disease pathogenesis
best summary because it mentions that the best treatment is (development of the disease) in Indians. This argument is
available to only those who are wealthy, thereby distorting summarised in option C - SPINK1 gene mutation is the
the original passage. Choice (D) predominant cause of pancreatitis disease in Indians. Option
A goes against the paragraph. Option B is a distortion since
29. The essence of the paragraph is that PRSS1 gene mutation doesn’t cause the disease; it only helps
(1) Citizens normally consider their actions insignificant in in its development. Option D goes against the passage.
the running of the state. SPINK1 gene is not related to digestion. Choice (C)
(2) Gandhi debunked this assumption and felt that citizens
should understand that their actions, though 4. According to the passage, even though the Indian economy
insignificant when compared to the mighty state, is growing, businesses are worried about the unavailability
sustains it. of skilled workforce as our engineering curriculum does not
(3) Hence they must feel responsible for their actions. suit our needs. Option A is incorrect as a summary because
These ideas have been captured in choice (B). Choice (A) is the focus of the passage is on unskilled workforce, and not
incorrect as it mentions that citizens act irrationally. This is on improving engineering curriculum. The passage does not
not in agreement with what is mentioned in the text. Choice point to any disillusionment of Indian businesses; they are
(C) omits mentioning that the citizen is responsible for his only concerned or anxious; so option B is incorrect. Option
actions. Choice (D) erroneously mentions that citizens are C summarises the main points of the paragraph. Option D is
aware that the state cannot be run without their participation. a general statement about education sector which is not
Choice (B) relevant to the paragraph. Choice (C)

30. The main points in the paragraph are the following. 5. The passage mentions that the Indian government will not
(1) Detente does not happen by imposing conditions and only continue to pursue the Pathankot investigation, but it will
ultimatums. also make it clear to the Sharif government that they would
(2) A constructive approach must be adopted in order to have to show sincerity as the maximum attacks come from
solve all contentious issues. Pakistan. Only choice (D) brings this out well. Choice (A) is
(3) The less contentious issues should be tackled first and incorrect as the Indian government is not cautious of the
then the more complex issues should be attended to. attacks, but condemns them. Choice (B) is incorrect as “back
(4) Recent developments hint at a détente between India to being engaged” goes against the idea of the passage.
and Pakistan. Choice (C) is incorrect as “continuing to engage” goes
These ideas have been captured in choice (A). Choice (B) is against the idea of the passage. Choice (D)
incorrect as it says that the more complicated issues should PRACTICE EXERCISE – 5
be tackled first. Choice (C) is incorrect as it seems to
Explanatory notes for questions 1 to 10:
incorrectly imply that détente can adversely affect people to
people relations. Choice (D) fails to mention the third point 1. The choice of the opening sentence is between (1) and (2).
mentioned above, and hence is not comprehensive. (1) can be ruled out when we look closely at the two
Choice (A) sentences because (1) explains the importance of
‘consistency’ mentioned in 2. So 2 has to be the opening
EXPLANATORY NOTES FOR ADDITIONAL QUESTIONS sentence. (1) follows (2). Sentences (3) and (5) constitute
the story and the order is evident. (4) is the clear concluding
1. The key points in the passage are: sentence. Ans: (21354)
(A) Banks are better off now, financially.
2. It is not difficult to identify the central idea of the passage: the
(B) This is because of low interest rates which has
passage describes the consequences of the lack of public
increased their income.
awareness about brain death. So sentence (5) can be taken
(C) With increased income banks have been able to wipe
as the opening sentence. Sentence (1) describes brain death,
off the losses.
so it should follow (5). Sentence (3) tells the reason for what is
(D) Increase in interest rates has again led them to their
stated in (1), so (3) is the next one. Sentence (2) tells the result
traditional business, namely lending, for earning
of the belief of most people mentioned in (3), so (2) follows (3).
income.
Sentence (4) tells the final result: the organs are not in a
Choice (A) is right. Choice (B) is wrong because it leaves out
condition to be transplanted. Ans: (51324)
important information. Choice (C) is wrong because it says
'......... enter into lending ..........' whereas banks have always 3. The paragraph talks about a particular university offering
been doing it. Choice (D) is wrong because it says 'banks degrees which are highly sought after all over the world. The
are better' – banks are not 'better' but 'better off' (better off ideal way to begin the paragraph is with sentence (5), which
means 'have more money'). Further they don't 'resort to' says they export education to customers. The next sentence
lending. Choice (A) could be the one that describes the customers, so sentence (2)
can be considered. The degree that is mentioned in (2) is
2. The main points in the paragraph are - further described in (3), so (3) can be the next sentence.
1. When we use the term 'animated films' we assume it is Sentence (1) gives more information about professors
for children. mentioned in (3), so (3) is the next sentence. ‘These
2. Many animated films have adult appeal, as well as commodities’ in sentence (4) relate to ‘intellect and information’
appeal to the child in adults. in (1), and so (4) can be the next sentence. Ans: (52314)
3. Many animated films have themes that appeal to both 4. ‘World Conference on International Telecommunication’ and
children and adults. ‘heated debate’ are key words to pick b as the opening
4. Animated films may however, not appeal to all adults sentence. The phrase ‘some countries’ is a clue to pick a as
either-It is a question of choice. the second sentence as the sentence mentions a point of
Option A is a distortion because it says the varied themes debate. (4) begins with ‘some’ which can easily be
draw children whereas the original paragraph says that the considered as a continuation of ‘some countries’ in the
varied themes draw adults. previous sentence. The clue to select (3) as the next
Option B omits points 2 and 4. sentence comes from the word ‘others.’ Sentences (1),
Option C is the best summary as it includes all the main ideas. (4) and (3) mention the stand taken by various countries. The
Option D too is a distortion because of 'many adults' are still concluding sentence can only be (5) as it states the result.
children. Choice (C) Ans: (21435)
Triumphant Institute of Management Education Pvt. Ltd. (T.I.M.E.) HO: 95B, 2nd Floor, Siddamsetty Complex, Secunderabad – 500 003.
Tel : 040–27898195 Fax : 040–27847334 email : info@time4education.com website : www.time4education.com SM1001941/156
5. The choice of the opening sentence is limited to (2) since all Explanatory notes for questions 11 to 20:
other sentences begin with the pronoun ‘he’ which obviously
refers to the driver in (2). Now the task is to order sentences 11. The arguments put forward in the paragraph suggest that
(1), (3), (4) and (5). ‘General’ to ‘specific’ is a commonly Vivek is comfortable with the BPO with regard to the current
accepted principle followed in description of a person or salary and life-style, but he doesn’t appear confident of the
explanation of a concept. This makes us select a as the next scope of maths in future. With regard to option B, there is no
sentence since it gives a general description of the driver. suggestion in the paragraph that he doesn’t see any future
Going by the same principle, we can choose (4) as the next for mathematics. May be, one can conclude that he is not
sentence as it talks about the ‘global crisis’ and ‘Mexico.’ sure. But such an option is not available. Option C is not
‘Unemployment’ and ‘inability to get the right job’ in e can based on facts presented in the passage. Option D is also
easily be related to ‘things were bad in Mexico for people like incorrect since mathematics is a subject that he still loves.
him’ in (4), so (5) can be chosen as the next sentence. ‘Wasting a few more years’ in the last sentence may be
Sentence (3) is the logical conclusion as the driver blames the related to financial aspects alone. Choice (A)
U.S for the present situation. Ans: (21453)
12. The expression ‘putting a spotlight on the importance of
6. A reading of the sentences will make it clear that the topic mould remediation for public health’ in the last sentence
being discussed is ‘boundaries in relationships.’ So the leads us to option C. Other options are contrary to the facts
opening sentence is the one that talks about the topic in in the paragraph. Choice (C)
general terms. This makes us pick sentence (3), which tells
us that boundaries in relationships exist all over the world. 13. It is clear from the paragraph (especially the words “...unlike
As the next one, sentence (4) can be chosen as it further traditional market researchers...”) that the anthropological
explains where these boundaries exist. ‘They’ in (2) can be researcher is also engaged in market research. The words
related to ‘boundaries’ in (4), so (2) is the next sentence. “may seem inefficient” indicate that the contrary would
Sentence (1) explains how these ‘boundaries’ affect our follow. An indication of the efficiency of the method is
transactions with others, and so a can be the next sentence. presented only in B. Choice (B)
What is stated in (1) is further illustrated in 14. The very fact that the shortcomings are referred to as
(5) with an example. Hence (5) is the next sentence. “struggles with challenges” implies that we do try to overcome
Ans: (34215) them, and that their outcome is of significance. We wouldn't be
7. A quick reading of the sentences will suggest that the trying if it wasn't necessary. Therefore, statements A and B are
paragraph is about a recent report in The Daily Mail. So we inappropriate. The paragraph is not about competition or
can choose (3) as the opening sentence. The response of competitive environments, so statement D is inappropriate.
‘most people’ suggested in (3) is explained in a and ‘their Statement C is appropriate– indicating that there are things we
brain’ in (1) is the clue for choosing a. Logically, (5) is the can look to for inspiration, in coping with these challenges.
next sentence as it gives more information about the study Choice (C)
referred to in the previous sentences. The ironical thing
15. While the para focuses, specifically, on how to get the best
explained in (2) is well linked to what is stated in (5) that the
out of an individual, statements C and D are general
story has been echoed by a large number of news sources.
statements presenting beliefs or perceptions about
So (5) should precede (2). Sentence d further explains the
management. They would be appropriate to open the
point mentioned in (2). Ans: (31524)
paragraph, but not to close it. While statement A presents
8. It is not difficult to identify (5) as the opening sentence. the kind of thought that would close this paragraph – it
The beginning of (2), (3) and (4) (They, However and As a presents an assertion of what happens, whereas it should be
result) gives us the clue that they cannot be opening in the nature of what could or would happen, since all the
sentences of a paragraph. The pronoun ‘these’ in (1) makes prior statements speak of what would be expected of
it unsuitable for the opening sentence of a paragraph. ‘They’ managers (commencing with 'should' in the first sentence).
in (2) can refer to ‘most people’ in (5), so (2) can be the next Therefore, B is the appropriate choice, indicating surprise
sentence. ‘However’ in (3) gives the indication that that an uncomplicated arrangement such as that discussed
something contrary to what has been said is going to follow is hardly followed. Choice (B)
- ‘once a follower suddenly becomes a leader’ is what
follows. Sentence (4) that begins with ‘as a result’ is a 16. The two possible answers are choice (A) and (C). Choice (A)
continuation of (3). Sentence (1) concludes the argument may sound reasonable, but the passage talks about polio
saying we have seen instances of these qualities with our eradication. There is no reference to eradication in choice
managers. Ans: (52341) (A). The paragraph says polio virus can spread rapidly in
places where hygiene and sanitation are poor.
9. Statement '4’ is the opening sentence as it mentions the So vaccination is important. The logical conclusion can be
property originally owned by four generations of the Baker choice (C) which says if enough children are immunized, the
family in Kumarakom. Next is statement '2’ as it lets the virus can’t find susceptible children to infect and dies out. So
reader know that the Kerala tourism department first choice (C) suits the context. Choices (B) and (D) are
acquired property and the Baker Bungalow from the Baker irrelevant. Choice (C)
family. Sentence ‘1’ maintains the chronological order and
tells us how the mansion was 'later' leased to the Taj Group 17. The paragraph talks about India’s public health policy pitted
which runs it as a resort 'now' Sentence '3' mentions the against some powerful sectors and the government’s
features of the resort which are reminiscent of Baker's inability to tighten control on them. The ideal conclusion can,
handiwork. Finally sentence '5' mentions the layout of therefore, be the one that talks about the policy, and not the
Zuri Kumarakom that opened in 2006. Therefore 42135 is ones that talk about just one issue - tobacco. So choices (A),
the correct sequence. Ans: (42135) (B) and (C) are ruled out and choice (D) can be chosen as
the correct conclusion. Choice (D)
10. Sentence ‘5’ is the ideal opening line as it introduces the
reader to the 'robotic arm which is one of the most important 18. ‘A new era of Asian-style succession politics’ in the
parts of a robot's anatomy. Sentence ‘2’ comes next because paragraph gives the clue to select choice (C), which
it tells us that a robot's capabilities are defined by its compares the House of Commons to the Lok Sabha, in the
hand/hands. Next comes statement '4' which mentions the event the offspring of the senior leaders win the election.
'tasks' done by a robotic arm/hand. Sentence '1' mentions This is the logical way the paragraph can be concluded.
the word 'moreover' strengthening sentence '4' and adding Choice (A) is not suitable as a concluding sentence; it might
more information to it. Finally sentence '3' suggests fit in as an introduction. Choices (B) and (D) talk about
'improvement' for greater 'dexterity' for complex tasks like creating a brand name, which can happen only if they win
working with and caring for people. Hence 52413 is the the election. So they can be ruled out as the concluding
correct sequences of sentences. Ans: (52413) sentence in this context. Choice (C)
Triumphant Institute of Management Education Pvt. Ltd. (T.I.M.E.) HO: 95B, 2nd Floor, Siddamsetty Complex, Secunderabad – 500 003.
Tel : 040–27898195 Fax : 040–27847334 email : info@time4education.com website : www.time4education.com SM1001941/157
19. The paragraph discusses a new method of pricing being what it was earlier. Hence 4532 form a sequence. Statement
adopted by a new airline. The penultimate sentence (1) talks about universal basic income – an idea which is
mentions the feelings of some passengers, particularly the totally different from what is stated in 4532. Hence (1) is the
slim ones, about this method of pricing. Option B concludes odd man out. Ans: (1)
the paragraph, endorsing the rationale behind this method of
pricing. All other options are about future consequences of Explanatory notes for questions 26 to 30:
this system of pricing. Hence they don’t conclude the
paragraph. Choice (B) 26. The focus of the paragraph is on the harmful effects of sleep
deprivation. It can affect metabolism, heart function,
20. The paragraph focuses on the Indian perception of duty. cognition and neurological activity. Option (2) summarises
Sanatana Dharma is not practised today, and we have no clear the paragraph. Option (1), though a close option, uses a very
perception of what is good and what is bad; what is good for strong expression ‘disastrous consequences’, which makes
some is bad for some others. Option D concludes the it inapt. Options (3) is only a supporting point. No attempt is
paragraph by completing the argument -what some see as made in the paragraph to compare psychological
duty is seen by others as abuse of duty. Option A gives a new consequences with bodily discomforts, so option (4) is not
idea, and it doesn’t conclude the paragraph. B cannot be a summary of the paragraph. Ans: (2)
related to the argument; it is quite remote. Option C is totally 27. The crux of the paragraph is that in order to evolve into a
irrelevant to the paragraph. Choice (D) leader, one has to reflect on the real motive of becoming a
leader. This is best reflected in option (4). Option (1) is
Explanatory notes for questions 21 to 23: a sweeping statement which distorts the meaning of the
paragraph. Option (2) goes against the paragraph.
21. Statements B, C and D (in the order CDB) discuss the Option (3) is only a sub point to the main one. Ans: (4)
potential of mutual funds in making money. Statement A,
however, says that mutual funds in India have gone after 28. The paragraph says that the economy today is controlled by
corporate and institutional investors. This has no relevance finance, and this has nothing to do with the real economy of
to the context. Choice (A) output and jobs. So the offering of schemes in the budget for
new derivatives etc. will not make any changes in the real
22. Statements A, C and D (in the order ADC) talk about how economy. These points are summarised in option (1).
Linfen in China improved living conditions by constructing Finance and economy being independent of each other is
vital things like public toilets. But statement B talks about the not suggested in the paragraph, so option (2) is incorrect.
design of toilets that has changed people’s perceptions. This Gains from derivatives not being consistent in option (3) is
statement does not match the idea presented in this not the focus of the paragraph. Option (4) is a sweeping
paragraph. Choice (B) comment not supported by the paragraph. Ans: (1)

23. Statements A, C and D (in the same order) describe 29. According to the passage, a few leaders seek money, power
a tendency among workers not to take their entitled vacation. and prestige, rather than valuing fair compensation for their
Statement B talks about the worry of workers. This is out of achievements. When they get these they develop a deep
context. Choice (B) desire to get more, and in the process of acquiring these,
they break their ethical standards and fair conduct. Option
Explanatory notes for questions 24 and 25: (3) succinctly summarises these points. Option (1) is too
general and sweeping, and distorts the import of the
24. Statement (4) begins the para by giving an introduction to paragraph. Option (2) is not supported by the paragraph;
what follows in the subsequent part of the passage. rather it goes against the argument. The focus of the
Statement (5) follows (4) by elaborating on one of the factors passage is not the types of leaders, so option (4) is incorrect.
which enable free speech. (1) follows (5) by citing some more Ans: (3)
instances. Statement (3) concludes the paragraph by summing
30. The topic of the paragraph is the consequence of the recent
up what is stated in the preceding statements. Option (2) brings
WTO ruling. At present it may not appear to be
in another idea which is not related to what is stated in 4513
disadvantageous, but in the long run this will crush our solar
and is therefore the odd man out. Ans: (2)
equipment industry. The best summary of the paragraph is
option (3), which highlights the detrimental effect of the ruling
25. Statement (4) begins the paragraph by defining work as the
on our fledgling equipment industry. Option (1) covers only
most important institutions of society. (5) and (3) further
part of the paragraph. Option (2) is distorted. Option (4) is
elaborate on what is stated in (4). (2) presents a contrast by
totally unrelated to the paragraph. Ans: (3)
stating how work is now less generous and less certain than

EXPLANATORY NOTES FOR VOCAB EXERCISES


PRACTICE EXERCISE – 1 4. The words bellicose (derived from the root belli = war, fight)
and pugnacious convey the same meaning. The words
Explanatory Notes for questions 1 to 5: astute and canny mean shrewd and crafty. Oppressive
means brutal or tyrannical. Choice (C)
1. Integrate is a synonym of the word amalgamate (derived)
from the root am = love). The words imbue (instill, permeate), 5. The words indolence (derived from the root dol = pain,
buttress (support) and ostracize (expel) are not synonyms of sorrow) and slothful are synonymous. Docile (derived from
amalgamate. Choice (A) the root doc = teach) means passive or compliant.
Lugubrious means sad or gloomy, intransigent means
2. Convergence is synonymous with the word confluence stubborn or obstinate. Choice (D)
which is derived from the root flu = flow. Conflagration
means a blaze or inferno. Choice (B) Explanatory Notes for questions 6 to 10:

3. The words exculpate (derived from the root culp = fault, 6. Amiability (derived from the root ami = love) is an antonym
blame) is synonymous with exonerate meaning free from of acrimony (acr = sharp, sour) meaning bitterness.
blame. Extirpate is to eliminate or abolish, excoriate is to flay Raucousness means discordance or harshness, tyranny
or critize sharply, extradite is to bring to trial. means severity or absolution and irascibility means anger or
Choice (C) acerbity. Choice (A)

Triumphant Institute of Management Education Pvt. Ltd. (T.I.M.E.) HO: 95B, 2nd Floor, Siddamsetty Complex, Secunderabad – 500 003.
Tel : 040–27898195 Fax : 040–27847334 email : info@time4education.com website : www.time4education.com SM1001941/158
7. Applaud is an antonym of bemoan meaning lament. The Explanatory Notes for questions 16 to 20:
word bemoan is derived from the root be = to make, to have
a certain quality. Berate is to castigate or criticize, deplore 16. Option A is absurd because if issues of environmental
and bewail (derived from the root be = make) both mean to impacts are incontrovertible (derived from the root contra =
disapprove of. Choice (C) against) or indisputable they will not fall on a spectrum
between two antagonistic camps. The words incessant (cess
8. Resist is an antonym of the word capitulate, meaning = yield, to go) meaning relentless or continuous and
surrender, which is derived from the root capt = to take, hold, polemical (hostile, bitter) do not make sense in this context,
seize. Elevate is to raise, enumerate (numer = number) is to hence option D can be eliminated. Between options B and C
itemize or count. Choice (D) the former is more logical because the word inimical is too
strong and is therefore unwarranted in this context.
9. The words intermittent (sporadic) and incessant (cess = Choice (B)
yield, to go) (constant) are antonyms. Steadfast means
abiding, fitful means periodic and incessant (cess = yield, to 17. The words execrable meaning appalling, dreadful and artful
go) means never-ending. Choice (D) (derived form the root art = skill, craft) do not make sense in
the second blank hence options B and C can be ruled out.
10. The word doctrinaire (derived from the root doc = teach) Option A is incorrect because the word ambiguous (derived
means rigid or inflexible. Liberal is its antonym. Rebellious from the root ambi = both) meaning vague is a misfit in this
(belli = war, flight) means disobedient, capricious means context. Further, the word ambiguous does not collocate with
whimsical or arbitrary. Choice (D) dispute. The words acrimonious (derived from the root acr =
sharp, sour) and amicable (derived from the root ami = love)
Explanatory Notes for questions 11 to 15: are most appropriate in the given blank. Choice (D)
11. An autocrat (derived from the root crac/crat = rule, power) 18. Option A is illogical because hamfisted (clumsy) handling of
lacks benevolence (derived from the root ben = good) or a crisis does not receive adulation or applause. Proficient
compassion. A recidivist (one who repeatedly relapses into handling of something does not receive flak (criticism).
crime) lacks compunction (regret). The word compunction is Slipshod (careless) handling does not receive
derived from the root (com = with, together). An ascetic commendation (praise). Hence both options B and D are
(abstainer, hermit) does not lack abstemiousness absurd. The words applause and adept (derived from the raft
(abstinence, moderation). The word abstemiousness is apt = correct, fit, skill, ability) fits perfectly in the given blank.
derived from the root ab = off, away, apart. Choice (C)
A criminal (derived from the root crim = judge, accuse) is not 19. The word resurrection meaning revival is a misfit in the first
lacking in acrimony (derived from the root ac = sharp, sour). blank because it runs contrary to what is intended in the
A connoisseur (arbiter of taste) is not lacking in taste. Only sentence. Hence, choice C is inapt. Choice B is absurd
option A conveys a relationship which is similar to that because an inflammatory (provocative) speech does not
expressed in the question pair. Choice (A) result in the termination of hostilities. The word interruption
does not make sense in the first blank because interruption
12. The words commodious, (derived from the root com = with, of hostilities is not a correct collocation. Hence choice D can
together) and expansive are synonyms, among the given be eliminated. The words cessation meaning to stop (derived
options the words cloistered (derived from the root clois = from the root cess = yield, to go) and inciting (derived from
cloister, reclusive) and confined (cramped) share a the root cite = to call, to start) is apt in this context.
synonymous relationship. The words decadent (cad = fall, Choice (A)
happen) and repulsive (hateful, abhorrent) squalid (filthy)
and alluring (charming), defiled (polluted) and pristine 20. Although the word acerbic (derived from the root acr =sharp)
(undefiled) are not synonymous. Hence only choice C is apt may fit into the second blank the word amplify is a misfit in
Choice (C) the first blank because it cannot be said that differences are
amplified. The word incite (provoke) does not fit into the first
13. The words in the question pair share an antonymous blank because it cannot be said that differences are incited.
relationship. Compliant, meaning acquiescent or Hence option B can be ruled out. The word ‘differences’
conforming, and are derived from the root com = with, implies that the election campaign has been rancorous
together, is the antonym of resistant (opposed to). Among (bitter; malignant). Options A and C can be eliminated
the given options defamatory (slanderous) and because the words zealous (enthusiastic) and fiery
complimentary (flattering) share an antonymous (passionate) are too mild in this context. Choice (D)
relationship. The words debilitating (weakening) and
Explanatory Notes for questions 21 to 25:
enervating (enfeebling; exhausting), clamorous (noisy; loud)
and boisterous; incessant (continuous) and relentless are all 21. Yes, Ambivalent (derived from the root ambi = both) means
synonyms. Choice (C) hesitant or in two minds. Hence a person who is ambivalent
is indecisive.
14. Euphemistic means extenuative or to make something less
serious. Malapropos refers to something which is untimely or 22. Yes. The word avionics, derived from the root avi = bird, fly,
infelicitous. Only option A shares a relationship which is is the science of, electronics when used in designing and
analogous to that expressed by the question pair. The words making an aircraft.
lucid (clear) and ambiguous (doubtful, unclear), gratuitous
23. Yes. An abbot is a monk; the superior of an abbey of monks
(done without purpose or reason) and prudent (wise;
who leads a cloistered existence.
rational), disingenuous (tricky, cunning) and veracious
(truthful) do not share a synonymous relationship and are 24. No, an agnostic is a person who believes that it is not
hence inappropriate. Choice (A) possible to know whether god exists or not. The word
agnostic is derived from the root cogn/agnos = to know.
15. Something which is illusory (derived from the root lus
= play) is characterized by deception; something which is 25. An ungrateful person is called an ingrate (derived form the
judicious is characterized by prudence. Only choice B root grat = please). Choice (D)
expresses a relationship which is analogous to that exhibited
by the question pair. The word judicious is derived form the Explanatory Notes for questions 26 to 30:
root jud = judge. The words transitory and impermanence,
26. The word absolve (derived from the root ab/abs = off, away,
grandiloquent (derived form the root loqu = speak) meaning
apart, down) means to forgive. Pardon and reprieve are its
pompous, and deceit, profound and flippancy are not
synonyms. Abjure meaning renounce or reject is the odd one
appropriate. Choice (B)
out. Choice (C)
Triumphant Institute of Management Education Pvt. Ltd. (T.I.M.E.) HO: 95B, 2nd Floor, Siddamsetty Complex, Secunderabad – 500 003.
Tel : 040–27898195 Fax : 040–27847334 email : info@time4education.com website : www.time4education.com SM1001941/159
27. The word armistice (derived from the root arm = weapon) is 42. It is too far-fetched that insatiable consumption of resources
synonymous with truce and ceasefire, Armada (also derived would lead to cataclysms (a sudden disaster like a flood or a
from the same root) refers to a large group of army ships war). Upheavals (bedlam, chaos) or incursions (raid,
sailing together and is hence the odd man out invasion). (The word incursion is derived from the root cur =
Choice (B) run.) But it is quite likely that it would lead to crises (times of
great danger or difficulty). Choice (A)
28. The word anarchy (derived from the root arch = rule) is
synonymous with mayhem and lawlessness. Cataclysm 43. The expression ‘run amok’ is most apt in this context. To run
meaning disaster is the odd word in this context. amok is to get out of control. The words viciously (brutally),
Choice(C) tempestuously (turbulently) and menacingly (threateningly)
are overstated and are therefore inappropriate in this
29. The word concord (derived from the root con = with,
context. Choice (B)
together) is synonymous with, accord and harmony. Combat
meaning fight is the odd word here. Choice (D)
44. An assumption is not championed, cultivated or endorsed.
30. The words approbation (derived from the root appro = But it is appropriate to say that an assumption is fostered.
correct, fit), approval and applause are all synonyms. Choice (D)
Appropriation, meaning allotment or grant, is the odd man
out. Choice (D) 45. The word noxious (poisonous) cannot be used with way of
thinking. Hence choice D is inappropriate. The word quirky
Explanatory Notes for questions 31 to 38: is redundant because it conveys the same meaning as the
word strange. The word calamitous (catastrophic) is also too
31. Statement D is incorrect because the expression ‘bound in’ strong in this context when compared to the word
is incorrect. The correct expression is ‘bound up in’. To be destructive. Choice (C)
bound up in something is to be very busy with something or
very interested or involved in something. Choice (D) PRACTICE EXERCISE – 2

32. The expression ‘on all accounts’ is incorrect in choice D. The Explanatory Notes for questions 1 to 5:
correction is ‘by all accounts’ or ‘from all accounts’ which
means according to what other people say. Choice (D) 1. The word approbation (derived from the root appro
= correct, fit, skill, ability) means approval or consent. The
33. The usage of screen is incorrect in choice B. The correct word endorsement is synonymous with it. The words
phrasal verb to be used here is screen off meaning to denunciation meaning censure or blame, obsequiousness
separate part of a room etc. from the rest of it by putting a (ob = against) meaning flattery, and condemnation which
screen around it. Choice (B) means disapproval are inappropriate in the given context.
Choice (B)
34. The use of the phrasal verb cut up is incorrect in choice B.
The correct phrasal verb is cut off meaning to cause a person 2. The words contravene (derived from the root contra
to be or feel alone. Choice (B) = against) and flout are synonymous. Debunk (de = down,
out, away, apart) is to demystify, refute is to contradict and
35. Choice C is incorrect. Blame is pinned on someone and not acquiesce is to agree or comply. Choice (D)
at someone. Choice (C)
3. The word itinerant, derived form the root iter = between;
36. The use of ‘off’ after face is incorrect in choice C. The correct among, means travelling or nomadic. The word peripatetic is
expression is vanish or disappear off the face of the earth. synonymous with it. The word languid means lazy.
Choice (C) Capricious means whimsical. Erratic means unstable or
unreliable. Choice (C)
37. The word rank should be used in the plural in choice A. To
rise through the ranks is to reach a high position in an 4. The word malodorous, derived form the root mal = bad,
organization, society etc. Choice (A) means rotten or foul-smelling. The word putrid is
synonymous with it. Alluring means fascinating or
38. The expression act out is incorrect in choice D. The correct enchanting. Redolent means fragrant or aromatic.
phrasal verb is act up which means to behave badly. Choice (C)
Choice (D)
5. Ludicrous, derived form the root lud = play, means absurd or
Explanatory Notes for questions 39 to 45: ridiculous. The word risible is synonymous with it.
Incredulous (cred = belief) means doubtful. Disjointed (dis =
39. The word revered (worshipped) is too strong and not, apart, away) means displaced and conceited means
unwarranted in this context. The words denounced and arrogant or haughty. Choice (A)
condemned run contrary to the idea implied in the paragraph
Explanatory Notes for questions 6 to 10:
where a parallel is drawn between the dysfunctional thinking
which prevailed in the erstwhile Nazi Germany and the 6. The words aggrieve (derived from the root griev = heavy)
present situation where dysfunctional thinking has resulted and please are antonyms. Mollycoddle is to overprotect.
in the abuse of natural resources. This is best brought out by Beguile is to entice or mislead. Choice (B)
the word institutionalized. Dysfunctional thinking was
institutionalized (considered normal) in Nazi Germany. 7. The words gregarious (greg = herd, crowd) and taciturn
Choice (B) (reserved) are antonyms. Odious means obnoxious or
repulsive. The words imperious and hubristic both mean
40. Only the word ravenous brings out the voracious arrogant. Choice (B)
consumption of resources, which the paragraph intends to
convey. The word restrained (restricted) and unwarranted 8. Coldness is the antonym of bonhomie, meaning geniality.
(unjustified) are absurd in this context. The word exorbitant The word bonhomie is derived from the root - bon = good.
is not used while taking about the consumption of resources. Bondage means servitude. Malice (mal = bad) is rancour or
Choice (C) animosity. Choice (B)
41. ‘Inordinate consumption of resources’ is an incorrect 9. Outstanding (exceptional) is the antonym of mediocre
expression. Hence option B can be ruled out. Among options (of medium quality), the word mediocre is derived form the
A, C and D, D is most appropriate because it brings out the root med = middle). The words prosaic and pedestrian mean
extent of greed which is seen in the consumption of dull and monotonous. Exceptionable means offensive or
resources. Choice (D) disagreeable. Choice (B)
Triumphant Institute of Management Education Pvt. Ltd. (T.I.M.E.) HO: 95B, 2nd Floor, Siddamsetty Complex, Secunderabad – 500 003.
Tel : 040–27898195 Fax : 040–27847334 email : info@time4education.com website : www.time4education.com SM1001941/160
10. The word prolix (derived form the root lix = speak, speech) Polemics means – controversial argument. Except (A) the
means wordy. Concise is its antonym. Turgid means bloated other choices do not have any relevance to the context.
or inflated. Periphrastic and discursive mean bombastic or Choice (A)
long-winded. Choice (C)
22. All choices except B are irrelevant. ‘Lucubrate’ means to study
Explanatory Notes for questions 11 to 15: at night and ‘luminary’ means a ‘celebrity’. ‘Amnesia’ (mnem =
remember) means loss of memory, usually, due to brain injury,
11. Catholic (derived from the root hol = whole). shock, fatigue, repression or illness. ‘Alumnus is a past student
of a school, college or university. Perspicuity might fit into the
12. Abdicate (derived from the root dic = to say)
first blank but the word protagonist (central character in a play
13. Fidelity (fid = faith, trust). etc.) is a misfit in the second blank. The word ergophobia
(phob = fear) (fear of work) and democrat (one who practices
14. Depravity (de = down, out, away, apart) social equality) are illogical. Choice (B)
15. Gratuity (from the root grat = pleasing). 23. ‘Lector’ refers to a reader in a church service or a lecturer in a
foreign university who gives instruction in his native language.
Explanatory Notes for questions 16 to 20: ’Jetsam’ means discarded material, in other words, ‘worthless
things’. ‘Nestor’ refers to a wise old man, but ‘stygian’ refers to
16. Tortuous and sinuous (sin = curve) both mean twisted or
‘darkness’. So (B) is not a meaningful choice. ‘Progenitor’
circuitous and are synonyms. Circumspect (circum = around)
means ancestor but ‘congenial’ means ‘pleasant’ or
and cautious are synonyms. obsolete means antiquated and
‘compatible’, which makes it a wrong choice. Choice (D)
caustic and scathing both mean critical in a bitter or sarcastic
way. Pugnacious in (B) means war-like, belligerent, whereas 24. The word ‘but’ clearly indicates that the blanks are to be filled
amatory (am = love) means related to sexual desire. They with two contrasting words. She is docile or tractable, but her
have no connection. Choice (B) job requires her to be rigid. 'Doctrinaire' (doct = teach) means
to be 'dogmatic'. ’Naïve’ means marked by unaffected
17. A misanthropic (mis = bad, hate) person (one who hates
simplicity. Insouciant means light-hearted unconcern or
human beings) has rancour, ‘a hatred and desire to hurt
nonchalance. ‘Caddish’ means discourteous or ungentle
others’. Similarly one who is prudent (wise) has
manly in behaviour. ‘Judicious’ means having, exercising, or
circumspection (wisdom); an embodied person can be called
characterized by, sound judgment.’ Gregarious’ (greg =
an incarnation and a cloistered person lives in seclusion.
crowd, flock) means of or relating to social groups.
Incredible means unbelievable and disbursement is the
’Prescient’ (scien = to know) means omniscience (scien = to
distribution of money. Hence, the answer is (D).
know) or foresight. Choice (C)
Choice (D)
25. The clue word 'although' suggests that the second blank
18. 'Regicide' (cide = to kill) refers to the act of killing a king,
should be filled with a word that means ‘similar’, because
'fratricide' refers to the act of killing one’s brother, 'parricide'
'although' counters the word ‘difference’. Similarly, the first
refers to the act of killing one's parents, and 'uxoricide' refers
blank needs a word that is synonymous with 'wisdom'. Amity
to the act of killing one’s wife. But 'filicide' refers to the act of
means friendly relations between nations. ‘Dour’ means
killing one’s children and not siblings. Choice (C)
stern, harsh. It can also mean obstinate, unyielding gloomy
19. ‘Trojan Horse’ is a symbol of perfidy. Except (D), all pairs and sullen. Sagacity or ‘sagacious’ means one who has a
have the same relationship. ‘Trojan horse’ in history refers to keen sense of perception and discerning, ‘cognate’ is of
a large hollow wooden horse filled with Greek soldiers and similar nature or generically alike, ‘prudence’ is the ability to
introduced within the walls of Troy by a stratagem. So, now govern and discipline oneself by the use of a reason.
Trojan Horse means someone or something intended to ‘Morose’ means having a sullen or gloomy disposition.
defeat or subvert from within usually by deceptive means. The words declivity (clin/cliv = slope, lean) meaning a
‘Perfidy (fid = faith, trust) is the quality or state of being downward slope and infallible (fal = deceive) are illogical in
faithless or disloyal. Cornucopia is a curved goat’s horn the given context. Choice (B)
overflowing with fruit and ears of grain that is used as a
Explanatory Notes for questions 26 to 30:
decorative motif emblematic of abundance. This is a
synonym of ‘bounty’ ‘Chimera’ is a fire-breathing she- 26. The words diffidence (derived from the root fid = faith, trust),
monster in Greek mythology having a lion’s head, a goat’s reticence and reserve convey the same meaning. Effusion
body and a serpent’s tail. It also means an ‘illusion’ ‘Phoenix’ (derived from the root fus = pour) is the odd one out.
is a legendary bird which according to one account lived 500 Choice (D)
years, burned itself to ashes on a pyre, and rose alive from
the ashes to live another period. ’Frankenstein is the 27. Foreboding (fore = before), premonition and presentiment
character in Mary W. Shelley’s novel Frankenstein who convey the same meaning. Caveat is the odd word.
creates a monster by which he is eventually killed Choice (B)
Choice (D) 28. Entreaty (en = in, into), appeal and imploration convey the
20. Jettison, derived form the root jet = to throw, means to same meaning. Insinuation (sin = curve) is the odd man out.
discard or throw. Bowlderize is to remove something which Choice (C)
is improper or offensive from a text. The word expurgate is 29. Diaphanous (derived from the root dia = across) is a
synonymous with bowlderize. Elucidate (explain) and synonym of gossamer and translucent. Turgid does not
explicate, disgorge (expel; pour out) and regurgitate also belong to this group. Choice (A)
convey the same meaning. The word inculpate (culp =
blame) means to accuse or blame. Absolve (solv = loosen, 30. Amorphous (morph = shape), nebulous and vague convey
free) meaning pardon or forgive is not synonymous with the same meaning. Nascent (is the act of being born) is the
inculpate. Hence only choice A does not exhibit the same odd one out. Choice (B)
relationship as the capitalized pair. Choice (A)
Explanatory Notes for questions 31 to 38:
Explanatory Notes for questions 21 to 25:
31. The expression ‘quick in the draw’ is incorrect in choice B.
21. ‘Lexis’ (lex = read, speak) means vocabulary and The correct expression is ‘quick on the draw’ meaning to be
‘logomachy’ refers to a dispute about words. Deportment quick to understand or react in a new situation.
(port = to carry) meaning demeanour might fit into the first Choice (B)
blank but the word ‘eugenics’ (eu = good) – the science of
improving mankind by controlled breeding – is a misfit in the 32. The use of the article before home makes choice D incorrect.
second blank 'Epistemology’, is the science of knowledge. The correct expression is ‘…..home to’. Choice (D)

Triumphant Institute of Management Education Pvt. Ltd. (T.I.M.E.) HO: 95B, 2nd Floor, Siddamsetty Complex, Secunderabad – 500 003.
Tel : 040–27898195 Fax : 040–27847334 email : info@time4education.com website : www.time4education.com SM1001941/161
33. The use of the word effect is incorrect in choice (A) context. 2. The word seminal (sem=seed) means highly original and
The correct word to be used here is affect, meaning to influencing the development of future events. The word
produce or change in somebody and something influential is clearly its synonym. Hypothetical, derived from
Choice (A) the root hypo = under, too little (assumptive, debatable),
pervasive (prevalent) and pertinent (relevant) are misfits in
34. Option D is erroneous. The correct expression is work the given context. Choice (A)
yourself into the ground meaning to make oneself tired or ill
by working hard. Choice (D) 3. The root in the word insurgent is surg = rise. The word rebel
is synonymous with it. Insurgent can be used both as a noun
35. The expression ‘------give the game off’ is incorrect in choice and as an adjective. If used as an adjective, the words
D. The correct expression is ‘----- give the game away’ which rebellious, insurrectionary and dissatisfied are synonymous
means to spoil a surprise or a joke by telling someone with it. An infidel is an agnostic or a non–believer. A
something that should have been kept secret. rationalist is a free-thinker. A pacifist is a peace-lover.
Choice (D) Choice (D)
36. The expression ‘form up’ does not make sense in choice (C). 4. The word taciturn (tacit = silent) means cold or reserved. The
The correct expression is ‘shape up’ meaning to develop in word reticent is synonymous with it. The words hostile,
a particular way, especially in a good way. Choice (C) hubristic (arrogant) and congenial (affable) are not relevant
37. The expression ‘drop off from’ is incorrect in choice(C). The in this context. Choice (A)
correction is ‘------drop out of’. Choice (C)
5. The word subvert, derived from the root vert = to turn, means
38. In choice A the word edge should be followed by to or in. To undermine or weaken. Intimidate (tim = fear) is to coerce or
have an edge to in one’s voice is to have a small but bully. Condescend is to demean oneself. Vilify means abuse
noticeable amount of annoyance in one’s voice. or curse. Choice (B)
Choice (A)
Explanatory notes for questions 6 to 10:
Explanatory Notes for questions 39 to 45:
6. The word desultory is derived from the root sult = jump, leap.
39. The passage states that evil is not self-sustaining and that it Desultory means aimless or haphazard. Methodical is its
cannot survive on its own. Hence it is obvious that its antonym. Superficial (super = above) means cursory,
enduring existence requires the acquiescence (acceptance) haphazard means aimless or uncoordinated. Choice (B)
of the good. Hence choice (B) is apt. The words ratification
7. The word tactile is derived from the root tact = touch. It
(approval), corroboration (confirmation) and patronage do
means perceptible or palpable. Intangible is the antonym of
not make sense in this context. Choice (B)
tactile. Robust (strong); enervating (debilitating) and
40. It is stated that the irrational survives by feeding off the insouciant (easy-going) are inappropriate in the given
rational hence it can be said that it survives parasitically. The context. Choice (C)
word triumphantly which is a positive word does not make
8. The root in the word tremulous is trem = shake, fear.
sense here. The words haplessly (derived from the root hap
Tremulous means quavering or shaky. The word steady is
= by chance) and dispassionately (derived from the root dis
its antonym. The words irresolute (undermined)
= apart) are inappropriate. Choice (D)
presumptuous (arrogant) and obdurate (adamant) are not
41. It is stated in the paragraph that an irrational idea gains pertinent in the given context. Choice (D)
influence by covering itself with rationality, therefore it is 9. Urbane is derived from the root urb = city. It means polished
obvious that it gains influence fraudulently (deceitfully). or sophisticated. Boorish meaning uncultured or unrefined is
Hence choice A fits the blank perfectly. The words opposite in meaning to the word urbane. The words suave
fallaciously (erroneously), forebodingly (menacingly) and (stylish, elegant), rational(pragmatic) and antiquated
errantly (guiltily) can be easily eliminated. Choice (A) (ancient) are not suitable in this context. Choice (C)
42. It is obvious form the passage that an irrational idea covers 10. The word convivial is derived from the root vita / viva = life,
itself with a veneer (façade) of rationality. The words edifice alive. The meaning of convivial is welcoming, friendly. The
(building, structure) modicum (small amount)and protection word unfriendly is its antonym. The words jovial, vivacious
do not make sense in the context. Choice (C) (lively) and flamboyant (flashy; dazzling) are not related.
43. Only the word defend (protect) fits in the blank most Choice (D)
appropriately. The words obscure (vague), strive and Explanatory notes for questions 11 to 15:
conceal (hide) are inappropriate in comparison.
Choice (B) 11. Quintessence (derived from the root quin = five), epitome
and embodiment are synonyms. Metaphor, which is a figure
44. The words blatant (flagrant) and covert (secret) which have of speech in which a term or phrase is applied to something
a negative connotation are misfits in the given blank. Only to which it is not literally applicable in order to suggest a
the word unequivocal, derived from the root meaning resemblance, is the odd one out. Choice (A)
unmistakable is most appropriate here. The word categorical
meaning emphatic is inapt in this context because here the 12. The word corroboration, derived from the root robor
emphasis is on identifying evil as evil without doubt or = strength, is synonymous with validation and confirmation.
ambiguity. Choice (D) Elucidation (clarification) is the odd one out.
Choice (B)
45. The word impotent meaning powerless or worthless is most 13. The words conjecture (derived form the root – ject =to throw)
appropriate in the given blank. The words indisposed, crippled surmise and speculation convey the same meaning.
(immobilized) and incapacitated (indisposed, immobilized) are Exposition meaning explanation is the odd one out.
inappropriate in comparison Choice (C) Choice (D)
PRACTICE EXERCISE – 3 14. The word commiserate derived from the root miser =
wetched, pity, means to empathize or console. Wheedle
Explanatory notes for questions 1 to 5: (wax; cajole) is the odd word in the category.
Choice (B)
1. The word sacrilege, derived from the root sacr = sacred, holy
means blasphemy or disrespect. The word profanity (pro = 15. The words premonition (derived from the root mon = warn)
ahead, forth) is synonymous with it. Castigation means forboding and portent are synonyms. Exhortation
severe reprimand or criticism. Choice (B) (persuasion) is the odd word. Choice (A)

Triumphant Institute of Management Education Pvt. Ltd. (T.I.M.E.) HO: 95B, 2nd Floor, Siddamsetty Complex, Secunderabad – 500 003.
Tel : 040–27898195 Fax : 040–27847334 email : info@time4education.com website : www.time4education.com SM1001941/162
Explanatory notes for questions 16 to 20: 25. A prodigal (a spendthrift) lacks thrift (economy), a reprobate
(an immoral person) lacks morality. The words desperado
16. The words undermining (subverting, weakening) and
(a person who does dangerous things without caring about
obliterating (effacing) cannot be used with fundamental
himself or other) and fear, maverick (a non–conformist) and
rights. It cannot be said that fundamental rights are
rebellion, ascetic (a person who renounces physical
undermined or obliterated. Hence options A and D can be
pleasure) and abstemiousness (renouncing things which
eliminated. The words suppressing and demolishing can fit
give pleasure) are not analogous to the original pair.
into the second blank but the word risible (rid/ris = laugh)
Choice (C)
(laughable) does not make sense in the first blank. The
words ludicrous (lud/lus = play) and demolishing are most Explanatory notes for questions 26 to 30:
appropriate. Choice (C)
26. Yes. The word sanctuary is derived from the root
17. The first words of options A, B and D are possible in the first sanct = sacred holy. It refers to a safe haven or an asylum.
blank but the word abating (subsiding) is a misfit here. Hence
option C is ruled out. Options B and D can be eliminated 27. Yes. Prescient means prophetic or psychic. It is derived from
because the words deterioration and erosion cannot be used the root scien = to know.
with habitats. Only option A is apt. Choice (A)
28. No. Proscribe is derived from the root scrib = write. It means
18. The structure of the sentence suggests that the words which to ban something or to make something illegal.
fit into the two blanks are contradictory. Among the given
options only the words suave (agreeable and courteous) and 29. Yes. A somnambulist is a sleep walker. The word
stern (harsh; severe) present a contrast which is intended in somnambulist is derived from the root somn = sleep.
the sentence. The words charismatic (charming) and grim,
urbane (dignified) and gaunt (haggard), debonair 30. Yes. The word contentious is derived from the root
(sophisticated) and deadpan (emotionless) are inappropriate tent = stretch. Contentious means arguable or debatable.
in comparison. Choice (A)
Explanatory notes for questions 31 to 38:
19. The words lectern, (derived form the root lect = read speak)
which refers to a stand for holding a book, notes etc when 31. The use of ‘out’ after keep is incorrect in choice D. The
you are reading in a church, giving a talk etc does not fit in correct expression is '……keep from' meaning to manage to
this context. The words rostrum, podium and platform are prevent yourself form doing something. Choice (D)
synonymous meaning ‘a raised horizontal surface’.
32. The error lies in option C. The correct expression is ‘match
However, the term platform has a sense of being a place or
up with something meaning to be the same or similar.
means/opportunity for public expression. Hence ‘C’ is most
Choice (C)
suitable. Choice (C)
20. Only option A is most logical. Mutually exclusive qualities are 33. The expression 'light up' does not make sense in choice A.
best described as dichotomies. But mutually analogous The correct expression is 'light upon' which means to find or
(related) qualities are not chasms (braches), mutually think of something unexpectedly. Choice (A)
complementary (harmonizing) qualities are not polarities 34. The phrasal verb 'jump at' is incorrect in option A. The correct
(conflicts) mutually reciprocal (correlative) qualities are not phrasal verb to be used here is 'jump on', meaning to criticize
disjunctions (polarities). Hence options B, C and D are someone as soon as they have done something wrong or
illogical. Choice (A) said something that you disagree with. Choice (A)
Explanatory notes for questions 21 to 25:
35. The expression 'look ahead' is incorrect in choice D. 'Look
21. A pedagogue (teacher; instructor) teaches, a pontiff ahead', meaning to think about what will happen in the future
sermonizes (preaches). An arbitrator negotiates but not an and plan for these events, does not make sense in this context.
arbiter. A lyricist (a person who composes songs) does not The correct phrasal verb is 'look forward'. The expression
sing. Clairvoyant (a person who is able to see future events) ('look forward + verb + ing) is used at the end of a formal letter
and apprise are not related. Choice (C) to say you hope to hear from or see someone soon, or that you
expect something from them. Choice (D)
22. Seismic means of or related to earthquakes. Fluvial means
pertaining to rivers. Option B is incorrect because the word 36. Choice A is incorrect. The use of the article is incorrect in the
pluvial is an adjective pertaining to rains. Pluvial clouds refer expression 'take issue with’ meaning to disagree strongly.
to rain clouds. Fluvial means connected with rivers. Insular Choice (A)
means of or related to islands. Funicular refers to a railway
on a steep slope, used to transport passengers up and down 37. '……..slow off the mark' is the correct expression in choice
in special cars by means of moving a cable. Choice (A) C. To be slow off the mark is to be slow to act or react to an
event or situation. Choice (C)
23. An iconoclast (a person who strongly opposes generally
accepted beliefs) is associated with scepticisim (disbelief), a 38. The phrasal verb lead up is incorrect in choice D. The correct
sophist (one who uses fallacious arguments) is associated expression is 'lead off' meaning to start something.
with casuistry (a way of solving moral or legal problems by Choice (D)
using clever arguments that may be false). A heretic (a
Explanatory notes for questions 39 to 45:
person holding a belief or opinion that is against the
principles of a particular religion) is not associated with 39. Totalitarianism can be described as a threat (a danger) to
conformism, a jingoist (one who believes that his country is mankind but it cannot be described as a plague (epidemic,
best) is not associated with idiosyncrasy (physical condition disease) torment (agony, anguish) or burden. Hence only
peculiar to an individual), an ideologue (a person whose option B is apt here. Choice (B)
actions are influenced by a belief in a set of principles) is not
associated with cynicism (the belief that people only do 40. The passage precisely speaks about totalitarianism (tyranny,
things to help themselves rather than for good or honest dictatorship) hence option A fits the blank perfectly. The
reasons). Choice (B) words sovereign (ruler), emperor and baron are
inappropriate in comparison. Choice (A)
24. An imbecile (a dullard, an ignorant person), lacks intelligence,
similarly a hedonist (a pleasure-seeker) lacks abstemiousness 41. Choice A is most appropriate in this context it is correct to
(self-denial; restraint). Tact is an essential quality of a diplomat. say that horrors are perpetrated. The words executed
The word virtuoso (a person who is extremely skilled at doing (performed; accomplished) or worked (brought about) do not
something) and ethics; felon (a criminal) and erudition, collocate with horrors. The word interpreted is clearly out of
(learning) are not related. Choice (D) context. Choice (A)
Triumphant Institute of Management Education Pvt. Ltd. (T.I.M.E.) HO: 95B, 2nd Floor, Siddamsetty Complex, Secunderabad – 500 003.
Tel : 040–27898195 Fax : 040–27847334 email : info@time4education.com website : www.time4education.com SM1001941/163
42. Only option D is most appropriate in this context. "Altruists' 45. The word which fits into the blank should contrast with the
justify the horrors perpetrated by them by saying that they word evildoer, hence benefactor is the appropriate word
are doing so for the common good. The word propitiate here. The words reprobate (outcast), puritan (prude) and
(appease), sustain (aid, approve) and establish don't make debauchee (libertine; sybarite) do not suit the context.
sense in this context. Choice (D) Choice (A)
PRACTICE EXERCISE – 4
43. Throughout the passage the author states that collectivism
is a threat to individual rights. Collectivism is one of the Explanatory notes for questions 1 to 20:
spiritual weapons which have enslaved (Coerced' innocuous – positive, weak – neutral, insipid – negative
compelled) men. The words enriched and empowered which innocent – positive, flat – neutral, banal – negative
are positive words are misfits in this context. The word painless – positive, unobjectionable – positive,
embittered (irked; irritated) does not make sense here. safe – positive, sapless – negative, jejune – negative,
Choice (D) inobnoxious – positive
1. Only the word insipid, which means bland or tasteless, fits
44. Those rights which men cannot be deprived of are inalienable
into the given context.
rights. Only option C suits the blank most appropriately. The
words ineluctable (inevitable ; inescapable) injudicious (jud 2. Innocuous is the appropriate word in the given context. A
= judge) and intangible (impabable) are inappropriate in remark which is innocuous is not intended to offend or upset
comparison. Choice (C) anyone.

Set –II

3. Only the word vicious (violent, cruel) collocates perfectly with the word criminal. All the other words are inappropriate in comparison.
4. It is stated that the person led a depraved (immoral) life earlier and he now intends to turn over a new leaf. This contrast intended in
the sentence is best brought out using the word virtuous (moral, principled). Opposite of a depraved life is a virtuous life

SIMILAR IN SENSE OPPOSITE IN SENSE

way in, access, door, exit, wayout, outlet,


entrance, opening, door
doorway SIMILAR IN SENSE

development,
advancement, growth,
SIMILAR IN SENSE improvement,
RETROGRESSIVE evolution, movement
wrongdoing, lapse,
misbehaviour,
disobedience,
indiscretion OPPOSITE IN SENSE

retreat, decline, failure,


OPPOSITE IN SENSE ingress stagnation, halt,
progress
perfection, goodness hindrance
kindness, obedience,
virtue
trangression SIMILAR IN SENSE

retreat, revert,
GRESS = to stop, relapse, degenerate,
OPPOSITE IN SENSE to go move back, go back

entrance, arrival regress


closure, stay OPPOSITE IN SENSE

egress digress develop, improve,


increase, forge,
forward, strengthen

SIMILAR IN SENSE SIMILAR IN SENSE


OPPOSITE IN SENSE
exit, door, outlet, deviate, wander,
wayout, retreat be direct, stay ramble, stray

5. The words punctilious (precise, proper) and meticulous (thorough, scrupulous) clearly indicate that perfection is the appropriate word
to fit into the blank
6. Lapse meaning a small mistake, especially one that is caused by forgetting something or by being careless, is the correct word which
brings out the meaning intended in the sentence.
Triumphant Institute of Management Education Pvt. Ltd. (T.I.M.E.) HO: 95B, 2nd Floor, Siddamsetty Complex, Secunderabad – 500 003.
Tel : 040–27898195 Fax : 040–27847334 email : info@time4education.com website : www.time4education.com SM1001941/164
I Clin/cliv = slope, lean II De= down, out away, apart
(a) inclination : leaning, slope (a)depart : go away, leave
(b) recline: lay down stretch out (b) deprecate : express disapproval, criticize
(c) incline : slant, gradient (c) deplete : reduce in quantity or numbers
(d) proclivity : penchant, predilection (d) defamatory : spoiling the good repudiation
(e) declivity : declination, descent (e) deface : spoil the appearance of
III Mis = wrong, bad, hate IV Dis = not, apart, away
(a) misanthrope : one who hates mankind (a) discordant: clashing, inharmonious
(b) mishap : an unlucky accident (b) disparage : underrate, vilify
(c) misconstrue: to intercept incorrectly (c) dissuade : prevent, hinder
(d) disparate : contrasting, diverse
(e) disseminate : disperse, spread
(f) distend: amplify, expand
(g) dissipate : squander, waste

7. It is quite obvious from the context that his income was so meagre that it was hardly enough for him to lead a decent life. This
meaning is brought out only by the word comfortable.

8. ‘Scanty’ is the only word, among all the given options, that collocates with the word rainfall.

Set – III

AFFLUENT

SIMILAR IN SENSE SIMILAR IN SENSE

extra, surplus, easy, articulate, flowing,


redundant, unessential, assured, confident,
excessive, unnecessary effortless

fluent OPPOSITE IN SENSE


OPPOSITE IN SENSE
halting, stuttering
superfluous
reasonable, important, faltering, slow, inept,
useful, necessary maladroit
needed

Flu/Flux = flow

SIMILAR IN SENSE

flowing together,
meeting, confluence, flux OPPOSITE IN SENSE
union
stability, constancy,
confluence steadiness. endurance
substantiality
SIMILAR IN SENSE

fluctuation, instability,
OPPOSITE IN SENSE change, unrest

divergence, aberration,
separation, detour,
departure

9. ‘Detour’, which means a longer route that you take in order to avoid a problem or to visit a place, is the appropriate word to be used
in the sentence.

10. Only the word ‘confident’ is the appropriate word to fill the blank

Triumphant Institute of Management Education Pvt. Ltd. (T.I.M.E.) HO: 95B, 2nd Floor, Siddamsetty Complex, Secunderabad – 500 003.
Tel : 040–27898195 Fax : 040–27847334 email : info@time4education.com website : www.time4education.com SM1001941/165
HOMEWORK EXERCISE (SET – III)
I art/ arti = skill, craft II fid = faith, trust
(a) art: painting, skill (a) confidence : sure, positive
(b) artificial: false, insincere (b) diffidence: shyness, reverse
(c) artifice : pretense, deception (c) prefidious : disloyal, treacherous
(d) artifact : relic, object (d) fidelity : loyalty, faithfulnes
(e) artful : crafty, devious
(f) artless : simple, natural
III co/com/ con = with, together IV super / sur = above
(a) compatriot : fellow citizen (a) supercilious : arrogant, haughty
(b) coalesce : unite, come together (b) supersede : succeed, displace
(c) compliant: acquiescent, conforming (c) superfluous : extra, redundant
(d) complicity : involvement (d) surfeit : excess, surplus
(e) concord : agreement, treaty (e) insurmountable :insuperable insolvable
(f) concur : agree, assent

11. Halcyon meaning peaceful and happy is the appropriate word to be used in the given context.
12. Only the word turbulent (agitated, disturbing) collocates with childhood and fits appropriately in the given context

SIMILAR IN SENSE SIMILAR IN SENSE

admiring, amorous, AMICABLE benevolence, comity,


ardent, devoted, concord, cordiality
passionate, goodwill, harmony
rapturous

amatory amity
OPPOSITE IN SENSE

aloofness, hostility,
disagreement, discord,
OPPOSITE IN SENSE illlwill, dislike
hateful,unfriendly AMI–love
hostility,

amorous amiable

SIMILAR IN SENSE

ardent, erotic,
romantic, enamour
SIMILAR IN SENSE
impassioned
passionate, lustful affable, amicable,
charming, cheerful,
OPPOSITE IN SENSE
OPPOSITE IN SENSE cordial, sociable,
lenient, mild
aloof, repulsive,
cold, frigid, hateful,
gloomy, irritable,
indifferent,
sad, rude, hasty
unfriendly

SIMILAR IN SENSE OPPOSITE IN SENSE

beguile, bewitch, annoy, bore, depress,


enthral, gratify, disappoint,disenchant,
hypnotize, spellbind disgust, dissuade,
repulse

Triumphant Institute of Management Education Pvt. Ltd. (T.I.M.E.) HO: 95B, 2nd Floor, Siddamsetty Complex, Secunderabad – 500 003.
Tel : 040–27898195 Fax : 040–27847334 email : info@time4education.com website : www.time4education.com SM1001941/166
13. ‘Spellbound’ is the appropriate word to be used in this sentence. To hold someone spellbond is to hold some one’s attention
completely.

14. ‘Harmony’ which means a state of peaceful existence and agreement is the appropriate word to be used in the given context.

HOMEWORK FOR STUDENTS (SET – IV)

I been = good II cord/card = heart

(a) benefit: advantage, gain (a) accord : agreement; treaty

(b) benevolence: kindness, charity (b) discord: disagreement dissonance

(c) benediction : blessing (c) cordial : pleasant; genid

(d) bonafide : genuine (d) cardinal : basic, fundamental

(e) benign : kind

III be = to make, to have a certain quality IV en = in, into

(a) belittle :disparage, decry (a) enter : go into; go through

(b) belated : late, delayed (b) entice : attract; beguile

(c) bemoan: lament, be wail (c) encumber: burden; hamper

(d) befriend : make friends with (d) endemic : widespread; prevalent

(e) bewilder: confuse, boffle (e) ensconce: entrench; establish

(f) bequeath : leave, donate (f) entreat: plead, beseech

PRACTICE EXERCISE – 5 4. An ingenious person is one who is very clever and skilful.
Only a makes sense here.
Explanatory notes for questions 1 to 20: Only the word flair which means the natural ability to do
something well makes sense here. Hence a.
1. Something comes into effect i.e., something begins to To compliment someone is to praise someone. Only a
happen. Hence a makes sense here.
Only the word 'currant' meaning a small black dried grape A prerequisite means something which must exist or happen
without seeds, makes sense here. Hence a before something else can exist or happen. Hence only b
The word venal meaning corrupt is the apt word here. makes sense here. Hence aaab. Choice (C)
Therefore b Eminent - meaning famous is the right word to
be used here. Hence aaba. Choice (B) 5. Turgid style of writing refers to that style of writing which is
pompous or bombastic. Hence only a makes sense here. The
2. Only the word allusion (something that is said or written that word 'tortuous' which means with many twists and turns is the
intentionally makes you think of a particular thing or person) more appropriate word in the given context. Therefore a.
makes sense here. Hence (b) The word proscribe (to ban; prohibit) is apt here. Hence a.
The word 'conscience' meaning the part of you that judges The word principle meaning a moral rule or standard of good
the morality of your own actions and makes you feel guilty behaviour is apt here. Hence aaaa. Choice (A)
about the bad things that have done, is the appropriate word
6. Portent is a noun meaning an omen or prophetic sign of the
to be used here. Therefore a.
future. Potent is an adjective meaning strong and powerful.
The word wreath (an arrangement of flowers and leaves in a
Only a makes sense here.
circular shape) is the apt word in this context. Therefore a.
The word equitable meaning fair and reasonable is the most
To cite is to speak or write words taken form a particular
appropriate word in the given context. Hence b.
writer or written work. Hence only a is apt here.
An inflammatory speech is something which is intended or
Hence baaa. Choice (A)
likely to cause anger or hate. Hence only b makes sense here.
Only the word exiguous in b(meagre; inadequate) suits the
3. Only the word elicit which means to obtain or produce
given context. Hence abbb. Choice (B)
something, especially information or reaction makes sense
here. Hence a 7. The word alter (change) makes sense in the given context – a.
The word 'conscious' is apt here. a conscious effort is a The word forbear which means to prevent yourself form
determined effort. Hence a saying or doing something is most appropriate in the given
The word counsel meaning advice is appropriate in this context – a.
context. Therefore b. The word officious meaning Only the word ascent (rising to a particular position or higher
overbearing fits the context appropriately. Hence b. rank) is apt here – a.
Hence aabb. Choice (D) The word stationary (static) makes sense here. Therefore
only a is apt. Hence aaaa. Choice (A)
Triumphant Institute of Management Education Pvt. Ltd. (T.I.M.E.) HO: 95B, 2nd Floor, Siddamsetty Complex, Secunderabad – 500 003.
Tel : 040–27898195 Fax : 040–27847334 email : info@time4education.com website : www.time4education.com SM1001941/167
8. If you wait for something with bated breath you wait for it in 15. The capitol refers to the legislative building where the U.S.
an anxious or excited way. Hence a congress meets. Hence b.
Only the word appraise which means to assess or estimate The word notable meaning eminent is apt in the given
suits the given context. Hence a context. Hence a.
Balmy weather refers to weather which is pleasantly warm. Raze means to completely destroy a city, building. Only
Therefore b a makes sense here.
A serial killer is one who repeatedly commits the same The word devise meaning design, frame etc. suits the
serious crime, using the same method. Only a makes sense context – b. The word device is a noun and devise which is
here. Hence aaba. Choice (A) the verb form make sense. Hence baab. Choice (B)

9. To defuse is to make a difficult or dangerous situation calmer 16. The word precedent which refers to an action, situation or
by reducing or removing its cause. Obviously b suits decision which has already happened and which can be
the context. used as a reason why a similar action or decision should be
The word oral which means of or relating to the mouth is apt taken, is appropriate in the given context. Hence b.
here – a. The word signatory, which refers to a person, organization
The word discreet meaning secret is apt in the given or country which has signed an agreement suits the given
context – a. context. Hence b
To pore over something is to look at and study something To declare something formally is to make something public
carefully – b. or official. Hence a
Hence baab. Choice (C) Plain in this context refers to pure, downright. Plain bad luck
means sheer bad luck. Hence a. Hence bbaa.
10. Censure is to criticize or reprimand someone for wrong Choice (D)
doing. Hence b.
The word palate which refers to the top part of the inside of 17. To avenge is to do harm to or punish the person responsible
the mouth also refers to a person's ability to taste and judge for something bad done to you or your family in order to
good food. Hence only the word palate suits the context. achieve a fair situation. Only avenge makes sense here.
Therefore a Hence a.
To broach a subject is to begin a discussion of something Shear means to cut off. The word sheer, which means ulter
difficult. It is obvious that only a suits the context. in the given context, is apt in the sentence. Hence a.
The course of a meal refers to a part of meal which is served The word refer is more apt in the given context. To refer
separately from the other parts. Only a suits the context. somebody to someone means you direct someone to
Hence baaa. Choice (B) a different person for information, help action, often to
a person with more knowledge. Only b is apt in this context.
11. The word amoral means not concerned with or affected by Braze is to solder or create with metals such as bronze.
morality, immoral means 'not conforming to accepted Braise meaning to cook(usually meat) slowly in the liquid is
standards of morality. Only the word immoral suits the given appropriate in the given sentence. Hence b.
context – a. Choice (B)
The word complacent meaning smug or self-satisfied is apt in
the given context. Hence a. 18. The word demure meaning shy and bashful is apt in the
Only the word aisle referring to a long narrow space between given context. Hence a. Aggravate meaning worsen is apt in
rows of seats in an aircraft cinema or church makes sense the given context. Hence a.
here – a. Only the word breach which refers to a split or rift To adopt is to take another person’s child into your own
makes sense in the given context. Hence b. family and legally raise him or her as your own child. Hence
Choice (B) it is appropriate to say an adopted son and not adoptive son.
Hence a. Restful refers to having a quiet and soothing
12. The word cue (hint) is most appropriate in the given context quality. Only a makes sense in this context. Hence aaaa.
– a. Only the word draft which means a piece of text, a formal Choice (B)
suggestion or a drawing in its original meaning, containing
the main ideas and intentions and not the developed form is 19. The word mettle meaning spirit and strength in the face of
apt – b. difficulty is apt in this context. Hence a. Auger is a tool for
Only the word curb meaning control is apt in the given making holes. Augur means to predict or forecast. Hence a.
context. Hence a Metier meaning profession suits the context perfectly.
To ensure is to make something certain to happen. Only a Hence a
makes sense here. Hence abaa. Choice (C) Adept (skilful) is apt in this context – a. Choice (D)
13. The word duel which refers to a deadly context between two 20. The word elusive (difficult to find, catch or achieve) is apt in
people is the most appropriate word in the given context. this context. Hence b.
Hence a Grisly (causing horror or revulsion) is apt in the given
To hail from is to come or have been born in a particular context. Hence a
place. Hale means sound or healthy. Only a is apt in this The word quiet meaning making little or no noise suits the
sentence. To hoard is to stack up on food money etc. hence context appropriately – a.
(a) hoarding is apt. A sceptic is a non-believer. A person who doubts the truth or
Foreword (a short introduction to a book) is the appropriate value of an idea or belief. Only a suits the context. Hence
word here. Hence a. baaa. Choice (D)
Hence aaaa. Choice (B)
Explanatory notes for questions 21 to 35:
14. The word mitigate (make less severe) is apt in the given
context. Hence b 21. Only choice C is correctly spelt. The correct spellings of the
Only the word career (an occupation undertaken for a remaining words are 'persuasive', 'unfaltering' and
substantial period of a person's life) makes sense here. 'prestigious'. Choice (C)
Hence a
If someone is loath to do something they are unwilling to do 22. Except the word 'resilience' all the remaining words are
something. Further, only ‘loath’ is followed by the preposition incorrectly spelt. The corrections are ‘accommodation',
to and not ‘loathe’. Hence a 'somersault' and accomplishment. Choice (D)
A diary is a book with spaces for each day of the year in
which you can write down things you have to do in the future. 23. The correct spellings of options A, B, and C are 'clamorous',
A dairy is a form where milk products are produced. Only a 'parochial' and 'precocious'. The word 'mammoth’ is correctly
suits the context. Choice (A) spell. Choice (D)
Triumphant Institute of Management Education Pvt. Ltd. (T.I.M.E.) HO: 95B, 2nd Floor, Siddamsetty Complex, Secunderabad – 500 003.
Tel : 040–27898195 Fax : 040–27847334 email : info@time4education.com website : www.time4education.com SM1001941/168
24. The correct spellings are 'manoeuvre, 'precipitate' and 6. Hold back is the correct phrasal verb to be used in this
'legitimize'. The word 'aggrandize' is correctly spelt. sentence. If you hold back something such as tears or
Choice (B) laughter, you make an effort to stop yourself form showing
how you feel. Choice (D)
25. Except the word 'acrimonious' all the remaining options are
incorrectly spelt. The correct spellings are 'fractious', 7. 'Catch up' is most appropriate in the given sentence. If you
fraudulent' and narcisstic. Choice (C) catch up on friends you have not seen for some time, you
talk to them and find out what has happened in their lives
26. The correct spellings of options, A, C, and D are 'finesse',
since you last talked together. Choice (C)
'virtuosity' and 'squalor'. Choice B does not need any
correction. Choice (B)
8. If a quality 'runs through' something it is in all parts of it.
27. Option A is correctly spelt. The correction in the remaining Hence the phrasal verb run through is apt in the context.
options are 'avaricious' 'adventurous' and 'posthumous'. Choice (A)
Choice (A)
9. The phrasal verb 'pull down' meaning to destroy a building is
28. Except the word 'preposterous' all the remaining words are apt in this sentence. Choice (B)
incorrectly spelt. The correct spellings are 'pusillanimous',
'timorous' and 'diminutive'. Choice (D) 10. To roll something back is to reduce the cost or price of
something. Only option C is logical in the given context.
29. Only 'primeval' is correctly spelt. The remaining options are Choice (C)
'medieval', 'coeval' and 'alluvial'. Choice (B)
30. Only 'deciduous' is rightly spelt. The remaining words are Explanatory Notes for questions 11 to 20:
'stupendous' 'tremendous' and 'horrendous'.
Choice (B) 11. To put on means to wear. Thus, the answer is ‘in the process
of wearing’. It is never used in the context of on oneself.
31. The word 'tempestuous' is correctly spelt. The correct dressing up someone else. One always ‘puts on’ something
spellings of the remaining options are 'conscientious', Choice (C)
'licentious' and 'malicious'. Choice (A)
12. To zero in on something means to zoom in on it, separate it
32. Options A to C are incorrectly spelt. The corrections are out as distinct and inspect closely. Thus, the answer is
'ambiguity' 'magnificence and 'solicitousness'. Choice D is ‘narrowed down to’. Choice (B)
correct. Choice (D)
13. The idiomatic phrase ‘boils down to’ means ‘can be
33. Only choice B is correctly spelt. The remaining corrections summarized as’. Choice (A)
are 'deferential', 'providential', and 'consequential'.
Choice (B) 14. ‘Brush up on’ means to improve or refresh one’s knowledge
on something. Thus, the answer is ‘refresh’. Choice (C)
34. The corrections are 'paean', 'panegyric', and 'acclamation'.
The word 'extolment' is correctly spelt. Choice (D)
15. Burn out is fatigue, frustration, or apathy resulting from
prolonged stress, overwork, or intense activity.
35. Options A to C are incorrectly spelt. The corrections are
Choice (C)
'euphonious' 'diaphanous' and 'mellifluous'.
Choice (D)
16. To call upon means to formally invite or request.
Solutions for questions 36 to 40: Choice (B)

36. The word 'ephemeral' is correctly spelt in Choice (B) 17. To clamp down on means to act strictly to prevent
Choice (B) something. Thus, the answer is ‘take strict action in order to
prevent’. Choice (A)
37. Option A is correctly spelt. Choice (A)
18. To dress up means to wear elegant clothes, for a special
38. The correct spelling is given in choice C. Choice (C) occasion. Choice (D)

39. Choice (D) has the correct spelling Choice (D) 19. To clam up means to become very quiet and reserved and
keep to oneself. Hence, the answer is ‘to be very quiet’.
40. Choice A is spelt correctly. Choice (A) Choice (B)

PRACTICE EXERCISE – 6 20. Back down means to withdraw or concede defeat. Thus, the
answer is retracted, meaning to go back upon.
Explanatory Notes for questions 1 to 10: Choice (D)

1. The phrasal verb 'square off' meaning to prepare to fight, Explanatory Notes for questions 21 to 35:
compete or argue with someone is appropriate here.
Choice (B) 21. Rifle means to search with the intention of stealing or taking.
Riffle is to shuffle or flip quickly through papers. Only a is
2. 'Cut back' is to do less of something or use something in apt.
smaller amounts. Hence option A is most suitable. Piquant means pleasantly tart or spicy. Pique is to arouse or
Choice (A) provoke. Only a makes sense here.
Esoteric means likely to be understood or enjoyed by only a
3. The phrasal verb "fall apart" is most appropriate, in the given
few people with a special knowledge or interest. Exotic
context. If an organization, system or agreement falls apart, means seeming exciting and unusual because it is
it fails or stops working effectively. Choice (C) connected with foreign countries. Only a is apt.
4. 'Lay aside' meaning to keep something usually money, for Ambiguous is used to describe a phrase or act with more
use in the future is apt in the given sentence. Choice (A) than one meaning or it is used to refer to something that is
unclear.
5. The phrasal keep off, meaning not to eat, drink or use Ambivalent means uncertainty and having conflicting
something that can harm you, is apt in the given sentence. attitudes and feelings. Only b makes sense in this context.
Choice (A) Therefore a a a b is a sequence. Choice (A)
Triumphant Institute of Management Education Pvt. Ltd. (T.I.M.E.) HO: 95B, 2nd Floor, Siddamsetty Complex, Secunderabad – 500 003.
Tel : 040–27898195 Fax : 040–27847334 email : info@time4education.com website : www.time4education.com SM1001941/169
22. Flout is to violate a rule or law. Flaunt is to show something Healthy means having good health and not likely to become
you are proud of to other people. Only a makes sense here. ill/sick. Healthful means promoting good health. The latter is
To work for the common good is to do something for apt in the given sentence. Hence b
common good so that it helps everyone. Hence b Credible means something which can be believed or trusted.
Perpetrate, meaning to commit a violent or harmful act, is the Creditable means deserving praise or approval. Only b is
appropriate word in the given sentence. Hence a appropriate in the given sentence. Choice (D)
Meritorious (Excellent; worthy) is the appropriate word to be
used in the given sentence. Meretricious meaning tawdry or 28. Childish means silly or immature. Childlike means simple
having little worth or value does not suit the given context. and innocent. The latter is appropriate in this sentence.
Hence b Choice (A) Hence b
Voracious means eating or wanting large amounts of food or
23. Beside means next to or adjacent to. Besides meaning in wanting a lot of new information and knowledge. Vociferous
addition to is the appropriate word here. Hence b. means expressing your opinions or feelings in a loud and
Historic is usually used to describe something that is so confident way. Only b is logically appropriate in the given
important that it is likely to be remembered. Historical usually sentence.
describes something that is connected with the study of Stature refers to the importance and respect that a person
history. Hence b is appropriate in this sentence. has because of his/her ability and achievements. Statute,
Connive in something is to seem to allow something wrong meaning a law that is passed by a parliament, council, etc is
to happen. Conspire is to agree together especially secretly, the appropriate word to be used here. Hence a
to do something wrong, evil or illegal. Only a is correct. Faint means very small; possible but unlikely. Feint
Pander is to do what somebody wants or try to please them, (especially in a sport) refers to movement that is intended to
especially when this is not acceptable or reasonable. make your opponent think that you are going to do one thing
Pamper is to take care of somebody very well and make when you are really going to do something else. The word
them feel as comfortable as possible. Only a makes sense faint makes sense in the given sentence. Hence a.
in the given context. Choice (A) Choice (B)
29. Adverse means negative or unpleasant and not likely to
24. Bemused is to show that you are confused and unable to produce a good result. Averse means not liking something
think clearly. Amused is to be entertained. Only a makes or wanting to do something. Only a is appropriate in this
sense here. Ethereal describes something that is light, airy sentence.
and intangible. Ephemeral refers to anything lasting for a Foul means unpleasant or very bad. Fowl is a bird that is
short period. Only a is appropriate in the given context. kept for its meat and eggs. Only a is correct in this sentence.
Accede is to agree to a request, proposal etc. To exceed Cache refers to a hidden store of things such as weapons.
something is to be greater than a particular number or Cash is money in the form of coins, notes, etc. Only a is the
amount. appropriate word to be used here.
An ascetic is a person who renounces all material comforts, Whole means complete or in one piece. Whole grain means
often for religious devotion. Aesthetic refers to the made with or containing whole grains, especially wheat. Only
philosophy of beauty or the pleasing qualities of something. a is correct in this sentence. Choice (A)
The former is appropriate in the given sentence. Hence a. 30. Wreak is to cause great damage or harm to somebody or
Choice (D) something. Wreck is to damage or destroy something.
‘Wreck havoc’ is an incorrect collocation. Therefore option b
25. Concurrent means existing or happening at the same time. does not make sense here. Hence a
Consecutive means following one after another in a series. Afflict is to affect somebody or something in an unpleasant
Only a is appropriate in this sentence. or harmful way. Inflict is to make somebody or something
Discreet is to be careful in what you say or do in order to suffer something unpleasant. Only b is more appropriate in
keep something secret or to avoid causing embarrassment this sentence. Between means happening between two
or difficulty for somebody. Discrete means independent of people. Among means involving three or more people. Only
other things of the same type. Only b makes sense in the a is correct in this context.
given context. Dissenting means having or expressing views that are
Abhorrent is causing hatred, especially for moral reasons. different from those that are officially accepted. Descent
Aberrant means not usual or not socially acceptable. Only a refers to the action of coming or going down. The former is
is more appropriate in the given sentence. correct in this sentence. Hence a Choice (B)
Anecdote is a short, interesting or amusing story about a real
person or event. Antidote is a substance that controls the 31. Its is a determiner which means belonging to or connected
effects of a poison or disease. Only a is logically appropriate with a thing, an animal or a baby. It’s which is a short from for
in the given sentence. Choice (B) it is or it has, is the correct option to be used here. Hence b
Sleight of hand refers to the skilful movements of your hand
26. Alleviate is to make something less severe. that other people cannot see. Slight, meaning very small in
Ameliorate is to make something better. Only a is precise in degree, does not make sense here. Hence a
the given context. The word archaic means old and no longer used. Arcane
Eager means very interested and excited by something that means secret and mysterious and therefore difficult to
is going to happen or about something that you want to do. understand. Only a makes sense here.
Anxious means feeling worried or nervous. Only b makes Decent means of a good enough standard or quality.
sense in this sentence. Descent refers to ancestry. The latter makes sense in the
Certainty refers to a thing that is certain. Certitude is a feeling given sentence. Hence b Choice (B)
of being certain. The former is appropriate in this sentence.
Hence b 32. Vain refers to something that does not produce the result you
Lightening is a verb that means to reduce the weight of. want. Vane is a blade that is moved by wind or water and is
Lightning, which refers to the electric discharge in the sky, is part of the machinery in a windmill. Only a is apt in this
the exact word to be used in this sentence. Hence a sentence.
Choice (D) Altogether means in total. All together means all in one place
or all at once. Only a is suitable in the given sentence.
27. Viscous means thick and sticky. Vicious means violent and Avenge which is a verb, means to punish or hurt somebody
cruel. Only b makes sense here. in return for something bad or wrong that they have done to
Aide is a person who helps another person, especially a you, your family or friends.
politician, in their job. Aid refers to money, food etc, that is People avenge something or average themselves on
sent to help countries in difficult situation. Only b makes somebody. You take revenge on a person. Only a makes
sense in this sentence. sense is this sentence.
Triumphant Institute of Management Education Pvt. Ltd. (T.I.M.E.) HO: 95B, 2nd Floor, Siddamsetty Complex, Secunderabad – 500 003.
Tel : 040–27898195 Fax : 040–27847334 email : info@time4education.com website : www.time4education.com SM1001941/170
Broach is to begin talking about a subject that is difficult to Explanatory Notes for question 5:
discuss especially because it is embarrassing or because
people disagree about it. Broach is a piece of jewellery that 5. Balk is to place an obstacle in the way of, hinder, thwart.
can be fastened on clothes. The former is the appropriate Baulk is to be unwilling to do something or become involved
word to be used in this sentence. Hence a Choice (D) in something because it is difficult, dangerous etc. The latter
is apt in this sentence. Hence b
33. Acumen is the ability to understand things quickly and well. Incredible is impossible or very difficult to believe.
Acuity is the ability to think, see or hear clearly. The former Incredulous is not willing or not able to believe something.
is appropriate in the given sentence. Hence b The latter is apt in this sentence. Hence b
Allusions means containing references to another person or The noun practice means repeated performance or
subject in an indirect way elusive is difficult to find, define or systematic exercise for the purpose of acquiring skill or
achieve. Clearly a suits the context. proficiency. Practise is the verb form of practice. The former
Who’s is the appropriate word to be used in the given is appropriate in the given context. Hence a
sentence. Who’s who refers to the list or book of facts about Waive means to refrain from claiming or insisting on. Wave
famous people. Whose is used in questions to ask who us a disturbance on the surface of a liquid body, as the sea
something belongs to. Only a is appropriate in this sentence. or lake, in the form of a moving ridge or swell. Only a fits into
Portable refers to something which can be carried form place the given sentence. Choice (A)
to place. Potable refers to water which is safe to drink .
Hence b Choice (D) PRACTICE EXERCISE – 7
Explanatory Notes for questions 1 to 5:
34. Reluctance is the feeling of hesitation before doing
something because you do not want to do it or you are not 1. The words 'apart from', 'a famous victory' are clear-cut clues
sure that it is the right thing to do. Reticent is being reserved to a positive note. Except Choice (A), the rest of the answer
or uncommunicative. The latter is apt in the given sentence choices are a mixture of both positive and negative words.
Hence b. 'Garnering' is obtaining or collecting something, such as
Through means past or beyond. Thorough means done information, support etc; it is 'gathering' or 'acquiring'. Though
completely with great attention to detail. Only b is apt in this 'bestowing' is awarding, granting, giving or presenting,
sentence. 'rebuffing' is an abrupt or unkind rejection. 'Denying' and
Ascent is the act of climbing or moving up. 'thumbing down' are synonyms. Choice (A)
Assent means official agreement to or approval of
something. Only b makes sense in this sentence. 2. The connections between physical, emotional and mental
Breath, which is a noun, refers to the air that you take into health are accepted (regarded favourably) not regulated
your lungs and send out again. The word breathe which is a (control or maintain the rate or speed) nor acclaimed (praise
verb and refers to the act of breathing does not make sense enthusiastically) or disproved (prove to be false). Regular
in the sentence. Only a is appropriate in this sentence. exercise is to achieve optimal (best or most favourable)
Choice (B) health. It is definitely not minimal (minimum amount) or even
maximum (regulated cannot fit in the first blank) or salubrious
35. Conscience is the part of the mind that tells you whether your (health giving) health. Choice (B)
actions are right or wrong. Conscious means being aware of
something. Only a is apt here. 3. The sentence talks about medicine and micro-organisms,
like bacteria. You would want a medicine to destroy the
Principal means most important or main. Principle is bacteria not replicate (reproduce), fortify (invigorate,
a moral rule or a strong belief that influences your actions. encourage) or nourish it. Antibiotics is apt in the first blank.
Clearly a suits the context. Nutrients and capsules are not medicines. Steroid is
Already means before now or before a particular time in the unsuitable since we don't want to replicate bacteria.
past. All ready means completely ready or ready in all Choice (A)
aspects. Only b is apt in this sentence. Rational refers to the
behaviour, ideas etc, that is/are based on reason rather than 4. The context of the sentence, even with the blanks indicates
emotions. that interest rates have dropped (lowest level) and that this
was a deliberate attempt to tackle deflation. This context
Rationale is the principles or reasons which explain would need ‘reduction’ and ‘prevent. Choice (C)
a particular decision, course of action, belief Only b is
appropriate in this sentence. Choice (C) 5. A food recipe can be learnt from a set of instructions but
hospitality cannot. Since a contrast is implied 'unlike' is apt
EXPLANATORY NOTES FOR ADDITIONAL QUESTIONS in the first blank. Further 'only through example' implies that
the word in the blank is imbibe (absorb) rather than instruct
Explanatory Notes for questions 1 and 2: (teach, direct) or observe (perceive, notice) or learn (acquire
knowledge or skill). Choice (D)
1. The idiom, by the skin of one’s teeth means, just barely or
very narrowly. Thus, the answer is ‘very narrowly’. Explanatory Notes for questions 6 to 10:
Choice (B)
6. The word has been inappropriately used in Choice (D).
2. To keep one’s nose to the grindstone means to work The appropriate usage is ‘pitched in with’ which means to
doggedly or persistently. Hence, the answer is ‘works very contribute. Choice (D)
hard’. Choice (A)
7. The word ‘right’ is inappropriately used in Choice (D).
Explanatory Notes for questions 3 and 4: The right phrase is ‘in her own right’ not “in her own rights”.
‘In her own right’ means “as a result of one’s own claims, or
3. Option (A) ‘stand in’ means to take the place of someone efforts”. Choice (D)
temporarily, Choice (B) ‘stand down’ means to step down.
Choice (C) ‘stand over’ means to overlook, Choice (D) ‘stand 8. The word ‘rate’ has been inappropriately used in
back’ means to let something happen. Choice (B) is the apt choice A. The rate of something is not the correct expression
choice. Choice (B) to be used. When the price of something is referred to, it is
indicated by the word ‘cost’. Choice (A)
4. Option (A) means to accede to or support a plan or
suggestion and is apt here. Option (B) means parts of 9. The word ‘stop’ has been inappropriately used in choice B,
something that break off and drop. Option (C) means ‘to fall’. because the word stop should be followed by the preposition
Option (D) means to be tricked. Choice (A) ‘over’ in order to mean a ‘halt’. Choice (B)

Triumphant Institute of Management Education Pvt. Ltd. (T.I.M.E.) HO: 95B, 2nd Floor, Siddamsetty Complex, Secunderabad – 500 003.
Tel : 040–27898195 Fax : 040–27847334 email : info@time4education.com website : www.time4education.com SM1001941/171
10. The word ‘ill’ has been inappropriately used in choice D. The 25. ‘Abjuring’ and ‘abdicating’ can easily be ruled out. They
correct expression to be used is a ‘sick child’. mean renounce or give up. ‘Spurning’ means to reject. The
Choice (D) clubs are averse to reform and continue their old practices,
unmindful of others opinions. ‘Ensconced’ conveys this idea.
Explanatory Notes for questions 11 to 15: Choice (B)
11. ‘Spurt’ refers to a sudden increase in something for a short Explanatory Notes for questions 26 to 30:
period of time. Therefore ‘surge’ is the best substitute.
Cascade (to flow downward in large quantities), torrent 26. Balmy means ‘warm’, barmy means foolish and crazy.
(a large amount of something that comes suddenly and Broach is to open a sensitive topic. A brooch is a large
violently) and flurry (an occasion where there is a lot of decorative pin or clasp. To canvass is to propagate one's
activity and excitement) are clearly unsuitable as idea while canvas is a surface for painting. Bough refers to a
replacements. Choice (A) branch a of a tree and bow refers to the hull of a ship. Hence
baab is the correct sequence. Ans: baab
12. ‘Whittled down’ means to reduce by repeated actions and the
27. Censure is harsh criticism and to censor is to ban. The critics
word that conveys this sense is ‘eroded’. Choice (C)
don't have a right to ban a movie hence censure fits better.
13. ‘Conciliatory’ means to reconcile and the word propitiatory Complacent is self satisfied and complaisant is extremely
meaning placatory, is similar in meaning to the word acquiescent. To titivate to become more attractive which
conciliatory. Choice (A) makes more sense here than titillate which means to tempt.
'Palate' is one's appreciation of various tastes whereas
14. ‘Trounce’ means to defeat decisively and the word that palette is a paint mixing board. abaa is the appropriate
conveys this meaning is ‘rout’. Choice (D) sequence. Ans: abaa
15. ‘Incontrovertible’ means ‘indisputable’ or ‘conclusive’. 28. Immure means to confine and inure is to make insensitive.
Choice (A) The residents were more likely to be confined to their houses
than become insensitive to them. Dual is for two people and
Explanatory Notes for questions 16 to 20: a duel is a flight. Wreath is a circular floral decoration usually
adorned on a tomb. Wreathe is surround. Imply is to state
16. 'Vituperative' and 'scurrilous' both mean bitter and abusive. indirectly and infer is to conclude. Therefore the correct
'Pristine' is synonymous with 'immaculate'. Both mean sequence is aaaa. Ans:aaaa
‘perfectly clean.’ Reclusive (isolated, solitary) and
melancholic (gloomy), loud tenacious (inflexible) and strident 29. Eschew is to oppose and espouse is to support. To minister
(loud, hoarse) are not synonymous. Choice (A) is to attend to someone's needs. Administer is to supervise.
Inculpate is to blame and exculpate is to absolve or free from
17. 'Influential' and 'impuissant' (impotent) are antonyms. Similarly blame. Pursuit is the act of chasing or striving. Perusal is
intransigent (obstinate) and ductile (adaptable) are antonyms. careful reading. C is correct. Therefore the correct sequence
The words taciturn (curt; reserved) and stupefied is aabb. Ans: aabb
(astonished), diaphanous (thin, transparent) and awkward,
salacious (indecent) and gratuitous (spontaneous) are not 30. To beseech is to beg and bespoke is custom made. Conflate
antonyms. Choice (B) is to create by mixing and conflagrate is to set fire to. Indigence
is poverty and indolence is laziness. As the sentence talks
18. 'Ballista' is a catapult for hurling boulders. 'Trebuchet' is a about inheriting and finances indigence is more appropriate.
weapon used for hurling stones. A harpoon is a long, heavy Inept is unskilled and adept is skilled. Since it drew flak, the
spear attached to a rope, used for killing large fish or whales. performance was inept. Option C is correct. Therefore the
A boomerang is a curved stick that when thrown in a particular correct sequence is aaaa. Ans:aaaa
way, comes back to the person who threw it. A bazooka is a
long tube-shaped gun, fixed from the shoulder, that is used to Explanatory Notes for questions 31 to 35:
shoot missiles at military vehicles. Choice (D)
31. 'Collate" means to analyse and compose or to assemble
19. 'Bureaucracy' is the form of government run by the state officials. from different sources. "mustered", 'composed' and
'Hierocracy' is a rule by priests. 'Democracy' is run by elected examined will fit the blank, but "encumbered" which means
representatives and not the constitution. ‘Dictatorship’ may or to burden can be ruled out. Choice (A)
may not be by a 'monarch' Choice (C)
32. 'Fragmented' means disconnected. The inappropriate word
20. 'Vigilantes' work to promote morals in their community. is 'immoderate' [excessive]. Shattered, splintered broken all
'Arbiters' work to settle 'disputes'. An 'abbeter' gives mean fragmented. Choice (C)
assistance, an 'emperor' rules', an ‘acolyte’ is a ‘follower’. 33. "Bereft" means deprived. "emitted" is the inappropriate word
Choice (C) which means to send out or put out. 'Devoid', 'robbed' and
Explanatory Notes for questions 21 to 25: 'divested', all mean deprived. Choice (B)
34. "Beset' means to attack persistently or to cover round with.
21. Both ‘resorts’ and ‘haunts’ fit in the blank. ‘Resort’ is a holiday
Since the mention is to ignorance, it will take the latter
place. ‘Haunt’ is a place of frequent visit since it is enjoyable.
meaning. Ignorance can 'besiege' [to crowd around
Keeping in mind the tone of the passage and the words
oppressively] it can 'perplex' [puzzle or bewilder] and it can
‘clubs of colonial affectations’, ‘haunts’ will be the most
enclose [shut in on all sides]. But it cannot 'exert' [strive].
appropriate word. Choice (C)
Choice (B)
22. ‘Galling’ and ‘exasperating’ can be ruled out since they mean
35. 'Stultify' means to make ineffective or futile esp. as a result
‘annoying’. The passage is not discussing, ‘erudite’ or
of tedious routine. ‘Benumbing’ [paralyzing], ‘constraining’
learned people but it refers to the ‘privileged’ people which
(limiting) and ‘hampering’ (hindering) can fit the blank.
means respected and admired, and ‘hallowed’ conveys this
'Stirring effect' would mean a result of motivation. It does not
meaning. Choice (C)
fit the context. Choice (B)
23. After the word ‘hallowed’, the only word that would suit the
blank is ‘urbane’. It means polished and high-class. The EXPLANATORY NOTES FOR ADDITIONAL QUESTIONS
other three words do not pertain to the paragraph.
Choice (D) Explanatory Notes for questions 1 and 2:

24. The word that suits the context is ‘inertia’. It means to be 1. extending and identified: When we consider the two words
unwilling to move or be active. It is in keeping with the idea together the only right option is D. In other choices, one
of ‘haughty resistance to criticism’. It also takes the word may appear to be correct as ‘relating’ in option A, but
preposition ‘against’ after it. Choice (B) the other one will not suit. Choice (D)
Triumphant Institute of Management Education Pvt. Ltd. (T.I.M.E.) HO: 95B, 2nd Floor, Siddamsetty Complex, Secunderabad – 500 003.
Tel : 040–27898195 Fax : 040–27847334 email : info@time4education.com website : www.time4education.com SM1001941/172
2. Poor state of food and nutrition ‘security’ is to be Explanatory Notes for questions 5:
‘addressed.’ ’Value’, ‘shares’ or ‘standard’ cannot be
‘addressed.’ Options like ‘directed’, ‘attended’ and ‘focussed’ 5. fulminate – to burst out in anger : appropriate
do not go with ‘on a war footing.’ culminate – to bring to a close : inappropriate
Choice (A) flounder – to struggle, to have difficulty coping : appropriate
founder – to sink, to fail : inappropriate
Explanatory Notes for questions 3 and 4: integrity – having high standards of ethical and moral
behaviour : inappropriate
3. The words eclat (confidence; flair) and panache (self-confidence) honesty – speaking the truth : appropriate
share a synonymous relationship. Among the given options house – the residence : appropriate
the word rectitude (integrity) and probity (morality) share a home – the residence and the circumstances and
synonymous relationship. Choice (A) relationships within : inappropriate
egotist – a braggart, one who thinks he's the centre of
4. The words humane (compassionate) and fiendish attention : appropriate
(wicked; cruel) are antonyms. Among the given options the egoist – one who is self-centred and does not think of others
words defamatory and complimentary are antonyms. : inappropriate
Choice (B) Therefore the correct sequence is aabaa Ans: aabaa

EXPLANATORY NOTES FOR GRAMMAR EXERCISES


PRACTICE EXERCISE – 1 13. The sentence talks about one workshop that belongs to both
Raju and Ravi. Hence the possession should be indicated by
1. Army is a collective noun that takes the singular verb. Hence adding an ‘’s’ (apostrophe and s) to Ravi. Hence the
the correction is army has …… is poised. correction is Raju and Ravi’s workshop ………. .
(Refer to V02; Section–II; Unit-I; Rule 2) (Refer to V02; Section–II; Unit-I; Rule 14)
2. Youth represents a stage of life and hence is an uncountable
14. The error in the sentence is because of the apostrophe (‘s)
abstract noun. Hence it takes the singular verb. Hence youth
to the word Robert’s. The intended idea is Robert, the
is wasted is the correction.
author’s view. It is because, when two nouns refer to the
(Refer to V02; Section–II; Unit-I; Rule 4)
same person, the possessive sign is added to the latter.
3. Courage is an abstract noun, hence should go with the (Refer to V02; Unit-I; Section–II; Rule 15)
singular form of the verb. The correction is Courage makes
us …. . 15. It is incorrect to say novel’s first part because inanimate
(Refer to V02; Section–II; Unit-I; Rule 2) things do not take an apostrophe. The correction is ‘the first
part of the novel’.
4. Aircraft is a noun that has the same form for the plural as it
has for the singular. Hence aircrafts is incorrect. Hence the 16. The correction is sisters-in-law. It is a compound noun.
correction is twenty fighter aircraft ……. . Hence, the plural is formed by adding ‘s’ to the first word.
(Refer to V02; Section–II; Unit-I; Rule 6) Father-in-law’s is correct because the possessive of a
compound noun is formed by adding apostrophe (’s) to the
5. The noun cattle is always used in the plural. Hence the last word of the compound noun.
correction is cattle have gathered …… . (Refer to V02; Unit-I; Section–II; Rule 16)
(Refer to V02; Section–II; Unit-I; Rule 10)
6. Information is uncountable hence takes the singular. Hence 17. The correction is the Prime Minister of Isreal’s plan. When a
the correction is is relevant. noun or a title consists of several words, the apostrophe and
(Refer to V02; Section–II; Unit-I; Rule 7) s (’s) is attached only to the last word.

7. Machinery is a noun that is always used in the singular. 18. The error is because of the absence of the
Hence the correction is There is no machinery for ………… apostrophe (’s) to the word ‘Charles’. The ‘correction is
. Prince Charles’ friend because Charles is a proper noun
(Refer to V02; Section–II; Unit-I; Rule 7) which ends with ‘es’ hence the apostrophe is used but ‘s’ is
omitted.
8. Scissor is incorrect. It is used only in the plural. Hence the
correction is scissors that he has …… are not very sharp….. 19. Bradman be`comes the common noun in the sentence
(Refer to V02; Section–II; Unit-I; Rule 8) because when the article is placed before a proper noun it
becomes a common noun. The intended idea is that Sachin
9. Measles appears to be in the plural form, but is actually Tendulkar is the best player of this age. The correction is .....
singular in usage because it is the name of a disease. Hence the Bradman of ....... .
the correction is measles is a disease that generally afflicts (Refer to V02; Unit-I; Section–II; Rule 1)
………. .
(Refer to V02; Section–II; Unit-I; Rule 9) 20. The sentence has two errors. The first error is the mismatch.
The correction is a mismatch. The second error is an
10. Six-hundred runs is considered as one unit and hence takes information. The correction is ‘the information’ i.e., the idea
the singular form. Hence the correction is six-hundred runs expressed is there is a mismatch in the information sent,
is ……… . hence the correction.
(Refer to V02; Section–II; Unit-I; Rule 11)
21. The words heroine of the novel defines the name Isabella.
11. Uncountable nouns like advice are used with expressions Hence, the heroine of the novel is the correction. Using the
like a piece of in the singular and with much and a lot of, definite article indicates that the reference is being made to
plenty of in the plural. Therefore, the correction is a piece of someone in particular.
advice.
(Refer Q 6) 22. The correction is the Atlantic. It is the name of an ocean,
hence should be preceded by the.
12. The correction is was felicitated because the poet who is also (Refer to V02; Unit-VIII; Section-II; Rule 36)
a politician was felicitated. If the given sentence reads the
poet and the politician, then, it takes the plural because it 23. The sentence has two errors (1) furniture is never used in
shows that the reference is to two different people. plural (2) ... there is no room .... not place.
Triumphant Institute of Management Education Pvt. Ltd. (T.I.M.E.) HO: 95B, 2nd Floor, Siddamsetty Complex, Secunderabad – 500 003.
Tel : 040–27898195 Fax : 040–27847334 email : info@time4education.com website : www.time4education.com SM1001941/173
24. The word team is a collective noun but the words split into PRACTICE EXERCISE – 2
two groups indicate that the members of the team are being
talked about separately, hence the correction is they were 1. The error is in the ordering of the pronouns. When we talk
split ...... . about a failure or negative, the order is 1st person, 2nd person
(Refer to V02; Unit-I; Section-II; Rule 2) and 3rd person. Hence the correction is I, you and Michael
must take the blame …….
25. Public is a noun that always takes the plural form of the verb. (Refer to V02; Section–II; Unit-II; Rule 2)
Hence the public have been …….. .
(Refer Q 5) 2. The sentence has two errors. One is in the ordering of the
pronouns. When we talk about something good or positive,
26. The correction in the sentence is a band of robbers has the order is II person, III person, I person. The second error
because band is a collective noun and it should take the is because of the use of the pronoun I. It should be me. The
singular form of the verb. idea intended is as follows.
(Refer Q 1) Whom does the management plan to congratulate?
You, Mary and me. Hence the correction.
27. six scores boxes is incorrect. The correction is six score (Refer to V02; Section–II; Unit-II; Rule 3)
boxes. Nouns used in counting (score), when preceded by
numerals, are unaltered in the plural. 3. The error is because of the use of the pronoun me. The
(Refer Q 10) correction is It was I ……. It + be verb always takes the
subjective case of the pronoun (i.e., I, we, you, he, she, it,
28. The correction is two loaves of bread or two slices of bread they and the be-form verbs are is/was/are/were have been,
because it is an uncountable noun and is referred to in parts. has been had been).
(Refer to V02; Section–II; Unit-II; Rule 9)
29. Some uncountable nouns like whereabouts can be used with
4. When neither ….. nor or either …… or are used in the
a plural verb as well as a singular verb. Hence whereabouts
sentence, the verb agrees with the second subject. In this
………. is is correct. The sentence requires no correction.
sentence, the second subject members of the team is plural
and hence the verb should be have. The correction is …..
30. The correction is for goodness’ sake. It is because the last
members of the team have …….
syllable of the word ends with ‘s’ and is followed by sake.
(Refer to V02; Section–II; Unit-II; Rule 7)
Hence, the apostrophe is used but ‘s’ is omitted. Hence the
correction. 5. The error in the sentence is due to the word that. The
(Refer to V02; Section-II; Unit-II; Rule 17) correction is than those because her house is being
compared with the houses of her friends and relatives.
EXPLANATORY NOTES FOR ADDITIONAL QUESTIONS The appropriate pronoun is those of. Hence the correction.
(Refer to V02; Section–II; Unit-II; Rule 17)
1. The correction in the sentence is Raju, my cousin’s letter.
(Refer Q 16) Choice (A) 6. The error in the sentence is in the usage of the pronoun me.
The pronoun to be used is I. When a personal pronoun is
2. The expression ten year’s experience is incorrect. It is a plural connected by a conjunction with some other word in the
noun; hence the ten years’ apostrophe is after ‘s’ – ten years’ subjective case, it must also be in the subjective case.
experience. Choice (D) (Refer Q 3)

3. The correction is a Gandhian revolution because the 7. The correction is It is art because it refers to art. That, as
reference is to a type of revolution. Choice (B) given in the sentence is erroneous because it is an adjective
and not a pronoun.
4. The word economics in the context does not refer to a (Refer to V02; Section–II; Unit-II; Rule 16)
subject but to the economic details of the situation, hence
should be preceded by the. Choice (D) 8. The correction is him and her because if a sentence begins
with let, it takes a pronoun which is in the objective case.
(Refer to V02; Section–II; Unit-II; Rule 10)
5. The error is in the expression the London University. The
correction is the University of London. If it is written as 9. The correction is one finds because the pronoun must be
London University it does not take any article. consistent throughout the sentence.
Choice (B) (Refer to V02; Section–II; Unit-II; Rule 11)
10. The sentence has two errors. The first correction is now it’s
6. The error in the sentence is goes to the church. The being ––. The pronoun it takes the apostrophe (‘s) because
sentence should read goes to church because when a place the context does not require the possessive case. The
is visited or used for its primary purpose, it does not take the. intended idea is it is. The second correction is its deliberate
In the second part of the senctence the church is correct destruction i.e. The usage of apostrophe is erroneous as the
because it refers to the particular church which has been possessive case is intended.
referred to in the first part of the sentence. Choice (A) (Refer to V02; Section–II; Unit-II; Rule 8)
7. The correction in the sentence is the Nilgiris because, the 11. The correction in this sentence is reminds. Each and every
names of mountains, rivers, oceans, hills etc take the definite are distributive pronouns which take the verb in the singular
article. form.
(Refer to V02; Unit-VIII; Section-II; Rule 37) (Refer to V02; Section–II; Unit-II; Rule 5)
Choice (B)
12. The correction here is everyone does not get what is due to
8. The correction is ... colourful stone-material nouns have no him. The pronouns everyone, everybody take singular form
plural forms. Choice (D) of the pronoun.
(Refer to V02; Section–II; Unit-II; Rule 22)
9. Jury, a collective noun, needs to be looked at on a ‘divided
entity as the members all con’s take a simple seat. ‘Their 13. The words taught her own are erroneous. The correction is
seats’ is correct. Choice (B) taught herself i.e. she learnt without anyone’s assistance.
14. The correction is by its arm because child is of Common
10. A collection of crows is called ‘a murder’ It requires an article: Gender i.e. it is a noun that denotes either a male or a
‘a murder: Choice (B) female.
Triumphant Institute of Management Education Pvt. Ltd. (T.I.M.E.) HO: 95B, 2nd Floor, Siddamsetty Complex, Secunderabad – 500 003.
Tel : 040–27898195 Fax : 040–27847334 email : info@time4education.com website : www.time4education.com SM1001941/174
15. The sentence has two errors. The first correction is guests EXPLANATORY NOTES FOR ADDITIONAL QUESTIONS
whom…. not who because the word guests is the object of
the sentence. Hence we use whom which is also in the 1. The corrections are one’s knowledge and throughout one’s
objective case. The second correction in the sentence is life.
enjoyed themselves. The verb enjoy is transitive which can (Refer to V02; Section–II; Unit-II; Rule 11) Choice (C)
be used reflexively. In the context enjoy is used to denote an
action which cannot be done to anyone or anything. Hence 2. The correction is applicable because either means anyone
it takes the self pronoun i.e. enjoyed themselves. of the two, hence it takes the singular form of the verb.
(Refer to V02; Section–II; Unit-II; Rule 13; Rule 14) (Refer to V02; Section–II; Unit-II; Rule 18) Choice (C)
16. The sentence has no error. The pronoun none can take both 3. The correction is It is they …..
the singular and plural forms of the verbs. Hence no (Refer Q 3) Choice (A)
correction is required. Further, the pronoun he is appropriate
in the context. The sentence conveys the idea that of all the 4. The correction is have ever lived, because the relative
men I know, he is less inclined i.e. it is in the subjective case pronoun who refers to the word novelists which is plural.
not the objective case. Hence takes the plural form of the verb.
(Refer to V02; Section–II; Unit-II; Rule 12) Choice (C)
17. The first correction is each other. When there are only two
people being referred to, each other is used. The second
5. The correction is whom because the reference is to the girl
correction is My grandmother and I.
who is the object of the clause all of us thought so highly of.
(Refer to V02; Section–II; Unit-II; Rule 19 (1))
For a better understanding you can go about like this. Who
18. The correction here is he cannot help. The subject of the thought highly of the girl? (all of us) All of us thought highly
sentence is anyone. It does not take one but he (when the of whom? (The girl). Who should be changed to whom.
gender is not specified). (Refer to V02; Section–II; Unit-II; Rule 13) Choice (A)
(Refer to V02; Section–II; Unit-II; Rule 22)
6. The correction is who they say because the relative pronoun
19. The correction is that I have ever read. The pronoun that is refers to the noun in the subjective case.
used to refer to the book. (Refer to V02; Section–II; Unit-II; Rule 12) Choice (B)
(Refer to V02; Section–II; Unit-II; Rule 17)
7. The sentence has two corrections. The first is was asked
20. The correction is has to help. because every takes singular form of the verb.
(Refer Q 4) The second correction is his or her text book.
(Refer to V02; Section–II; Unit-II; Rule 5) Choice (B)
21. The first error in the sentence is because of the wrong
placement of the pronoun each. The correction is each 8. The correction in the sentence is if it excludes because the
happy with ___. The idea intended is that each boy was reference is to account which is the singular.
happy. The second error is because of the plural form of the Choice (D)
pronoun (their) when each is used in a sentence, it takes the
9. It should be ‘to practise’. Choice (D)
singular. Hence his prize is the correction.
10. The correction in the sentence is home-made. The word self-
22. It is incorrect to say each of us did not like ….. The correction
made does not convey the intended meaning.
is None of us …. liked the movie.
Choice (C)
23. The correction is whom I trust the most. The relative pronoun
PRACTICE EXERCISE – 3
who refers to the first part of the sentence. He is my brother,
which is the subject and the next part of the sentence should 1. The correction is ‘Some primitive societies believe that
take the relative pronoun whom because he is the one whom heavenly bodies are divine beings’. ‘Believe’ is a ‘verb of
I trust the most. i.e. it is in the objective case. Hence the perception’. Hence it is incorrect to use it in its continuous
correction. form. Hence the correction.
(Refer to V02; Section–II; Unit-II; Rule 15)
2. The correction is ‘This flower smells sweet’. Verbs
24. The error in the sentence is stories of her’s. The possessive associated with the senses are not used in the continuous
case of the pronoun and does not take the apostrophe(’s). tense. Hence the correction.
Hence the correction ….. short stories of hers …...
(Refer to V02; Section–II; Unit-II; Rule 8) 3. The correction is ‘She is a gifted sportsperson who has been
displaying her talent on the tennis court for the past two
25. The correction in the sentence is which but not who because years’. When an action starts in the past and is continuing till
the relative pronoun should agree with its antecedent i.e. the the present, the present perfect continuous tense is to be
noun which immediately precedes the relative noun. used. In the given sentence the words ‘for the past two
(Refer to V02; Section–II; Unit-II; Rule 16) years’ suggest that the action started sometime in the past
26. The correction in the sentence is everyone of us must remain and the verb ‘displaying’ suggests that the action is still
accountable because the subject of the sentence is Each of going on. Hence, the present perfect continuous form ‘has
us. been displaying’ should be used. The sentence can also be
(Refer to V02; Section–II; Unit-II; Rule 5) corrected as, ‘She is a gifted sportsperson who has
displayed her talent on the tennis court for the past two
27. The correction here is one another. When the reference is to years’.
more than two persons, one another is used.
(Refer to V02; Section–II; Unit-II; Rule 19) 4. The correction is ‘My brother is not at home, he went out
in the morning’. In the given sentence, ‘in the morning’
28. The correction is is important. indicates the past tense. Hence ‘has gone out’ which is in
(Refer Q 11) the present prefect is incorrect.
29. The correction is it is they….. 5. The sentence can be corrected in two ways, ‘Since its
(Refer Q 3) inception, the organization has relied heavily on the
30. The relative pronoun who refers to the noun boys which is sincerity of its employees’. (or) ‘Since its inception the
plural, hence ‘have’ (i.e., when the relative pronoun becomes organization has been relying heavily on the sincerity of
the subject of the sentence, then the verb agrees in number its employees’. The sentence takes the present perfect
and person with the antecedent of the relative pronoun). The tense (has relied) or the present perfect continuous tense
correction is …. boys who have .... . (has been relying) because of the word ‘since’.

Triumphant Institute of Management Education Pvt. Ltd. (T.I.M.E.) HO: 95B, 2nd Floor, Siddamsetty Complex, Secunderabad – 500 003.
Tel : 040–27898195 Fax : 040–27847334 email : info@time4education.com website : www.time4education.com SM1001941/175
6. The correction is ‘Ten people had been recruited and five 19. The correction is ‘More than two-thirds of the earth is
of them have resigned’. The idea expressed is a sequence covered with water’. The error is one of subject-verb
of events. The second part of the sentence requires a concord. The reference is to area, hence it takes the singular
different tense from that used in the first part of the sentence. form of the verb.
Hence the correction is ‘had been recruited’ and ‘have
resigned’. 20. The correction is ‘This is one of the most interesting
novels that have been published this year’. The word
7. The correction is ‘I have visited England and I know the ‘that’ in the sentence refers to ‘novels’ which is plural.
place quite well’. When we refer to an action which is Hence, a plural verb is required.
completed and the effect of which is felt now, we use the
present perfect tense i.e., (have + visited). 21. The correction is ‘Who hung this picture on the wall?’
When ‘hang’ is used to refer to the hanging of someone or
8. The correction is ‘A major change since the first stock the punishment given to a person, then both the past form
market boom of the 1990s has been the readiness of the and the past participle form will be ‘hanged’. But in the given
corporate sector to take risks’. The explanation given for sentence, the reference is to pictures. Hence both the past
question (5) holds good. and the past participle forms would be ‘hung’.
9. The correction is ‘Last week, a web-based newspaper 22. The correction is ‘Did anyone raise an objection to what
covering issues pertaining to South Asia, featured an you said?’. ‘Rise’ means ‘to get up’ or ‘to come up’ which
interesting report on Baluchistan’. The given sentence is is intransitive (i.e. does not require an object after it). In the
erroneous because of inconsistency in the tense. i.e., when given sentence ‘raise’ is the appropriate verb. It means to
adverbs of past time like ‘last week’ are used, the sentence bring something to someone’s attention’.
should be in the simple past tense. Hence ‘has featured’ is
incorrect, it should be ‘featured’. 23. The correction is ‘She knew that her end was near and lay
on her deathbed, praying to the Almighty’.
10. The correction is ‘This obscure village is on the Indo- The verb ‘lay’ is the past tense of the intransitive verb ‘lie’.
Bangladesh border where people had to walk across (lie, lay, lain). Hence the correction.
paddy fields before the border road was constructed a
couple of years ago’. The sentence refers to two past 24. The correction is ‘When I entered the house I found that
situations which require the verbs in their past form. Hence everything had been stolen’. The sentence is in the
‘had to walk’ is correct. passive voice. Hence, the verb should take the past
participle form of the verb.
11. The correction is ‘It is high time the teachers completed
the syllabus and started revising it’. When the phrase ‘It 25. The correction is ‘She said that she was busy and could
is high time‘ is followed by a verb in its past form it not meet anyone that day’ because ‘told’ is a transitive
expresses the idea of an action not happening when it was verb which requires an object after it. But in the given
supposed to. Hence the correction. sentence, the object is missing. Hence the correction is ‘she
said that she was busy…..’.
12. The correction is ‘He had just entered the class when the
bell rang’. The given sentence talks about two actions. The 26. The correction is ‘The students were happy when the
first completed action is ’he entered the class’ and the next principal promised them a holiday’. ‘Promised’ is a
completed action is ‘the bell rang’, which is in past tense. transitive verb which requires a direct object and an indirect
When the reference is to two past actions, the first completed object. Hence the correction is ‘promised them’.
action should take ‘had’ (past participle).
27. The correction is ‘In the olden days, people lived in small
13. The correction is ‘As soon as the teacher saw what the caves on the mountainside’. The word 'live', when used in
boy had done, she admonished him’. The explanation the continuous form does also mean ‘to exist’ which is
given for question (12) holds good. awkward in the given context. Hence the correction.
14. The correction is ‘I will have completed my work by this
28. The correction is ‘Bread and butter is the staple food of
time tomorrow’ because, with the use of ‘by’ context in the
most people in Europe’. The error is one of subject-verb
sentence is about a completed period of time in the future.
agreement. The correction is ‘Bread and butter is’. It is a
Hence the future perfect tense should be used i.e., (will/shall
single phrase which refers to a single idea. Hence it takes
+ have). If the ‘by’ were to be left out, the sentence would
the singular verb.
simply be –‘ I will complete my work tomorrow.
29. The correction is ‘Thorough knowledge of Mathematics
15. The correction is ‘Although most of us wish that
as well as English is very important for you to do this
employment opportunities would increase, not many
course well’. This is because the subject of the sentence is
new projects are being initiated by the government’. The
‘knowledge’ which is abstract and uncountable. Hence, it
sentence refers to a desire. Hence ‘would’ should be used
takes the singular form of the verb.
in the sentence. We are talking about a possibility, which is
more appropriately expressed using ‘would’. 30. The correction is ‘While I was in Bombay, I used to play
16. The correction is ‘Today, we live in a mind-boggling golf every week’. (or) ‘While I was in Bombay, I played
diversity, where, industrial and social development has golf every week.’ As given in the sentence, ‘was playing’ is
created many different kinds of occupations’. In the given incorrect because the reference is to a regular action of the
sentence ‘industrial and social development’ indicates a past. Hence, it should be expressed in simple past.
single idea. Hence the singular form of the verb should be
EXPLANATORY NOTES FOR ADDITIONAL QUESTIONS
used.
17. The correction is ‘A class of engineers, called ‘knowledge 1. The correction is ‘If I were you, I would not accept the
engineers’, is emerging to support the experts who are proposal’. Using ‘was’ is incorrect because we are talking
employed as independent consultants’. The subject of the about an imaginary situation. Hence the sentence takes
sentence is ‘class’ which is a collective noun and takes the ‘were’. Choice (A)
singular form of the verb i.e. ‘is emerging’.
2. The correction is ‘If you made your bed in the morning,
18. The correction is ‘The captain, along with the members of your room would look better when you get back in the
his team has protested against the changes effected by afternoon’. The verb in the if-clause (if you made your bed
the board in the selection process’. When two subjects in the morning) is in the past tense i.e. ‘made’. Hence, the
are combined using the connective ‘along with’, the verb verb in the main clause (your room would look better) should
agrees with the first subject. In the context, the first subject take ‘would’. The sentence can also be corrected like this ‘If
is singular. Hence the correction. you had made your bed in the morning, your room would
Triumphant Institute of Management Education Pvt. Ltd. (T.I.M.E.) HO: 95B, 2nd Floor, Siddamsetty Complex, Secunderabad – 500 003.
Tel : 040–27898195 Fax : 040–27847334 email : info@time4education.com website : www.time4education.com SM1001941/176
have looked better when you got back in the afternoon’. 4. The sentence should read ‘I don’t know where Jane is. She
The explanation is that when the verb in the main clause is could be at home’. This is because we are talking of a
‘would have’, the verb in the if-clause will be ‘had made’. probability, not a certainity.
The distinction between the two is that the former looks at
the past as a lesson for the future, whereas the latter only 5. The sentence should read ‘Jeniffer missed her class
looks at are unfulfilled past. Choice (C) today. She might have had some work’. This is because
we are talking of a possibility. ‘Should have’ is used when
3. The correction is ‘If she ate fewer sweets she would lose the reference is to a past obligation that is unfulfilled.
weight’. The given sentence is a conditional. When the verb
in the main clause (she would lose weight) is ‘would’ then 6. The sentence should read ‘I would have gone on a trip to
the verb in the if-clause takes the past tense. The if-clause America if I had had enough money’. The part of the
never takes ‘would’. Also see 16 above. Choice (A) sentence ‘If I had had enough money’ suggests that ‘would
have gone’ is the appropriate option. In the given sentence
4. The correction is ‘I wish I had invested my money in the
the first ‘had’ is a helping verb (Past participle of ‘have’) and
stock market’. In the given context we are talking of a
the second ‘had’ is the main verb which indicates
hypothetical situation. Hence ‘had invested’.
possession.
Choice (B)
5. The correction is ‘How I wish there were more useful 7. The sentence should read ‘If I had a bicycle, I would ride
books in our college library’. The sentence expresses a it everyday’. The ‘had’ represents possession; ‘If’
wish and therefore the past form of the verb is used. Hence represents condition and therefore needs to be followed by
the correction. Choice (B) ‘would’.

6. The correction is ‘If her mother had let her she would have Explanatory Notes for questions 8 and 9:
stayed longer’. The explanation given for question 16 holds
good. Choice (A) 8. Ι. ‘Should’ is used to indicate a recommendation,
advice, or an obligation. It is also used to indicate
7. The correction is ‘If you finish your work, we can attend expectation. Hence, Joseph should study tonight,
the seminar in the conference hall’. The ‘if-clause’ indicates that it is an obligation, advice, recommendation
suggests a condition that should be fulfilled for something or an expectation.
else to take place. Hence, the verb in the ‘if-clause’ cannot
ΙΙ to IV:
be in the past tense. Also see 16 above. Choice (A)
‘Ought to' indicates a moral obligation, a piece of
8. The correction is ‘Mathur feels that if his father had advice, or a recommedation ‘Had better’ indicates
allowed him to go to school, his fate would have been that, if the given suggestion is not accepted, or
different’. ‘If …… had’ in the subordinate clause and ‘would acted on, consequences await.
have’ in the main clause go together. Hence the correction. ‘Supposed to’ indicates a moral obligation or a
Another way to correct the sentence is, ‘Mathur feels that if responsibility.
his father allowed him to go to school, his fate would be
different’. Choice (A) V. ‘Must’ indicates that something is very important.
When ‘must’ is used the person usually has little
9. The correction is ‘For all its intellectual power and choice and should do as told.
empirical success as a creator of wealth, Free Market
Economics rests on a fallacy, which economists have 9. Ι. ‘Could have’ indicates a past possibility or ability
quietly agreed among themselves to overlook’. The error (when used as the past form of ‘can’)
is one of subject-verb concord. The reference is to ‘Free ΙΙ . ‘Rajani should have gone to the post office.’ This
Market Economics’ which is the subject of the sentence and means that she did not go. She was supposed to go,
is in the singular. Hence the correction is rests on a fallacy. but she did not.
Choice (C) ΙΙΙ. ‘Rajani would have gone to the post office’
indicates that Rajani did not go to the post office (there
10. The correction is ‘The leader of the party accompanied by must have been some reason for this). It is because
his wife and children is staying in Delhi until after the some condition (not mentioned in the sentence) was
meeting’. The subject of the sentence is ‘the leader’ which not met.
is singular. Hence, the correction is “is staying in Delhi”.
When two subjects are combined using the connective IV. ‘Might have’ indicates that the speaker is not sure of
“accompanied by” the verb agrees with the first subject. the occurrence or non-occurrence of the event.
Choice (C)
V. “Ought to have” indicates a past unfulfilled moral
obligation.
PRACTICE EXERCISE – 4
Explanatory Notes for questions 10 to 14:
Explanatory Notes for questions 1 to 7:
10. The correction is ‘He is a spoilt child, and is used to
1. The sentence should read ‘Mary did not attend office
disobeying his elders.’ This is because ‘be-form + used
yesterday. She may have gone for a picnic’. We are
to’ takes the “–ing” form of the verb. It conveys the idea of
talking of a possibility, which is expressed using ‘may’.
‘accustomed to doing something’.
2. The sentence should read ‘She did not finish her work and
this angered the teacher. She should have done her 11. The correct sentence is ‘You had better hurry if you don’t
work’. The given sentence says that she did not finish her want to miss the train’. ‘Had better’ does not take the ‘to +
work, although she was expected to do the work. (Hence verb’ form. Although ‘had’ appears to be in the past, ‘had
‘should have’ is the most appropriate). ‘Should have’ is better’ expresses strong advice for the future. Hence, the
used to suggest past unfulfilled obligation or expectation. structure of the sentence should be.
Subject + had better + verb.
3. The sentence should read ‘Sheela was supposed to be Or
here at 8 o’clock. She must have forgotten about our Subject + had better + not + verb. (for negative sentences)
meeting’. Must + have + past participle (V3) [here forget is
(V1), forgot is (V2) and forgotten is (V3)] is used to mean a 12. The correction is ‘She had better not go alone to see him’.
conclusion that can be drawn based on an event in past. For explanation refer to question (2).

Triumphant Institute of Management Education Pvt. Ltd. (T.I.M.E.) HO: 95B, 2nd Floor, Siddamsetty Complex, Secunderabad – 500 003.
Tel : 040–27898195 Fax : 040–27847334 email : info@time4education.com website : www.time4education.com SM1001941/177
13. The correction is ‘She said that she would rather play golf 27. The correction is ‘I am thankful to you because you
than cricket’. ‘Would rather’ expresses preference. Although helped me (to) get this job’. (or) ‘I am thankful to you
it appears to be in the past, it expresses preference in the because you helped me in getting this job’. In the given
present and in the future. It does not take the ‘to + verb’ sentence, ‘help’ is used as a causative verb. In a causative
form. verb, the person does not perform the action directly. It takes
the plain verb form, or ‘help + in + v-ing’.
14. The correction is ‘Larry said that he liked the way I had
my beautician do my hair.’ ‘Had’ is a causative verb (i.e., 28. The correction is ‘While I was walking in the garden, I
someone else is the cause for the work to be done) in the heard the clock strike ten’. The given sentence conveys
context. In such cases it takes the present tense form. the idea that, while the clock was walking in the garden, it
struck ten, which is absurd. In the given sentence, we don’t
Explanatory Notes for questions 15 to 30: find any noun or pronoun to which the participle ‘walking’
refers. Hence the correction.
ΙΙ and ΙΙΙ:
29. The correction is ‘While I was going round the corner, I
Both ‘Has to’ and ‘must’ mean the same. Both are used to saw a horrible sight’ or ‘he / she / they (were). For
indicate the imperative. explanation refer to question (29).
15. The correction is ‘The petitioners wanted to see a 30. The correction is ‘The doctor suggested that I (should) use
responsible government official’. The verb ‘want’ does these medicines regularly’. This is because the verb
not take ‘that’ but it takes the ‘to + verb’ form. ‘suggest’ should be used with a ‘that........’ clause. It cannot
be used with the to-infinitive.
16. The correction is ‘His mother forbids his studying late into
the night’. (or) ‘His mother forbids him to study late into EXPLANATORY NOTES FOR ADDITIONAL QUESTIONS
the night’. Forbid takes ‘to’ (or) ‘the possessive form’ of
Explanatory Notes for questions 1 to 10:
the noun/pronoun + the ‘ing-form’ of the verb. (We forbid
someone to do something and not from doing something. It 1. The correction is ‘I was displeased at your coming late’.
does not take an ‘ing’ form of the verb). ‘Coming’ in the given sentence, is a gerund (i.e. a verb
acting as a noun). It should be preceded by the possessive
17. The correction is ‘She makes all the students do their case of the pronoun. Choice (D)
work’. ‘Make’ does not take the to + verb form. It takes the
plain form of the main verb. 2. The correction is ‘I think you should let your hair grow’.
‘Let’ does not take the to + verb form. Choice (D)
18. The correction is ‘The management wants every
employee to be sincere and hardworking’. The verb 3. The correction is ‘As she does not want to make the issue
‘want’ should not be used with a ‘that clause’. It should be public, she avoids meeting people’. The error is due to the
used with the to-infinitive (to + verb). wrong usage of the verb. The verb ‘avoid’ is incorrectly used
with the to-infinitive. It should be used with the gerund
19. The correction is ‘Someone should help her because she (a verbal-noun) i.e., the ing-form. Choice (D)
does not know how to perform the task’. ‘Know’ when
followed by an infinitive (to + verb) it expresses ability. 4. The correction is ‘I hope this book helps you (to)
(Hence, the structure should be subject + know how + to + understand the concept better’.
verb) In the given context we are talking about the ability to The explanation given for question (28) holds good.
do something. Choice (C)

20. The correction is ‘His speech was difficult to follow’. Here 5. The correction is ‘I like the way Ramesh makes his son do
‘follow’ means to understand, ‘to be followed’ is incorrect, his homework neatly’. ‘Make’ does not take the to + verb
because ‘difficult’ takes (to + verb) from. It does not take ‘to form. Choice (D)
be’ because the ‘be – verb’ does not follow ‘to’ after the
6. The correction is ‘You had better meet the principal if you
adjective ‘difficult’. (Hence, the structure should be difficult
want to be sure of securing admission to this college’.
+ to + verb).
Although ‘had’ appears to be a past tense word, ‘had better’
21. The correction is ‘He stopped smoking because he is expresses advice for the future. It should not be followed by
afraid that it may affect his health’. ‘Stop’ does not take a a to-infinitive or a past form of the verb. Choice (B)
‘to + verb’ form after it. It takes the ‘ing’ form of the verb. AS 7. The correction is ‘Although these boys are twins they
is the sentence means that the halted somewhere to smoke, don’t tend to think alike’. ‘Tend’ takes the ‘to + verb’ form.
but the following reason would be incongruous. It does not take the ‘ing’ form of the verb. Choice (D)
22. The correction is ‘All of us at the office have been looking 8. The correction is ‘It is not necessary that you take an
forward to meeting our new colleague’. The verb phrase entrance examination to secure admission to a business
‘look forward to’ takes the ‘ing-form’ of the verb. school’. ’Must’ is redundant. Hence the correction.
Choice (B)
23. The correction is ‘The counsellor asked me if I would mind
waiting’. The phrases ‘do not mind’ and ‘would mind’ take 9. The correction is ‘In today’s competitive world, even small
the ‘ing’ form of the verb. businesses had better advertise on T.V. in order to be
able to compete effectively’. For explanation refer to
24. The correction is ‘Most interviewers insist on recruiting question (2). Choice (C)
people who make a favourable impression’. The verb
10. It should be than practising.
phrase ‘insist on’ takes an ‘ing-form’ after it.
(Refer to V02; Section–II; Unit-II; Rule 1 (a))
Choice (D)
25. The correction is ‘Her parents tried to dissuade her from
marrying an old man’. ‘Dissuade’ takes the preposition PRACTICE EXERCISE – 5
‘from’ and the gerundial (ing – form which acts as a verbal
noun) form of the verb. 1. The errors are “often bear highest costs of ------". The
adjective form of ‘environment’ is ‘environmental’. The
26. The given sentence says that most children enjoy doing second error is that – an adjective, when used in superlative
something. ‘Enjoy’ does not take the to + verb form. It takes degree should be preceded by the definite article ‘the’.
the ‘ing form’ of the verb. Hence, the correction is ‘Most Hence, the sentence should read. “It is the poor and
children enjoy playing board games and participating in vulnerable communities that often bear the highest costs of
sports’. environmental degradation”.
Triumphant Institute of Management Education Pvt. Ltd. (T.I.M.E.) HO: 95B, 2nd Floor, Siddamsetty Complex, Secunderabad – 500 003.
Tel : 040–27898195 Fax : 040–27847334 email : info@time4education.com website : www.time4education.com SM1001941/178
2. The correction is “the safest form --------”. Two comparatives, 13. The correction is “only a few brave souls or ‘few brave souls’
or two superlatives don’t go together. --------. “Few brave souls” means that hardly any brave
Also, the word ‘airlines’ is used here to refer, to the type of people exist and would not need ‘only’. When the adverb
passengers and should therefore be ‘airline’. ‘only’ is used, the idea to be expressed is that some do exist
Hence the sentence should read “Though flying is the safest i.e. ‘a few brave souls --------." Hence, the sentence should
form of travel it terrifies 40% of airline passengers”. read. “There are only a few brave souls in the world who dare
to conceive the truth and follow it to the end” or “There are
3. The correction is “will be felt sooner than -------”. The few brave souls…..."
comparative degree of ‘soon’ is ‘sooner’ not ‘more soon’.
Hence the sentence should read “The effects of climate 14. The question sentence has two errors.
change will be felt sooner than scientists expect and the 1. The usage ‘more high’ is incorrect. The comparative
world must learn to live with the effects”. degree of ‘high’ is ‘higher’. Hence, the correction is
‘higher at present -----'.
4. The correction is “no one listened to it more keenly”. The 2. The second error is ‘than in the poor ones’. It has to
question sentence conveys the idea that the speaker’s be ‘than in the poorer ones’ since a comparative is
grandfather listened to jazz more loudly; which is absurd required in that context.
because listening cannot be done loudly. The music can be
played loudly. Music can be listened to keenly. Hence the 15. The correction is “...... he believes are less than his brother’s
sentence should read “I used to listen to jazz, but no one (savings)”. The sentence has two errors. One is the usage
listened to it more keenly than my grandfather”. ‘lower than’. The correction is “less than”. The second
correction is “his brother’s”. The comparison is between ‘his
5. The correction is “the more hard working” of the two. When savings’ and his brother’s savings’. Hence the sentence
only two things or two people are being compared and if the should read. “Len wants to increase his savings, which he
sentence consists of the phrase (of the two) the structure believes are less than his brother’s”.
would be the + Adjective in the comparative degree. Hence
the sentence should read “Though both the brothers are 16. The error is the use of the word ‘expedite’, which is a verb. It
successful businessmen the younger one is the more should be ‘expeditious’. Also, the second correction would
hardworking of the two”. be “is ‘the’ easier and more expeditious of the two solutions".
‘Easier more expeditious’ is regarded as a single adjective
6. There are two errors:
phrase describing the solution and hence ‘the’ should
‘Valiant’ can be used either before ‘efforts’ or before ‘British’,
precede the phrase. The sentence should read “Allocations
but cannot be placed between ‘British’ and ‘explorers’.
of funds to train intelligent people who would be capable of
The second error is “more clear”. It has to be ‘clearer’ as the
becoming leaders is the easier and more expeditious of the
comparative form of ‘clear’ is ‘clearer’ and not ‘more clear’.
two solutions."
The comparative ‘clearer’’ is appropriate since they want to
make the picture ‘clearer’ than what it was before.
17. The correction is “among the perfectly executed --“.
7. The correction is ‘debilitating ailment’. ‘Debilitated is a verb Adjectives like ‘unique’ ‘perfect’ do not have the comparative
hence it is not appropriate in the context. ‘Ailment’ is the or superlative forms. Hence, the sentence should read '“The
noun which should be qualified and hence ‘debilitating’ Milkmaid” is among the perfectly executed works of Ravi
would be the appropriate qualifying word. Hence the Varma.
sentence should read “Depression is a debilitating ailment
which can destroy the quality of our lives and the lives of 18. The correction would be ‘opposing forces’ as ‘opposing;
people close to us”. ideas, forces or tendencies are totally different from each
other. ‘Opposed’ would be the past participle and ‘opposing’, the
8. The correction is “as contemptuous as we are -----". “As present participle. Since the forces are always different from
much contemptuous as -----" is an inappropriate usage. each other, opposing should be used.
Hence the correction. The sentence should read “I have not
come across any other society as contemptuous as we are 19. The correction is ‘little surprise’ and not ‘a little surprise’.
of countries that have done well "Better” is inappropriate as ‘Little surprise’ in this context would mean that it is not
no comparison is being made with that word. surprising at all that the festival has became dear to Tamil
Nadu because of reasons mentioned in the sentence.
9. The correction is ‘democratic temperament’. ‘Democratic’ is
the adjectival form which conveys the idea that the author’s 20. The sentence has two errors. One is the usage ‘less side
attitude reveals an inclination or predilection for the effects’, since the reference is to ‘side effects’ which are
principles or ideas of democracy. Hence the sentence countable, the usage ‘less’ is incorrect. The correction is
should read “The author’s attitude reveals a democratic “fewer side effects”. When the reference is to number we use
temperament worthy of admiration at a time when intolerance ‘few’ and when it is to quantity we use ‘less’.
has become the dominant philosophy". The second error is the usage “preferred than”. ‘Prefer’ is
followed by ‘to’ not ‘than’ Hence, the correction is “preferred
10. The correction is “little known poet”. The idea conveyed to chemical drugs by many”. Hence, the sentence should
would be a poet little known i.e. not known. ‘Less known’ is read. “Traditional Chinese medicine comes from natural
incorrect because ‘less’ is the comparative degree of ‘little’, herbal sources and is believed to have fewer side effects and
which is not appropriate in the context. is preferred to chemical drugs by many."

11. The correction is “Man is unique among -------“. ‘Unique’ is 21. The correction is “inferior to that of the westerners”. The
an adjective which means distinct or different from others. It adjective ‘inferior’ takes ‘to’ not ‘than’.
does not take ‘more’ or ‘most’ before it i.e. it does not exist The sentence should read. “It is time the world understood
in comparative or superlative degrees. Hence the sentence that our intelligence is by no means inferior to that of the
should read “Man is unique among all beings because he westerners”. The comparison is between the intelligence of
has a personality beyond the body, mind and intellect.” Indians and that of the westerners.

12. The correction is “an imposing ten foot-tall black stone idol”. The 22. ‘Economical’ has to be replaced with ‘economic’.
phrase ten-foot-tall is an adjective qualifying stone. Hence, it ‘Economical’ – providing good service or value in relation to
does not take the plural form i.e. when a noun is preceded by a the amount of time or money spent; low-cost, inexpensive.
compound word, the latter becomes an adjective and would be ‘Economic’ is connected with trade, industry and
singular. Hence the sentence should read “At the entrance to the development of wealth of a country an area or a society.
Ashram, we find an imposing ten-foot-tall black' stone idol of Hence, ‘economic’ is the appropriate adjective in the given
Lord Krishna on a pedestal'. context.
Triumphant Institute of Management Education Pvt. Ltd. (T.I.M.E.) HO: 95B, 2nd Floor, Siddamsetty Complex, Secunderabad – 500 003.
Tel : 040–27898195 Fax : 040–27847334 email : info@time4education.com website : www.time4education.com SM1001941/179
23. The adjective ‘fine’ should be replaced with ‘finest’ as the 7. The correction is ‘richer . . . than we realize’. The adjective
context demands an adjective in the superlative and not in should be in the comparative degree in the above context.’
the positive degree. Also, ‘the countries’, leads to the Choice (C)
superlative.
8. The sentence has the structure 'not so much this as that';
24. Since two things are being compared, the comparative hence 'as' must precede 'that "....... with this criticism as that
‘more’ has to come in. Hence, ‘opine that it is more they .......'. Choice (B)
difficult….’
9. The sentence takes the superlative 'the greatest' and not the
25. The sentence should have 'latter' (the second of the two comparative 'greater'. The sentence should read '....... it is of
options) and not 'later' (the comparative form of late) – The the greatest importance to ........'. Choice (A)
sentence should read '...... was prepared for the latter when
........' 10. It should be 'most travellers' not 'more travellers'.
Choice (B)
26. The sentence should have the comparative and not the
superlative degree since only two things are compared. PRACTICE EXERCISE – 6
Hence, "..... it is more important ...... (not most)."
1. Here, ‘rapidly’ is the adverb of degree and it should be placed
27. ‘Prejudices’ should be preceded by an adjective – ‘religious’ before the adjective i.e., changing. An adverb of degree
and not religion, which is a noun. modifies an adjective or another adverb and it is placed
before the adjective or adverb. Here, the correction is . . .
28. The word 'most' should be deleted. It should be '...... one of rapidly changing moods’.
the few sources of water'. The comparative and superlative
for few are 'fewer' and 'fewest'. 2. The error lies in the positioning of the adverb. When we have
verb + preposition + object, the adverb can be either before
29. The correction is ‘a little quiet thinking’ and not ‘little quiet the preposition or after the object. But if the object contains
thinking’. ‘Little quiet thinking’ would mean hardly any quiet a number of words we put the adverb before the preposition.
thinking, which is not what the sentence is intended to Hence, here, the correction is ‘He looked suspiciously at
convey. ‘A little quiet thinking’ would mean ‘some quiet the . . .’
thinking’ and hence is appropriate is the given context.
3. Further is used to denote place or position rather than
distance. When used figuratively ‘further’ denotes distance,
30. The right adjective is ‘impressive’. ‘Impressionable’ means
as in the given context. Farther is used to denote distance.
‘easily influenced or affected by something or somebody’.
Hence, there is no correction required.
Something that is ‘impressive’ impresses you, for example
because it is great in size or degree, or is done with a great 4. The error lies in the positioning of the adverb. The adverb
deal of skill. should be placed after the verb. Hence the correction is ‘. . .
is universally taken . . .’. Or ‘universally, the increase in levels
EXPLANATORY NOTES FOR ADDITIONAL QUESTIONS . . . .’
1. The correction is “No other architecture seems ------” The 5. Here, the adverb is used to modify the adjective ‘unique’.
question sentence suggests that the ‘mighty citadels’ are Because according to the sentence honey is unique in terms
being compared with themselves. Hence the sentence of nutrition. Hence the correction here is ‘……nutritionally
should read “No other architecture seems to express so unique’.
forcefully the spiritual longings of humanity as these mighty
citadels of faith towering over town and country.” 6. The adverb ‘scarcely’ is used erroneously here. ‘Scarcely
Choice (A) can mean almost not and is used as ‘Scarcely ever’,
‘Scarcely any’ or ‘Scarcely . . . when’. The correct adverb
2. The correction is “the only source of energy or the major which should be used here is ‘rarely which denotes the
source of energy” as an adjective should precede ‘source of adverb of frequency.
energy’, without which the sentence would be incomplete.
7. The error lies in the usage of ‘before’ instead of ‘ago’. ‘Ago’
Choice (B)
denotes a period of time from the present, dating backwards
whereas ‘before’ as an adverb means earlier or in front of.
3. The correction is “with the historical background”. Historic
Here, the reference is to billions of years backwards. Hence
means that which is important in history, or likely to be
the correct adverb here is ‘ago’.
considered important at some time in the future. Historical would
mean connected with the past. Hence, ‘historical’ is the suitable 8. The adverb of quantity ‘such’ is used here erroneously. Here,
adjective in the above context. Choice (A) the reference is not to quantity. The correct adverb to be
used here is ‘so’.
4. The correction is “these changes are faster than at any other
time after birth”. 9. The sentence is erroneous because of the word ‘highly’.
The comparison is between the changes that occur during ‘Aims highly is an incorrect expression. The correction is
infancy and those that occur at any other time after birth, aims high.
hence the correction. The sentence should read “The
changes in the overall size of a child’s body are the most 10. The adverb ‘visibly’ should precede the adjective ‘upset’
obvious signs of physical growth and during infancy, these which it modifies. Hence ‘. . . visibly upset’.
changes are faster than at any other time after birth."
Choice (D) 11. The adverb ‘very’ should not be used before a comparative
adjective (higher). The correction is ‘. . . much higher’.
5. The words '........ made engineering studies of the famous
two bridges ......' is incorrect. It should be '..... studied the 12. The sentence is erroneous due to the usage of the wrong
engineering of the two famous bridges ......' adverb. The correction is ‘far and wide’.
Choice (B)
13. When the reference is to a higher degree than is natural,
proper or expected ‘too’ should be used and not ‘very’.
6. The sentence combines the comparative (bigger) and the
superlative (most imminent). The correction should 14. The adverb of frequency is always placed between the
be '........ that its biggest and most imminent military threat subject and the verb. Hence the correction is, ‘He and his
.......' Choice (C) wife seldom see eye to eye’.
Triumphant Institute of Management Education Pvt. Ltd. (T.I.M.E.) HO: 95B, 2nd Floor, Siddamsetty Complex, Secunderabad – 500 003.
Tel : 040–27898195 Fax : 040–27847334 email : info@time4education.com website : www.time4education.com SM1001941/180
15. The sentence is erroneous because of the word ‘quiet’ which 32. You value something ‘highly’ and not ‘deeply’. Hence the
is actually an adjective. The correction is ‘quite’. adverb ‘highly’ should replace ‘deeply’ in order to make the
sentence grammatically correct.
16. The correction here is ‘twenty years ago’. The reference here
is to a period of time from the present dating backwards. 33. The adverb ‘sometimes’ modifies the adjective awful and
Hence the appropriate adverb is ‘ago’. The sentence should hence it should precede the adjective. Hence the correction
read “I visited Delhi twenty years ago”. is “. . . sometimes awful consequences’.
17. If a past participle is used in the purely adjectival sense, it is
34. The adverb ‘far and away’ which means beyond comparison
modified by ‘very’ and not by ‘much’. Hence the correction is
is misplaced. It should be placed after is’.
. . .’very learned man.’
18. The adverb ‘fairly’ is used only with favourable adjectives 35. The error lies in the usage of the adjective instead of the
and not with unfavourable ones. The appropriate adverb adverb. The adverb ‘dearly’ should replace the adjective
here is rather. ‘dear’ in order to indicate how a mere episodic response will
affect the country.
19. ‘Scarcely’ should be followed by ‘had’ or did. The correction
here is ‘Scarcely had the train arrived on the platform when EXPLANATORY NOTES FOR ADDITIONAL QUESTIONS
the people started rushing towards it’ or scarcely did the train
arrive when the people start rushing towards it. 36. The error is in the adverbial phrase – the correct phrase is ‘.
. . market democracy will reign supreme . . .’. Here, the word
20. The adverb ‘enough’ should always follow the adjective ‘supreme’ qualifies market democracy.
which it modifies. The correction here is ‘. . . smart enough’.
21. The error lies in the positioning of the second adverb 37. The error lies in the use of the incorrect adverb. Here ‘barely’
‘extremely’. Here the correction is ‘. . . extremely penitent is used instead of ‘rarely’. Barely is used for degree whereas
about it’. here, the reference is to frequency hence ‘rarely’ (i.e., not
often) should be used in order to make sense.
22. The sentence implies that stress has a negative effect on
productivity. Hence the adverb ‘negatively’ should follow the 38. The error is in the adverbial phrase. The correction is ‘….
word which it modifies . . . ‘Stress affects productivity understandably so . . .’.
negatively . . . ’
39. The error lies in the positioning of the adverb. ‘Appreciably’
23. The adverb ‘since’ is used erroneously. Since as an adverb which is an adverb of manner should always be placed after
refers to a time in the past until a later past time, or until now. the verb. The correction is ‘. . . raised appreciably’.
The correct adverb which should be used here is, ago. Ago,
which denotes a period of time from the present dating 40. Here, the adverb ‘farther’ which denotes distance is
backwards, is appropriate here. inappropriate. The adverb further, which is used to refer to
development, progress etc., would be appropriate.
24. Although both ‘fairly’ and ‘rather’ mean moderately, ‘fairly’ is
used with favourable adjectives and ‘rather’ is used with 41. ‘Fairly’ is used to express approval, while ‘rather’ is used to
unfavourable ones. Here, the reference is to the heat of the express disapproval. Here, the reference is to an
tropical plains and contains the idea of being unfavourable. unfavourable adjective. Hence, ‘rather’ is appropriate here
Hence the appropriate adjective would be ‘rather’ and not . . .’ these rather esoteric phenomena’.
‘fairly’.
25. The adverb ‘much’ is used erroneously. The correction is 42. The adverb ‘much’ is used erroneously. Much qualifies
‘very’. ‘Very’ qualifies adjectives in the positive degree and adjectives in the comparitive degree and not in the positive
much qualifies them in the comparative degree (much more degree. Hence the adverb ‘very’ would be appropriate here.
essential).
43. The error lies in the placement of the adverb, mostly. It is
26. The sentence is erroneous due to the usage of the wrong correct to say ‘mostly mountainous country’.
adverb ‘fairly’. ‘Fairly’ is used to denote moderately and
hence does not fit into the context. The sentence implies that 44. The sentence presents a contrast between what scientists
no more than a few years before the birth of Christ the earth now believe and what they believed earlier. Hence the
was battered by an earthquake, hence the appropriate adverb once in the last part of the sentence should be placed
adverb which should be used here is ‘barely’. Barely is used before ‘thought’.
to denote ‘not more than’.
45. The sentence is erroneous because of the usage of the
27. The error lies in the positioning of the adverb. The adverb modifier ‘more’. ‘So’ is more appropriate.
‘historically’ can either begin the sentence –‘Historically,
Britain has been . . .’ –or it can be ‘Britian has historically 46. The adverb ‘much’ is used erroneously. When the reference
been a . . .’ in order to mean from the point of view of history. is to a higher degree than is desirable, natural, proper or
expected, the adverb ‘too’ should be used and not much ‘. .
28. The adverb should follow the adjective. The rule is adjective . too rapidly . . .’
+ enough + infinitive. Hence the correction is ‘. . . well enough
to . . .’ 47. The sentence is erroneous because of the word ‘justly’. The
word ‘justly’ which means ‘in a morally fair manner’ does not
29. The adverb ‘enough’ which means sufficient is always make sense in this context. The correction is
placed before the adjective or adverb. Hence the correction “. . . supplied key insights just at the right moments”.
is . . . ‘fast enough to keep pace . . .’
48. The sentence is erroneous because of the word ‘presently’
30. The word ‘increasing’ should be replaced by the adverb of which refers to the time of speaking or writing. Here the
degree i.e., increasingly, because it answers the question reference is to contemporary secular societies hence the
how seriously is it treated? appropriate adverb to be used here is ‘today’s secular
societies’.
31. When there are two or more adverbs after a verb (and its
object), the normal order is adverb of manner, adverb of 49. The adverb ‘enough’ is always placed before the noun.
place and adverb of time. Hence in this sentence the correction Hence the correction is enough ‘room’.
is ‘The minister categorically denied his involvement in the
scam at the meeting with media persons last night’. 50. The correction is ‘at present’ and not ‘presently’.
Triumphant Institute of Management Education Pvt. Ltd. (T.I.M.E.) HO: 95B, 2nd Floor, Siddamsetty Complex, Secunderabad – 500 003.
Tel : 040–27898195 Fax : 040–27847334 email : info@time4education.com website : www.time4education.com SM1001941/181
PRACTICE EXERCISE – 7 19. The given sentence denotes a continuation of the protest till
a certain time. Hence the correct conjunction is until and not
1. When two clauses presenting similar or connected ideas are unless which denotes a condition.
put together, the correct conjunction to be used is ‘and’ not
‘but’. In this sentence losing weight and preaching about the 20. The correct compound conjunction to be used in this context
virtues of diet and exercise are related ideas. Secondly, diet is ‘so that’ and not ‘such that’.
and exercise help in losing weight not diet or exercise.
21. ‘Idea of’ death and resurrection would be apt since the ideas
2. The correct conjunction to be used in this context is yet. are presented in the sentence. Idea about something is an
When ‘yet’ is used in a sentence it conveys the meaning opinion or belief about something, which is not suitable in the
in spite of. ‘Yet’ like ‘but’ ‘and’ ‘or’ is a coordinating given context. Choice (C)
conjunction which joins two main clauses. Yet, deprived of
power himself . . . . . 22. ‘Commented upon’ would be apt in the given context. You
comment ‘on’ or ‘upon’ something. Also, you have a
3. As to help is erroneous. For helping me is appropriate to join comment to make ‘about’ something. Commented ‘by’ in the
the clauses. ‘For’ in this context means in order to. above context makes the sentence incomplete. ‘At’ and
‘against are not used with comment. Choice (C)
4. The correct subordinating conjunction to be used in this
context is ‘whether’ and not ‘if’ which denotes a condition 23. ‘Viewed in’ the perspective of another culture is apt in the
rather than a choice. above context. Viewed beside, beyond and despite do not
suit the context. If viewed outside the prespective they will
5. This sentence should begin with the subordinating not disclose any unexpected meanings. Choice (D)
conjunction ‘if’ to indicate the two contrasting situations
introduced by ‘things were no better . . .. 24. ‘Since’ is the right preposition as the chasm which started at
that time is continuing to widen. ‘has been’ points to the right
6. The error is in the use of the subordinating conjunction choice. Choice (C)
‘though’ with ‘even’, which implies that the valley is beautiful
in spite of the fact that it is out of bounds. Hence the use of 25. ‘Making regular small savings’ is the behaviour ‘of’ the
‘yet’ in place of ‘even though’ conveys the right meaning that customer. The blank cannot accommodate any of the other
in spite of being a beautiful spot, it is out of its locational prepositions. Choice (B)
disadvantage.
26. ‘Stand on’ something is to have a particular attitude or
7. When we wish to combine two ideas the compound opinion about something or towards somebody.
conjunction ‘as well as’ or the conjunction ‘and’ should be Choice (C)
used, and not the preposition ‘with’.
27. ‘Most of the activity’ suggests that ‘concentrated in’ (a
8. Some conjunctions are used in pairs and are called
particular place) is the right option. ‘Concentrate on’ would
correlative conjunctions. ‘Both’ should be used with ‘and’ not
mean to give all your attention to something. Along would
‘or’. Hence the sentence should read ‘sources say….. both
mean by the side of, on the edge of something and hence
curtailing productions to increase shortages and prices and
would not be apt in the above context. Concentrate is not
that a cartel is at work…..”
used with over and with. Choice (C)
9. The correlative conjunction ‘not only’….. should be followed by
‘but also’. Hence the sentence should read not only the 28. Since you sell something ‘to’ someone, ‘to’ would be the right
individuals who have delivered them but also the people……. preposition in the above context. Choice (D)

10. The ‘wherever they are’ should be followed by here, there, OR 29. ‘The most’ authoritative points to the answer. ‘Of’ all the
somewhere else. Hence at home, in hotels, or on the road. tomes, ‘History…’was the ’most’ is apt. The rest of the
prepositions will not bring out the intended meaning.
11. ‘Not only…..but also’ is the correlative conjunction and Choice (C)
should be used together. Hence ‘setting out not only
technological goals but also . . . . is the correction 30. No preposition is required after tackle. Choice (D)

31. To ‘restore something to’ its former condition, is to bring it


12. ‘Whether…or’ is another correlative conjunction used with
back to the way it used to be. Choice (A)
two conditional statements. Hence the sentence should
read….whether you are confronting hurricanes in the east or 32. ‘Reflect on’ would be the right combination. It means ‘to think
fires and earthquakes in the west. carefully and deeply about something’. Choice (C)
13. ‘Even if’ is the correct compound conjunction to be used to 33. Oscillation is a regular movement between one position and
indicate the meaning of ‘in spite of’. ‘Even though’ is erroneous. another. Hence ‘between’ would be the right preposition.
Choice (D)
14. ‘Neither….. nor’ is a compound conjunction that we use to
indicate two negative ideas. 34. Something is directed ‘towards’ the purpose of achieving a
‘Neither followed by ‘or’ is erroneous. It should be used with desired result. Choice (C)
nor.
35. If something is ‘named after’ somebody or something, it is
15. The error is in the positioning of the compound conjunction given that name. Hence, ‘after’ is the right preposition in the
‘either…..or’. The ‘either’ should be placed after the verb when given context. Choice (D)
the verb is applicable to both clauses. Hence the correction
is….. ‘they die either in the womb or just after birth’. 36. The words ‘new’, ‘extending’ and ‘traditional’ indicate that the
right preposition is ‘beyond’. Choice (B)
16. One should know one’s capacity for loss before one starts
trading not after. 37. To ban somebody or something ‘from’ doing something is to
forbid somebody to do something, go somewhere etc.,
17. The conjunction to be used when we make comparisions is especially officially. Hence, ‘from’ is the right preposition in
‘than’ and not ‘then’. the above context. Choice (C)

18. The conjunction ‘until’ refers to a time frame. The given 38. The essence of the sentence is that the trends that are
sentence however, indicates a condition. Hence ‘unless’ mentioned are spreading quickly ‘across’ Asia and also other
should be used in place of ‘until’. parts. Hence ‘across’ would be apt. Choice (C)
Triumphant Institute of Management Education Pvt. Ltd. (T.I.M.E.) HO: 95B, 2nd Floor, Siddamsetty Complex, Secunderabad – 500 003.
Tel : 040–27898195 Fax : 040–27847334 email : info@time4education.com website : www.time4education.com SM1001941/182
39. ‘Poised for’ would be apt. ‘Poised for’ would mean set PRACTICE EXERCISE – 8
(poised to do something and poised for something). Poised
‘on’ and over are also used, but will not suit the context. 1. The blank needs the words suggesting a ‘method’ and only
Choice (A) option 2 does it. Option (A) would mean the description of
the scenario is further detailed of which ‘the resurgent India’
40. ‘Insight into’ something would mean an understanding of is a part of it. Option (B) means a method or way of doing
what something is like. Choice (C) something. Option (C) means to say what you’ve noticed or
understood. Option (D) is inapt as it needs to be across the
EXPLANATORY NOTES FOR ADDITIONAL QUESTIONS
country and cannot be followed by the word ‘which’.
41. The correct conjunction to use in this context is the Choice (B)
concessional ‘although’ rather than the conditional ‘if’.
2. Option (A) means to say what you’ve noticed or understood.
42. The context suggests that a decade will pass before the Option (B) means to put something with something else and
positive change is likely, and not ‘in’ a decade. Hence the this is apt in the context. Option (C) means until a future point
sentence should read “after a decade……” in time. Option (D) refers to the present time.
Choice (B)
43. To explain a reason why something happens, we use the
subordinating conjunction ‘as’. ‘While’ denotes an ongoing 3. Option (A) means up to this point. Option (B) is used in
action and cannot be used in this context. So the sentence comparison to refer to the degree of something and fits the
should read Smaller towns have been quietly depopulated context. Option (C) would mean as the situation may be and
as their educated population . . . . does not logically follow in the context. Option (D) is used to
give permission. Choice (B)
44. The sentence suggests that the conditional conjunction ‘if’
should be used and not ‘when’ which is a subordinating 4. Option (A) means a method or way of doing something.
conjunction denoting time. Option (B) describes a plausible situation. Option (C) means
to say that you have noticed or understood. Option (D) means
45. The subordinating conjunction ‘though’ is appropriate in this in any situation or condition or whichever way it is analysed
context in place of ‘in as much as’ which is used to cite a and is apt in the context. Choice (D)
reason rather than concession.
5. Option (A) means a correct route. Option (B) is likely to
46. The correction requires the use of the conjunction ‘and’ in achieve. Option (D) is correct because the preposition ‘of’
place of ‘for’ to join the two ideas of outer space being a following ‘in’ would mean ‘during’. Option (C) is used to show
unifier and the last refuge. that what you are saying is generally known or accepted.
Choice (D)
47. To express a reason or a cause and as an equivalent to
‘because’ or ‘for’ we must use the conjunction that.
6. The blank has to be filled with words that mean because of
Hence the sentence should read “we are in such a rarefied
something. Option (A) states an information vital to the topic
atmosphere that…..”
being discussed. Option (B) means because of something.
48. The sentence should read “It is for this reason that I asked Option (C) means in the course of time. Option (D) is used
him to leave both the offices of President and Chief of Army to make comparisons. Choice (B)
Staff and to pave the way for the composition of an interim
government”. This is because ‘both’ should be used with 7. Options (A) and (B) means in connection with. Option (C)
‘and’ not ‘or’. means not affected by something and is right in the context.
Option (D) means in this particular way. Choice (C)
49. The present perfect continuous tense of the verb ‘sell’
indicates the use of since rather than ‘after’ as the 8. Options (A) to (C) are incorrect grammatical use of
subordinating conjunction. prepositions with the word ‘mistake’. Option (D) fits aptly.
‘Since’ when used as a conjunction in this sentence should be Choice (D)
preceded by the verb in the present perfect tense, (have been
selling) followed by a verb in the simple past tense (released). 9. Option (A) means considering the present situation; as
things are. Option (B) means in the same way as someone
50. Soviet cinema collapsed ‘when’ state funding disappeared at or something and is right as it links Maria and other working
the close of the communist period is the correction. women. Option (C) means something done to help
We use the conjunction ‘when’ to indicate something everyone. Option (D) is used to say what you have noticed
happens to interrupt something. or understood. The link between ‘working women’ and
‘Marie’ is not suitably established as a result.
51. The correction is ‘convinced of’ the report’s veracity. Choice (B)

52. The correction is – ‘approve of.’ The preposition is missing 10. Option (A) means ‘as stated by’ which fits the idea in the
in the original sentence. blank. Option (B) means the consequence of what is stated
earlier. Option (C) means with the passage of time. Option
53. The correction is ‘imposed on or upon’. (D) is used to compare. Choice (A)
54. The correction is ‘dissuade his/her students from’ and should
Explanatory notes for questions 11 to 35:
be followed by the ‘ing’ from of the verb. Hence from reading
.....
A Phrasal Verb provides a meaning (as a whole) which may
55. There is no correction. be different from the meaning of the words combined in it.

56. The correction is ‘pleased with’. 11. The blank in the given statement must be filled with words
that suggest to pass the baby to someone who will, from then
57. The preposition ‘against’ should be removed from the on take responsibility of the child. Option (A) suggests an
sentence since it is redundant. idea of passing on of knowledge, a possession or a skill to
one belonging to a younger generation. Option (B) means
58. The correction is ‘agreeable to him’. the person handed over responsibility and control to the one
59. There is no correction. who is being given and is opt. Option (C) means to dole out
or give out something. Option (D) means something that is
60. The correction is ‘fault with’. given to a person in authority. Choice (B)

Triumphant Institute of Management Education Pvt. Ltd. (T.I.M.E.) HO: 95B, 2nd Floor, Siddamsetty Complex, Secunderabad – 500 003.
Tel : 040–27898195 Fax : 040–27847334 email : info@time4education.com website : www.time4education.com SM1001941/183
12. The blank must be filled with words that suggest that one a form by writing information on it. This fits the blank. Option
wants to escape from an unpleasant situation. Option (A) (C) means to become larger, rounder or fatter. Option (D)
means one achieves success even when there are means to make something completely full. Choice (B)
difficulties and obstacles. Option (B) means to change or
escape from a situation and fits the context. Option (C) – 23. The blank has to be filled with words which means to be
‘break in’ – means to enter by force or to interrupt someone present in every part of something. This is the meaning
by some sudden remark. Option (D) – ‘break off’ – means to of option (C). Option (A) means that which often occurs.
remove something by force or to stop doing something Option (B) means trying to avoid being captured. Option (D)
suddenly. Choice (B) means to run away. Choice (C)
13. The blank ought to be filled with words that means to leave 24. Option (A) means to rise. Option (B) means to be successful
with the speaker. ‘Come across’ means to find by chance. in your career. Option (C) can be followed by preposition –
So Option (A) is ruled out. option (B) when used intransitively on, in, or with something and it means to manage to live or
means to leave with the speaker and is appropriate here. do a particular thing using the money, knowledge,
Option (C) means to succeed or to happen as arranged. equipment, etc. that you have and so it fits the context.
Option (D) means to enter when used in the transitive sense. Option (D) means to give serious attention to something.
Choice (B) Choice (C)
14. Option (A) means to fail to do something fast enough or on
25. Option (A) is an incorrect grammatical construction. Option
time. Option (B) means to use something for help because
(B) means to be aware of something. Option (C) if used,
no other choice is available and fits the sentence. Option (C)
would then refer to the speaker sleeping and not the baby.
means parts of something that break off and drop to the
Choice (D)
ground. Option (D) means to break. Choice (B)
15. The blank has to be filled with words that mean to be free to 26. Option (A) means to visit for a short time. Option (B) means
leave and only option (B) means that. Option (A) means to to enjoy something that is going to happen and is the right
circulate or move or travel in a general sense. Option (C) word since the sentence says ‘show off’. Option (C) means
means to be acquitted or receive no punishment. Option (D) to admire. Option (D) means to hope. Choice (B)
means to finish a piece of work, or finish successfully.
Choice (B) 27. Blank has to be filled with words that mean to come into
fashion once again. Option (A) holds good. Option (B) means
16. The blank has to be filled with words that mean to consider to appear or to arrive. Option (C) means to be conscious
the past. Option (A) means to be watchful or to beware. again. Option (D) means to be understood. Choice (A)
Option (B) means to consider the past events and so is right.
Option (C) means to take care of something. Option (D) 28. The blank has to be filled with words that mean to ‘reduce’.
means to examine a number of things, often in order to select Option (A) means to interrupt. Option (B) means to lessen
some of them. Choice (B) and so is right. Option (C) means to remove a part of
something. Option (D) means to stop working.
17. The blank has to be filled with words that mean to receive Choice (B)
teaching from someone. Option (A) means to sit with a
straight back. Option (B) means to be present in a meeting 29. Option (A) is used in the sense of cutting down (for e.g. a
or class. Option (C) means to receive teaching from tree). Option (B) means to cut off part of something with a
someone. Option (D) means to discuss something. sharp tool and fits the sentence. Option (C) means to cut up
Choice (C) something into pieces with an axe, knife or other sharp
18. The blank has to be filled with words that imply to do nothing. instrument, Option (D) meant to aim blows at something with
Option (A) means to support or defind something. Option (B) a heavy sharp tool as an axe. Choice (B)
means a group of people who agree strongly on a particular
matter and take action together about it. Option (C) means 30. Option (A) ‘stand in’ means to take the place of someone
to be much better than other similar thing or people. .. Option temporarily, Choice (B) ‘stand down’ means to step down.
(D) means to allow something unpleasant to happen without Choice (C) ‘stand over’ means to over-look, Choice (D)
doing anything to stop it and is the right choice. ‘stand back’ means to let something happen. Choice (B) is
Choice (D) the apt Choice. Choice (B)

19. The blank has to be filled with words that mean to encounter 31. Option (A) means to examine something – here the field of
a lot of obstacles. Option (A) means to incur say bills and to literacy criticism. Option (B) means to examine a number of
increase them by continuing to buy things and put them things, often in order to select some. Option (C) means to
down to one’s account. Option (B) means to meet someone consider past events and actions. Option (D) means to
accidentally. Option (C) means to encounter difficulties or inspect critically. Choice (B)
opposition and is the right choice. Option (D) means to
32. The blank has to be filled with words that mean to discuss.
pursue. Choice (C)
Option(A) means to give a rude reply to someone. Option (B)
20. The blank should be filled with words that imply to seek when used to refer to a person means ‘to prevent’ and in
refuge in a particular place. Option (A) means to remove or other contexts ‘to persuade’. Option (C) is used to speak with
extract. Option (B) means to attribute wrong identity or enthusiasm about something. Option (D) means to discuss
qualities to someone. Option (C) means withdraw remarks, a problem or situation with someone, often to find out their
accusations etc. Option (D) means to find safety in opinion or to get advice before making a decision about it
something and hence is suitable here. Choice (D) and hence is the right choice Choice (D)

21. The blank means to stop something. Option (A) means to 33. Option (A) means to pretend something is something else.
stay level or equal with a person or thing. Option (B) means Option (B) means to tell someone something that another
to continue to talk in an annoying way about something. person has told you and it fits the context. Option (C) means
Option (C) means to refrain from walking or from coming too to ‘become unconscious’ It can also mean ‘to leave college’.
close. E.g. keep off the grass. Option (D) means to prevent Option (D) means to ignore or not to give attention to
something from entering a place and so is right. someone or something. Choice (B)
Choice (D)
34. Option (A) means to accede to or support a plan or
22. The blank has to contain words that refer to the act of filing suggestion and is apt here. Option (B) means parts of
in details required in the form. Option (A) means an act of something that break off and drop. Option (C) means ‘to fall’.
filling up (ex: petrol in car). Option (B) means to complete Option (D) means to be tricked. Choice (A)
Triumphant Institute of Management Education Pvt. Ltd. (T.I.M.E.) HO: 95B, 2nd Floor, Siddamsetty Complex, Secunderabad – 500 003.
Tel : 040–27898195 Fax : 040–27847334 email : info@time4education.com website : www.time4education.com SM1001941/184
35. Option (A) is used to refer to something that stops working. 46. The blank must be filled with words that suggest the idea of
Option (B) means to agree or accept defeat. Option (C) ‘for example’. Looking at Option (A) the prepositional phrase
means to stop doing something and is the right phrase here. is used to introduce a particular event, situation or person
Option (D) means to return something to the person who that is an example of what one is talking about and hence is
gave it to you. Choice (C) appropriate. Option (B) is used to indicate that the statement
you are making applies equally to the two or more
36. The blank has to be filled with words that mean gradually, alternatives that you have mentioned. Option (C) is said in
disappear. This is seen in option (C). Option (A) means to order to mention something that is the first step in a series of
become thin and disappear after repeated use or rubbing. actions. Option (D) means any action that is done by one
Option (B) means to make someone feel tired and less able because it has been ordered or requested to be done. So
to deal successfully with a situation. Option (D) means to use only Option (A) fits the idea in the context. Choice (A)
something so much that it is damaged and cannot be used
any more. Choice (C) 47. The blank must be filled to convey the idea of ‘as much as’.
Option (A) means going regularly to a place and is often
37. Option (A) means to agree and fits the context. Option (B) followed by the preposition ‘of’ something. Option (B) is used
means to stop working. Option (C) means to stop doing in comparisons to refer to the degree of something. Option
something annoying. Option (D) means to stop doing a (C) means by a great amount. The words ‘by far’ would never
regular activity or job. Choice (A) followed by the word ‘as’. So only Option (B) aptly fits the
38. Option (A) is the right answer as it refers to the act of the car blank. Option (D) is used to say that an activity has gone well
running its tyres over the animal. Option (B) means on the until now. Choice (B)
opposite side of something. Option (C) is incorrect. Option
48. The blank has to be filled with words that means until a future
(D) means to run fast in order to get something.
point in time. Option (A) means because of something. But
Choice (A)
this is not mentioned in the context so cannot fit in the blank.
39. Option (A) means to surprise or shock someone so much Option (B) means from this moment and always in the future.
that they do not know how to behave for a short time. Option Option (C) means for the time being until a future point in
(B) means to take back something after giving it. Option (C) time and fits the blank aptly. Option (D) is said to attract
means to take to pieces. Option (D) means to be deceived. attention to what you are going to ask or suggest.
It can also mean to care for someone. Choice (A) Choice (C)

40. Option (A) means to begin a new or afresh. Option (B) 49. The blank must be filled with words that means something
means to help someone to start an activity, especially a that can or must be done within a short period of time. Option
piece of work. Option (C) means to start complaining angrily (D) indicates that something has to be done within a short
to someone about something they have done and is the right period of time. Other expressions where the word ‘notice’ is
phrase. Option (D) means to begin one’s life in a particular used with the same meaning is ‘at a moment’s notice’ or ‘at
way. Choice (C) twenty-four hours’ notice’. The rest of the options do not fit.
Choice (D)
41. If an event is ‘due to’ something, it happens or exists as a
direct result of that thing. This is the meaning in the context. 50. The blank must be filled with words that suggest an idea of
You can say ‘with due’ as in the case of giving the respect. So ‘as opposed to’. Option (D) is used with this idea. Option (A)
you can say ‘with due respect’ when you are about to disagree means ‘rather’ as in ‘ I longed instead for a quiet country life’.
politely with someone. If something is ‘due at’ a particular time, Option (B) means to a slight degree. Option (D) means a
it is expected to happen, be done or arrive at the time. If substitute or alternative to something. Option (C) is an
someone is ‘due for’ something, that thing is planned to incorrect grammatical usage. Choice (D)
happen or be given to them now, or very soon, often after they
have been waiting for it for a long time. Choice (A) PRACTICE EXERCISE – 9

42. The right option is Option (C) – ‘in case’. ‘In case’ would refer 1. In all the statements of the question we find the use of the
to an inclination or desire. However Option (A) – ‘in case of’ correlative conjunction not only - - but also''. In statements
refers to a situation. If you do something or have something A, B and D the correlative conjunction is incorrectly used.
‘in case of’ a particular thing, you do it or have it because that When conjunctions are used as correlatives, each of the
thing might happen or be true. If you say, that something ‘is correlated words should be placed immediately before the
the case’, you mean that it is true or correct. You say ‘as the words/phrases to be connected. In the given sentence the
case may be’ to indicate that the statement you are making entities to be connected are ‘in their own societies’ and ‘in
applies equally to the two or more alternatives that you have those constructed about ….. counterparts'. Hence, each of
mentioned. Choice (C) correlated words should precede each of these entities. This
is found only in statement C. Choice (C)
43. Your use expressions such as ‘at the moment’ or ‘at this
2. Statement B can be eliminated because an adjective in the
moment’ or even ‘at the present moment’ to indicate that a
comparative degree (less) should be followed by than and
particular situation exists at the time when you are speaking.
not by ‘but’. Statement C is ruled out due to the incorrect
You use ‘of the moment’ to describe someone or something
tense, ‘has been due less to (the present perfect). Since the
that is or was especially popular at a particular time,
reference is to an incident of the past, the sentence should
especially when you want to suggest that their popularity is
take the simple past tense. This leaves us with statements A
unlikely to last long. You use ‘for the moment’ to indicate that
and D. The use of ‘that’ makes statement A incorrect
something is true now, even if it will not be true in the future.
because there is no subject which is being referred to by
Options (A) and (C) seem to be a close call. Option (A) refers
‘that’. The idea to be conveyed is the fall in death rate is due
to the precise and specific ‘point’ in time. Option (C) however
less to medicine than to improvements in the environment.
talks of a ‘period’ of time. Option (D) is not suitable in the
And this is appropriately conveyed by statement D.
given context. Choice (A)
Choice (D)
44. Only Option (A) is a correct grammatical prepositional phrase
3. Since the sentence begins with 'had' (past perfect), the main
which means comprising of a heavy load. Choice (A)
clause should take 'would have'. Choice (C) is ruled out as it
45. The blank must be filled with words that means ‘never’. violates this rule.
Option (A) is not worded correctly. Option (B) means with no Statements A and B are ruled out for violating the rule of
money. Option (C) means ‘no possibility’ and is not parallelism.
appropriate in this context. The expression ‘not by any The use of ‘destroying’ after ‘rather than' renders the
means’ also conveys this same idea. Option (D) means sentence incorrect. ''Would have distributed goods rather
‘never’. Choice (D) than have destroyed them'', would be the correct structure
Triumphant Institute of Management Education Pvt. Ltd. (T.I.M.E.) HO: 95B, 2nd Floor, Siddamsetty Complex, Secunderabad – 500 003.
Tel : 040–27898195 Fax : 040–27847334 email : info@time4education.com website : www.time4education.com SM1001941/185
as this is in accordance with the rule of parallelism according 10. Statement A alone is right. In statements B and D, the error
to which, when two or more phrases or clauses are is in the use of the phrasal verb 'live in' and 'live up'
combined using a conjunction, all of them must belong to the respectively. The correction is 'live on in' which means
same grammatical form. Therefore they would – (have "continues to live even now". Statement 'C' is incorrect due
distributed goods rather than have destroyed them) is the to the use of 'lives on'. The subject is 'those aspects' which
correct break up. As the object (goods) is given in the both is plural hence, the verb does not take 's'. Choice (A)
places (after distributed and after destroyed) 'have' should
precede each of the verbs separately. This is found in 11. The relative pronoun ‘which' in the underlined part of the
statement D. Choice (D) sentence refers to ‘deforestation’ which is singular. Hence
"produces …and allows …" would be the correction.
4. Statement ‘A’ is ruled out due to of the use of the plural Choice (D)
‘embodiments'. The reference is to the Victorian period in
Britain and America. It is singular. Hence 'the embodiment' 12. The underlined part consists of an error in the verb used. The
is correct. Statement ‘B’ is incorrect because of the pronoun subject is 'a savvy and affluent class' which is a collective
'it' in the last part of the sentence. The sentence conveys the noun. Hence, the verb following it should also be in the
idea that the Victorian period may seem to be the singular.
embodiment of traditional values but when this era began in Choice (A) has a prepositional error i.e., ‘open for’ and the
Britain and America 'they' (values) were anything but correction is ‘open to'. The absence of the indefinite article
traditional. Hence, the pronoun ‘they’ is apt. The usage of the ‘a’ before ‘savvy’ rules out Choice (B). The perfect
present perfect tense 'has began' renders D incorrect. The continuous tense and the perfect tense used in choice (C)
sentence refers to a specific point of time in the past. Hence, respectively make them incorrect option. Only choice (D) is
it should take the past tense 'began'. Only statement ‘C’ is grammatically correct. Choice (D)
not erroneous. Choice (C)
13. The underlined part is incorrect as there is ambiguity in the
5. Statement A is incorrect because when a sentence begins idea expressed by it. The subject of the sentence is ‘science’
with “Apart from causing - - - “; it has to further continue but the absence of the pronoun 'it' in the underlined part
saying that climate change is causing something else also, makes it incorrect. Choice (A) also is ruled out on the same
which is not the case in A. Statement B talks about the grounds.
effects of climate change but 'one of the most dramatic In choices (B) and (C) the use of past perfect and simple past
effects have been' would be incorrect. The reference is to is incorrect because the sentence refers to a fact which
one among many effects. Hence ‘one of the effects … has should be expressed in the simple present.
been' is the correction. Statement D would be correct if it had The correction is leave them to their suffering. Hence, only
been ‘aside from' which means ‘apart from’. choice (D) has no errors. Choice (D)
Choice (C)
14. The underlined part of the question sentence is incorrect due
6. Statement A is ruled out due to the use of a plural verb ‘are to the absence of the verb ‘does’ after 'the U.S'. The
numbed’. Since the reference is to "ability", a singular noun, comparison is between the produce of the European Union
'is numbed' would be correct. Statement B is ruled out as the and that of the U.S. Choices (A) & (B) are ruled out due to
pronoun ‘they’ is incorrect. It conveys the idea that the incorrect positioning of the adverb. Choice (D) is ruled
technological processes numb the ability whereas, it is we out due to the use of ‘than that of'. Hence, the underlined
who numb our ability by focussing on technological part can be corrected in two ways
processes. Choice (D) (1) currently producing nearly a third more than the U.S
does.
7. Statement A is ruled out because of the absence of the or
pronoun ‘it’ in the second clause. The absence of 'it' leaves (2) that currently produces nearly a third more than the U.S
the idea conveyed ambiguous. Statement ‘B’ is incorrect due does. Choice (C)
to the use of ‘confronted by'. 'Confronted by' conveys the
idea that something needs to be dealt with by someone 15. The underlined part has a puntuational error. The underlined
which is not apt in the context. ‘Confronted with' is part should be appropriately punctuated as ‘which we all,
appropriate. It conveys the idea to have to deal with or react believers and unbelievers alike, make use of'. The words
to – i.e., having to deal with many theories and definitions, 'believers and unbelievers' enclosed by the paranthesis
globalisation begins to look more like a buzzword. The use define the word 'all'. The main idea is 'which we all make
of the past tense ‘became' in statement D makes it an use of'. Choice (D)
incorrect option. Choice (C) 16. The underlined part is incorrect due to the absence of the
preposition 'of' after understanding. The correction is 'lead to
8. Statements A and B are incorrect due to incorrect and
a greater understanding of….'
inappropriate comparisons involved. In statement A, the use
Choice (A) is ruled out due to the absence of the indefinite
of ‘has’ (to a far greater extent than has any) is incorrect
article ‘a’ before greater understanding. The sentence refers
because the reference is to 'lower animals' which is plural.
to a greater understanding of how cells acquire their
Hence, ‘to a far greater extent than have ------', is the
specialized states and provide a strategy. Hence, the use of
correction.
'providing' in (2) makes it incorrect. ‘Understanding of’ is the
The absence of ‘have’ in comparison makes it incorrect
right usage but not ‘understanding as to' as used in
"Human beings developed" is in simple past which is not in
Choice (C). Choice (D)
Keeping with the tense in the rest of the sentence. Statement
D has atones error in statement B. Choice (C) 17. The underlined part is incorrect because 'seen like one' is
ungrammatical. The correction is 'seen as one'. Choice (A)
9. Choice (A) is ruled out because of the use of ‘as that of their is ruled out because the verb ‘combines’ does not agree with
…’. There is no adjective to be referred to by ‘that’ in the part the subject 'features of capitalism' which is plural. It would be
of the sentence preceding it. Choice (C) is eliminated due to correct to say 'features of capitalism combine'. Choice (C) is
the incorrect positioning of the adverbs ‘remarkably' and incorrect due to the incorrect positioning of the adverb. The
'particularly'. The idea intended is, 'patterns of life have words 'to providing' in Choice (D) make it incorrect.
changed significantly, especially in the past. Therefore, the Choice (B)
correction would be ‘patterns of life have changed
remarkably, particularly..’. Choice (D) is ruled out due to the 18. The underlined part is incorrect due to the absence of 'as' after
incorrect tense i.e., have been. The reference is to 'as long'. 'So long as’ is the correct phrase which conveys the
modifications which happened in the past. Hence, the past idea 'till the time the country does not …enable….'.
tense is to be used i.e., as their forebears 'were'. In Choice (A) the phrase is appropriately used but 'it's' is
Choice (B) incorrect. It‘s' means 'it is', which is incorrect in the context.
Triumphant Institute of Management Education Pvt. Ltd. (T.I.M.E.) HO: 95B, 2nd Floor, Siddamsetty Complex, Secunderabad – 500 003.
Tel : 040–27898195 Fax : 040–27847334 email : info@time4education.com website : www.time4education.com SM1001941/186
The correction is 'its'. The absence of 'a' before 'healthy and 27. Choices (A) and (B) are incorrect due to tense errors. The
productive life’ makes Choice (B) an incorrect choice. words 'for long' suggest that the sentence should be in the
In Choice (C) 'enable all its citizens having' is incorrect. past perfect i.e., 'Buddhism had for long' since this reference
Enable takes a ‘to infinitive. Hence, the correction is ‘enable is to an act of the long past. Both (C) and (D) have 'had for
all its citizens to have…’. long' but the use of singular present tense verb (has been)
Only choice (D) is free of errors. Choice (D) after philosophy makes Choice (C) incorrect. Hence only (D)
is free of errors. Choice (D)
19. The use of ‘as’ in the underlined part is incorrect. ‘As’ is used
in comparisons to refer to the extent or degree of something, 28. Choices (B) is ruled out is due to the inappropriate use of
but the intended idea in the context is that like all highly 'than that of' and (C) is ruled out due to the use of "than those
conflicted encounters the question of interpreting British and of". Choice (A) is ruled out due to inappropriate comparisons.
Indian histories is a matter of debate i.e., a similarity is drawn Choice (D)
between two situations.
29. Choice (B) is ruled out because ‘a little doubt’ means some
Hence ‘like all such …’ would be correct 'like' means 'similar
doubt; but the context should take ‘little doubt' which means
to'.
‘leaves no doubt’. Choices (C), (D) are ruled out because of
Choice (A) is ruled out because it makes little sense without
tense errors. The sentence which fits in the blank should be
'if' before 'not'. Choice (B) has a punctuational error. The
in the simple present tense. Choice (A)
comma should be placed after 'edifying' i.e, 'a matter of
strenuous, if not always edifying, debate. Only choice (D) is 30. The question sentence begins with an If – clause and the
grammatically correct. Choice (D) verb in the if-clause is in the past i.e., 'were'. Hence, the
verbs in the main clause should also be in the past, 'could'
20. The underlined part is incorrect due to the use of 'with that and 'would' respectively. Only Choice (D) is correct in all
of'. The comparison is between exertions of man with the ways. Choice (D)
exertions of his fellow beings. Hence ‘with those of his fellow
creature …’ is correct. Choice (B) involves an illogical 31. Statements b, c and d are incorrect. ‘Cited to be' is incorrect
comparison i.e., exertions are compared with fellow in B. Something is 'cited as'. ‘Opening to' in c is incorrect.
creatures. Hence it is ruled out. Choice (C) violates the rule The correction is ‘opening up to’. ‘Craves for’ in choice d
of parallelism 'the privilege is that ––– of combing ….. and -- makes it incorrect. 'Craves for' means 'pleading' whereas,
--- of acting'. The absence of 'of' before acting makes it 'Craves' means 'desire' which is apt in the context. Further,
incorrect. The absence of that in (D) renders both the ‘for’ is used with crave when the verb is intransitive. Eg; She
choices incorrect. Choice (A) craves for more. The verb in this sentence B intransitive. In
the question sentence the verb is transitive, hence 'for' is not
21. Choice (A) is ruled out due to a tense error. The past perfect used. Choice (C)
tense 'had existed' is incorrect since the reference is to the
past (the Second World War). Choice (C) is also ruled out on 32. Statements b, c and d are incorrect. Statement ‘b’ is incorrect
the same grounds i.e, incorrect tense (that has existed). due to the absence of a pronoun before 'engage'. The
Choices (B) have structural errors. Only choice (D) is correct. reflexive pronoun (themselves) after 'engage' has to be
Choice (D) used. ‘They engage themselves’ will be the correction.
In statement ‘c’ the correction is 'they can assume either
22. The phrase ‘one of the essential roles’ suggests that one out reproductive or non-reproductive roles ----.
of many aspects is being referred to. Choice (A), to challenge Statement d has an error in the verbs used i.e, 'epigenetics
and to make, conveys that two different aspects are being influence' is incorrect. ‘Epigenitics’ is the name of a subject. Hence,
referred to. This does not (logically) fit in the blank. Choices ‘influences', a singular verb, is the correction.
(C) and (D) violate the rule of parallelism. Hence, only Choice (C)
Choice (C) is grammatically and logically correct.
Choice (C) 33. Statements a, b and c are incorrect.
The error in ‘a’ is the absence of the definite article before
23. Only Choice (D) is the right option to go into the blank. The ‘last time.’ i.e., ‘the last time’, since the reference is to a
tense in the rest of the options is not in line with that given in specific point of time. We ‘clamber up’ a mountain. Hence
the first part of the sentence. The first part of the sentence is ‘clambered up’ is the correction in B. Statement c is incorrect
in the present perfect tense, therefore 'so too has been due to the wrong positioning of the adverb ‘dangerously’. It
our…..', which is also in the present perfect tense aptly fits should be positioned after the verb ‘swaying’. Hence a, b and
the blank. Choice (D) c are incorrect. Choice (A)
34. Statements b, c and d are incorrect statements. b begins with
24. Choice (A) is illogical and hence is ruled out. It would be right
a negative word. Hence the structure should take an
to say 'to exploit the earth for what we merely want apart
inversion. (verb + subject). ‘Neither is it' is the correction.
from what we need', but the structure is reversed
Statement ‘c’ should read ‘make little sense’ i.e., make no sense.
in (A).
‘A little' is incorrect in the context of the sentence. Statement d is
Choice (B) is grammatically incorrect. The error is in the
incorrect because of the absence of an apostrophe('s). The
structure. The correct structure should be "not just for what
correction is 'heart’s truth'. Choice (B)
we …but for..".
The use of as well as with both makes Choice (C) incorrect. 35. Statements c and d are incorrect. ‘Home for’ is incorrect. The
Again. Only choice (D) is grammatically correct. context in the sentence suggests that it is an 'abode for'.
Choice (D) Hence ‘home to’ would be the correction.
The verb ‘have’ in d does not agree with the subject
25. In all the options we find a variation in the use of articles ‘escalation’ which is singular. The verb ‘has’ is the correction.
which precede the words ‘theory and practice’. The Choice (D)
reference is to a particular theory and a particular practice of
a culture. Hence 'the theory and the practice of a culture' 36. Statements a, c and d are incorrect. ‘Reeling in’ in
‘would be correct. Choice (A) A makes it incorrect because we ‘reel under a crisis', and not
‘reel in'. 'Reeling under a crisis’ means suffer because of a
26. In all the five options there is a variation in the position of blow. The context in the sentence suggests that Europe is
‘only’ which distorts the meaning conveyed. suffering because of a blow. (crippling economic crisis). In
The right option would be the one which answers the ‘c’, ‘coping up’ is incorrect. The correction is ‘coping
question ‘by whom can the unbridgeable gap between measures' i.e, ‘measures to deal with something effectively’.
several powerful and competing ways of life be surmounted? Statement d is incorrect due to the use of ‘would’ which is
Hence, the choice in which ‘only’ is positioned after the past tense of ‘will’. The correction is ‘will now feel’.
‘surmounted', would be correct. Choice (D) Choice (C)
Triumphant Institute of Management Education Pvt. Ltd. (T.I.M.E.) HO: 95B, 2nd Floor, Siddamsetty Complex, Secunderabad – 500 003.
Tel : 040–27898195 Fax : 040–27847334 email : info@time4education.com website : www.time4education.com SM1001941/187
37. Statements b, c and d are incorrect. ‘Cut-away’ in B is ‘A little doubt’ which means ‘some doubt’ makes Choice (B)
incorrect. The correction is ‘cut off’ which means 'separated illogical. ‘Aware of’ is the correct usage. The absence of the
from the rest of the world'. In statement c, it is incorrect to preposition ‘of’ makes Choice (D) incorrect
say “the Gharwal hills in the Himalayas”. The correction is “of Hence, only Choice (C) is free of all errors. Choice (C)
the Himalayas”. “Full with rare - -" in d is incorrect. The
correction is “full of rare - - -". Choice (C) 46. Statements a, and d are incorrect. The correction in a is 'the
25th Marines'. The reference is to a specific unit. Hence, the
38. Statements a, b and c are erroneous. The absence of the definite article should precede 25th Marines. In statement ‘c’
definite article before '28th' makes a incorrect. The correction 'those of' is incorrect.
is 'the 28th state of the union'. In ‘d’ the use of ‘rose up’ is incorrect. It should be either ‘shot
‘Carved from’ is incorrect in b. ‘Carved out of' is the correct up or rose’. Choice (B)
usage.
‘Comprises of’ in c makes the sentence incorrect. ‘Comprise’ 47. ‘Dying off’ in statement a is incorrect the correction is ‘dying’.
means 'consist of', so using 'of' again is redundant. ‘Agree with’ in B makes it incorrect. We ‘agree with' a person
Choice (C) and 'agree on' an idea. Hence, the context in the sentence
takes ‘agree on’.
39. ‘Ingredient to feasting' in a is incorrect. The correction is In statement d, the use of 'not' with 'unless' makes the
‘ingredient of feasting'. sentence incorrect. ‘Unless’ means ‘if not’ so using ‘not’
The absence of 'the' before ‘streets’ in statement b makes it again makes the sentence incorrect. The correction is
incorrect. The correction is "but in the streets”. 'unless we address'. Choice (A)
Statement d is also incorrect. ‘For celebrating' in d is
incorrect. ‘Imperative to celebrate” is the correction. 48. In a ‘differ from’ is incorrect in the context of the sentence.
One should have an ‘imperative to do‘ something. We 'differ from' others. But we differ in doing something.
Choice (D) Hence, the correction is ‘differ in the way …’. Coming to join
us in C is incorrect. The correction is ‘coming in to join us’.
40. Statements b, c and d are incorrect. We ‘canoe to a nearby ‘As multicultural like’ in D is an incorrect usage. The correct
limestone cliff'. It is ‘a cliff’ or 'the cliff'. The absence of an structure is 'as multicultural as'. Choice (A)
article before 'cliff' makes this sentence incorrect.
‘Top the day’ in c ,makes it incorrect. The correction is ’top 49. Choice (A) is ruled out due to the absence of the definite
off’ . If we ‘top off’ a day or an event, we end it in an especially article ‘the' before the name of the organization Food and
satisfactory way. Agricultural Organization. Choice (B) has a tense error. The
In statement d, the correction is "far from the crowd". The past time adverb ‘ago’ suggests that the sentence should be
absence of 'the' makes it incorrect. Choice (D) in simple past which is not the case in Choice (B).
The use of ‘100 millions tonnes’ in Choice (C) is incorrect.
EXPLANATORY NOTES FOR ADDITIONAL QUESTIONS The correction is '100 million tonnes'.
Hence, only choice (D) is free of errors. Choice (D)
41. Statement A is grammatically incorrect because of the use
of the singular ‘has tended’. The subject is 'forecasts’ which 50. Choice (A) is ruled out because of the use of ‘such as’. The
is plural and hence the correction is ‘have tended to be part of the sentence which follows the word ‘canon’ should
upbeat‘. Statement B is ruled out due to the absence of the actually define it (canon). Hence the correct structure is ‘that
definite article before, ‘earlier one’. The reference is to every people accused of crimes are presumed innocent until found
forecast which reflected greater optimism than each earlier guilty’. The use of 'which' after ‘canon’ in Choice (B) renders
one. Statement C is incorrect due to the past perfect ‘had it incorrect.
tended’. Choice (D) makes no sense due to the absence of the
Only Choice (D) is the most appropriate. Choice (D) relative pronoun ‘that’ after ‘canon’.
The correction is ‘accused of‘. Only Choice (C) is free of
42. Statement A is incorrect due to the use of an incorrect errors. Choice (C)
pronoun. i.e., 'its believers'. The reference is to educational
television, interactive computer programmes and online PRACTICE EXERCISE – 10
learning i.e, a plural subject. Hence the use of ‘its’ is
incorrect. The correction is ‘their believers’. In statements B 1. Options (A), and (D) are incorrect because of the use of
and C the position of the adverb ‘yet’ is inappropriate. ‘Yet’ inappropriate prepositions in and for. The correct preposition
as used in the sentence means ‘still or even’. And it should is 'of'. It is 'of their fellow-creatures'. These fellow-creatures
be positioned after ‘nothing’. Only statement D is free of (i.e) the men are more privileged when compared to the
errors. Choice (D) women. As a comparison is made the use of 'more' is
warranted. The use of 'most privileged' is incorrect in 3. Only
43. The underlined part is incorrect because ‘let it know’ is option (B) is grammatically correct. Choice (B)
syntactically incorrect. ‘Let it be known’ is the correct syntax.
2. Option (A) is incorrect '.... and put' is inapt. The construction
Another important point to be noted here is ‘let’ does not take
logically takes a gerund. '.......could prevent ,.... putting the
the ‘to + infinitive. It takes the plain infinitive. This rules out
federation's reputation..........'. Also its reputation 'as a
(B) and (D). Something is ‘known to the world’ but not ‘known
business-friendly hub is at risk'. The use 'of' and 'in' in place
by the world’. Hence only Choice (C) is free of errors.
of 'as' is incorrect in options (A) and (B) and (E). 'At a risk'
Choice (C)
'or' 'at the risk' is incorrect, which rules out options (B), and
44. The underlined part is ruled out due to the absence of the (D). Only option (C) is grammatically consistent.
indefinite article ‘an’ before 'individual identity’. It is 'an Choice (C)
individual identity'.
3. Only choice (E) fits into the blank because the tense ie,
Only Choice (A) is grammatically correct. In all the other
present perfect tense (has brought) is in keeping with the
options the positioning of ‘ideally’ is incorrect. It is modifying
tense in the rest of the sentence. The pronoun 'they' is
the verb ‘provides’ hence it should be positioned before
correctly used in the latter part of the choice ie, 'we know are
‘provides’. Choice (A)
flawed in light of the ecological destruction they legitimize.'.
45. The underlined part has a prepositional error, i.e, ‘concerned the pronoun 'it' as used in (A) , (C) and (D) is incorrect
of’. The correction is ‘concerned over/about'. When we worry because the reference is to a plural entity – features of
about a situation or a problem we use ‘concern over’. ‘Raise’ economic philosophy – . Further the preposition 'to' is
in Choice (A) is incorrect. ‘Raise’ is used when something correctly used with 'obligation'. 'obligation to change….'
causes something to go up. ‘Rise’ is used when something Obligation for' as used in (B) and (C) rules out these two
happens naturally. Hence ‘the significant rise’ is correct. choices. Choice (D)
Triumphant Institute of Management Education Pvt. Ltd. (T.I.M.E.) HO: 95B, 2nd Floor, Siddamsetty Complex, Secunderabad – 500 003.
Tel : 040–27898195 Fax : 040–27847334 email : info@time4education.com website : www.time4education.com SM1001941/188
4. In choice (A) and (B) the positioning of 'generally' distorts the they are good, attract the best......' The verb 'obtain' to mean
meaning. Further the prepositions 'through' and 'with' which 'exist' is used in the progressive tense in (d) which is
follow the words 'cultural practices' in choices (A) and (D) incorrect. It has to be '.........factors do not generally obtain
respectively make those options incorrect. Besides the for.........' Choice (D)
above mentioned errors the positioning of 'intrinsically’ is
incorrect in (B) and (D). 11. The underlined part is incorrect due to use of the quantifier
'Intrinsically' means 'naturally' or 'innately' and it should be 'much' after 'crisis. The sentence involves a comparision
positioned before right or wrong. Hence, only (C) is free of between 'providing humanitarian assistance' and 'enabling
errors. Choice (C) countries to grow'. In a negative sentence the comparision is
"not so much …as". This structure is seen only in (D).
5. Choices (A) and (D) are ruled out due to a discord between The absence of 'by' after 'as' in choice (A) disrupts the
the subject and the verb. The subject of the sentence is parallelism. Choice (B) is ruled out due to inappropriate
'civilization' which is singular, hence verb takes(s) -. comparison. Choice (C) is incorrect due to the words 'as to
'continues to follow…' is the correction. enable' which violate the parallelism in the sentence. The
In (3), 'continue devastating…' is incorrect. The correction is correction is 'not so much by ….as by enabling …..".
'continue to devastate'. Choice (D)
Only (B) is free of errors. Choice (B)
12. The underlined part is erroneous for two reasons: one is the
6. Statements (c) and (d) are erroneous. 'Universally accepted' absence of a comma after 'which' and the other is the use of
means something that is accepted by everyone, which has the singular from of the verb 'was transforming'.
to replace 'acceptable' in (c). 'Acceptable' means something The words "from the latter part of the seventeenth century"
that is approved of by most people. Here in the given context, gives additional information hence this should be separated
handshake is accepted by everyone. There is no other with commas. Further the relative pronoun 'which ' refers to
choice. We cannot have 'leg shake' to say that of the two, the achievements which is a plural subject. Hence, the verb
handshake is more acceptable. Hence (c) is erroneous. The after 'century' should be a plural verb ie, were transformed.
use of the gerund 'clobbering' is incorrect in (d). It has to be Only choice (B) has all these correct. Choice (B)
the 'to infinitive'. The stranger might try 'to clobber him',
(i.e, hurt him). Try 'clobbering him' is absurd. 13. The underlined part, has two errors, 'lived for many years', is
Choice (B) incorrect. The correction is 'lived on for many years' which
means that it continued for many years is apt in the context.
7. Statements (a), (b) and (d) erroneous. The positioning of the The second error is in the positioning of the comma after
adverbial phrase 'in the world' is incorrect in statement (a). 'which'. It should precede 'which' and 'which' describes the
This phrase qualifies 'person'. 'Every 10th person in the 'events' of 1968.' All these corrections are found only in
world who dies in road accident' is the idea suggested. choice (C). Hence, Choice (C)
Hence 'in the world' should follow 'person' and not 'accident'.
Statement (b) has an error of subject-verb concord. 'The 14. Choice (A) and (D) are ruled out due to the presence of
number' is the subject and hence it has to be 'is increasing'. a definite article before 'nature' 'Nature is abstract hence it
The use of 'are increasing' makes it grammatically incorrect. does not take an article. The use of the continuous tense
The use of the word 'status' is incorrect in statement (d). 'mutilating man' is also incorrect since the first part is in the
'Status' means rank or honour. It has to be replaced by 'state' simple present tense. These errors rule out (A) and (D).
meaning 'condition'. We are talking about the condition of the Choice (B) is ruled out due to the absence of articles before
road. Hence the correction is '...the state of the roads'. 'system' and 'type'. Choice (C)
Choice (D)
15. Choice (C), and (D) are ruled out due to subject verb discard.
8. Statements (a), (b) and (c) are grammatically incorrect. The subject of the sentence is 'fact' hence 'suggests' is the
The use of an incorrect phrasal verb makes statement A appropriate verb. Further 'would continue and 'will be
incorrect. To 'make up' something is to constitute and to continue' as used as choice (A) and (D) respectively are
'make up for' something is to compensate. Here the 'deserts incorrect. The correct tense is 'will continue to evolve'.
constitute...' and not compensate. Hence '........which make Besides the above mentioned errors 'for adopting' is
up about...' is the correction in (a) '........at the risk of starving' incorrect in (C); the correction is 'to adopt' Only choice (B) is
is incorrect in statement (b). They are 'at risk of starvation'. free of errors. Choice (B)
'At risk of something' means in danger of something
unpleasant or harmful, happening. 'At the risk of doing 16. Part (c) is erroneous. "b and of robbers must have a sense
something' is used to introduce something that may sound of justice among them", is incorrect. The correction is 'among
stupid or may offend someone and hence is inapt in this themselves'. It answers the question 'among whom'. Hence
context. The use of 'one-thirds' is incorrect in statement (c). 'among themselves' is the desired response. Part (d) is
It should be 'one-third'. Choice (D) incorrect due to the incorrect phrasal verb 'pulling out'. 'Pull
out' means 'to withdraw'. 'Pulling off' is the right phrasal verb.
9. Statements (a), (c) and (d) are grammatically incorrect. 'Pull off' means 'to succeed in doing something difficult'. The
The positioning of the adverb 'enough' is inapt in (a). error in statement (e) is the absence of the auxiliary verb
'The temperatures are lowered enough to increase the ice before 'detrimental'. "They are constituent parts.......and are
sheets' is the idea suggested. Hence 'enough' should be detrimental........" The conjunction 'and' connects two
placed after 'temperatures' and not after 'ice sheets'. different ideas hence, both the ideas must be of the same
Statement (b) is grammatically correct. The use of 'farther' grammatical structure.
which is used to denote distance is incorrect in (c). It has to Only (a) and (b) are free of errors. Choice (C)
be 'further' which means additional. Statement (d) is
incomplete because of the omission of the preposition 'to' 17. Part A has a punctuational error i.e the absence of a comma
after 'adjust'. We 'adjust to' something. Choice (C) after 'Beck says'. The main idea is "September 11th was the
Chernobyl of globalization''. Hence, the words 'Beck says'
10. Sentences (a), (b) and (d) are erroneous. The use of the which occur in the middle of the sentence should be
preposition 'of' is incorrect in (a). There is a 'problem with separated by commas.
mass schooling' and the problem is that it creates Statement (c) is also grammatically incorrect due to the use
inequalities in the society. If we say 'problem of', then the 'promise for'. The preposition 'for' is incorrect. The correction
problem should be within the system which is not the case is 'promise of salvation'. In (d) the error is 'problem of
here. Hence the correction in (a) is '.......problem with mass humanity'. There are many problems which humanity faces.
schooling...... Statement (b) has a punctuational error. Hence, 'the problems of humanity' is the correction.
The correction is '......go to good schools, which, because Only (b) and (e) are appropriate. Choice (D)

Triumphant Institute of Management Education Pvt. Ltd. (T.I.M.E.) HO: 95B, 2nd Floor, Siddamsetty Complex, Secunderabad – 500 003.
Tel : 040–27898195 Fax : 040–27847334 email : info@time4education.com website : www.time4education.com SM1001941/189
18. Statements (a) and (d) are grammatically correct. '(b)' is 25. Only (a) and (e) are grammatically correct. The use of he
erroneous due to the use of an incorrect pronoun i.e 'it is', continuous tense 'being seen;' makes B incorrect. From the
Since the reference is to 'domination and inequities' plural tense in (a) it is clear that the imple present tense would be
number 'they are' should be used. In '(c)' the absence of the correct. Hence, the correction is 'are seen as'.
comma after 'its history' makes it incorrect. In (e) the use of The absence of comma after 'result' and 'objectors' makes
'like' makes it incorrect. The correction is 'as dominated and (c) incorrect. The positioning of 'rather' in (d) makes it
dependent as'. incorrect. The to infinitive should never be split. Hence, the
Hence, only (a) and (d) are free of errors. correction in (d) is 'to make rather seewping….' The absence
Choice (B) of the definite article before 'biotechnology industry' makes
(e) correct. Hence, only (a) is correct. Choice (D)
19. Only (e) is free of errors – (a) is incorrect due to the absence
of the preposition 'from' after 'much'. (b) is erroneous due to 26. The sentence has a punctuational error besides other errors.
the incorrect positioning of the adverb 'mainly'. 'Mainly' The sentence should correctly be punctuated as
modifies 'about England', hence, it should be positioned after "...............information overload, which is, in itself,...........".
'is'. The words 'which is' and 'in itself' should be separated by
commas. Further, in all the statements except 'D', the
In '(c)' the error is in the comparison. The comparison is
correlative conjunction 'not only........but also' is incorrectly
between the idea that is central to William's work to the idea
used. The intended idea is the sheer volume of available
that is central to the work of most scholars and critics. Hence,
information not only obscure but also dissuade. Hence, the
to 'that of most scholars…' is the correction.
sentence should be phrased as "..............available
In (d) 'deal in' is incorrect. 'Deal in' means to 'trade in'. information tends not only to obscure the important issues,
The context in the sentence takes 'deal with'. but also to dissuade....". When a correlative conjunction is
Choice (C) used to connect two entities, each part of the correlative
conjunction must immediately precede each of the word or
20. Only (c) and (d) are grammatically correct. (a) is erroneous phrase to be connected. Only in statement (d) the positioning
due to the word 'characterizes'. The correction is is correct.
'characterize'. The reference is to facts (plural) that Further, in (A) and (C) 'coping up with' is incorrect. 'Cope up'
'characterize world economy'. is incorrect. The correction is 'cope with'. Only (D) is free of
Part B is erroneous due to the absence of the definite article errors. Choice (D)
before 'high cost'. The correction is "the high cost of.........."
In (e) the correction is 'sketch out'. 'Sketch out' means 'to 27. 'Politician and economist are reluctant ….'. The word 'being
give a brief account or a general outline of something'. 'To reluctant' rules out Choice (B) and (E). Similarly 'fail to'
sketch' means to make a quick drawing of something' which follows 'reluctant to'; 'are fail to' and, 'failing to' rules at (A)
is inapt in the context. Only (c) and (d) are free of errors. and (D) respectively only Choice (C) is correct.
Choice (B) Choice (C)

21. Statements (a) and (d) are correct. (b) is incorrect due to 28. Statement (D) is the grammatically correct option because of
illogical comparison. The comparison should be between the the correctness of the tense, ie, (came from; past tense)
'social base of the party of the Mexican Revolution' and the Further, the adverb (safely) is correctly positioned only in this
'social base of the National Revolutionary Party'. Hence, option. 'Safely' should modify 'analyze and understand',
'wider than that of' would be the correction. hence, it should be positioned before "analyze and
(c) is erroneous because the positioning of 'together' which understand". The correlative conjunction 'both.....and' is
makes the sentence syntactically erroneous. The part of the connects "God" and "his creations" and it is correctly used
sentence after the colon always elaborates on the word only in (d).
which precedes the colon. Since, the part of the sentence Statement (A) is ruled out due to the incorrect positioning of
after the colon talks about groups the sentence should be the adverb 'safely'. Statement (b) and (c) have tense errors
correctly phrased as "and it brought together four groups: i.e "comes from his"(present tense), 'had come'(past
..........' (e) is a continuation to (d). (e) is erroneous due to the perfect),has come from (present perfect).Only statement (D)
use of 'rather'. When 'more' is used 'rather' should not be is free of errors. (apart from erroneous positioning of the
used. Hence, the correction is "It was more an attempt to adverb.) Choice (D)
create a functional democracy than............".
Choice (C) 29. Statement (A) is ruled out due to the use of the incorrect
pronoun 'them'. The reference is to 'liberty' which is singular.
22. Only (a) and (d) are free of errors. Hence, 'guaranteeing it' is the correction. This error is
The absence of an article before 'hidden reality' makes B repeated in (C) and E also. Further, in (a), the absence of 'in'
incorrect. The use of 'was' in (c) renders it incorrect. When after 'but' (at the end of the sentence) disrupts the
the reference is to an imaginary and unreal situation the past parallelism. This is repeated in (B). Besides the above
plural form of the verb (were) should be used. Hence, the mentioned errors, the absence of 'the' before 'means' in (B)
correction is "as if it were a". In (e), the use of 'from' after and (C) makes them incorrect. The reference is to 'the way
'escapes' makes it incorrect. Choice (D) through which something is achieved', hence, 'the means' is
correct. Again B and E have tense errors. i.e 'they had
crafted' and 'they have crafted' respectively. Since there is a
23. (a) and (d) are free of errors. reference to a past time frame, the use of simple past would
In (b) the error is the absence of 'the' before 1950s. make the sentences correct.
(c) is erroneous due to the incorrect positioning of by', the Only (D) is free of errors. Choice (D)
correction is "abandoned by both India and China........"
The error in (e) is the absence of 'a' after even. 30. Statements (A) and (B) are erroneous due to subject-verb
The correction is "even a complete l lack..........." discord i.e the use of 'suggest'. The subject of the sentence
Choice (C) is i.e 'the little ' which is singular, non-countable noun, and
the verb takes 's'. i.e (suggests). Secondly (A) is ruled out
24. (a), (d) and (e) are grammatically correct. (b) is incorrect due due to the absence of a pronoun 'it' after 'making'. 'It' is used
to the absence of preposition 'of' after 'concieve'. The error to refer to 'the possibility to attain rich and intricate systems
in (c) is the positioning of 'perhaps'. It should be positioned of knowledge _____'. Absence of 'it' makes the sentence
before 'originated'. 'Perhaps' is used to express uncertainity. incorrect. This error is repeated in (D),. Further in (B) and (d)
There is uncertainity about when 'it' (dualism) originated. the words "for us in attaining" make them incorrect. Thus
Hence 'perhaps' should modify 'originated. Choice (D) only '(C)' is free of errors. Choice (C)

Triumphant Institute of Management Education Pvt. Ltd. (T.I.M.E.) HO: 95B, 2nd Floor, Siddamsetty Complex, Secunderabad – 500 003.
Tel : 040–27898195 Fax : 040–27847334 email : info@time4education.com website : www.time4education.com SM1001941/190
31. In statement (A) and (B) it is seen that the correlative 35. Only choice (B) is right. The 1MF failed to understand, what?
conjunction 'either.............or' is incorrectly used. The correct 'the workings of …' not 'how financial markets work'. The
structure is "Man is seen either as a mechanical plural 'their' and not 'its' is correct since the subject is
robot.........or as a rational........". Further, it is said in the 'workings'. Choice (B)
sentence that man's reactions are similar (like) to those of
mindless matter. Hence, "reactions are ascertainable and 36 (a) and (c) are incorrect. The use of 'home for' in (a) makes it
predictable like those of......" would be correct. The use of incorrect. The correction is 'home to' which means 'abode for'.
'just as' would be correct if the sentence reads ".......just as The absence of the indefinite article 'a' before detailed
those of matter are". The use of 'as' is incorrect because the makes it incorrect. The correction is 'a detailed search …'
sentence refers to a similarity. This rules out (A), (C). Since the reference is to a search.
Besides the above mentioned errors, the use of plural (bear), Hence, Choice (D)
and negative (do not) with 'neither' makes (B), (C) incorrect.
Only D is free of errors. Choice (D) 37. Sentence (a) is incorrect since it should be 'was' not 'were'
as the subject is 'a series'.
32. All the statements except (B) have punctuational and Sentence (b) is incorrect because 'apparently' qualifies
prepositional errors. The adverb 'then' and 'it is argued' 'grew' and hence must precede it '… that apparently grew'
should be separated by a comma i.e; "to cultivate and Choice (A)
reorder creation, then, it is argued", is the correct structure.
38. The absence of 'a' before 'world dominant' and the use of the
Further, in (A) and (D) the absence of the preposition 'at'
comparative 'fewer' (when actually comparison is not
before 'deployment' disrupts the parallelism. The correction
involved) makes Choices (B) and (D) incorrect.
is "draw a line at the level........ or at the deployment....".
In choice (A) and (B) the absence of 'a' before 'position of
Besides the above mentioned errors, in (C) and (D) the
hegemony' renders them incorrect. The reference is to 'a
preposition 'of' before 'the use of' makes the sentences
position'. 'less' is used when the reference is to uncountable
incorrect.
entities. Since the reference is to historical incidents which
Only (B) is free of errors. Choice (B)
are countable, 'few' is appropriate word. Choice (C)
33. Only statement (A) is grammatically and logically correct. In 39 Only (a) and (b) are grammatically correct. The use of the
the other options i.e; in statements (B) and (C) the use of singular verb 'has' in (c) makes it incorrect. Since the
'mutually dependent system' is illogical. 'Mutually dependent' reference is to modern medicine and resource availability
should be followed by a plural noun i.e; 'mutually dependent (two different entities) the verb should be in the plural.
systems'. Similarly the use of the singular number 'a highly In (d), the use of past tense is incorrect., because 'over the
complex' makes (C) and (D) incorrect as the reference is to years' suggests that the present perfect tense should be
'systems'. Further, in all the statements expect (A), the tense used. Hence, the correction is 'longevity has increased'.
in the last part of the sentence is incorrect. The use of 'for The absence of the article before 'huge' in (e) makes it
centuries' suggests that the present perfect tense is correct. The correction is 'a huge elderly population'.
appropriate in the context. Only statement (A) is free of Choice (A)
errors. Choice (A)
40. (b), (c) and (e) are grammatically incorrect. In (b) the
34. Statement (A) is ruled out due to the use of past tense, "The correction is 'came about' with the development of modern
United States extended....." In statements (B) and (D) the humans; 'came about' means 'to take place' which is apt in
use of "could not even imagine..." makes the statements the context. 'Brought all the …' in (c) is incorrect. The
logically and grammatically incorrect. The reference is to a correction is 'brought in ' the sentence should read
past unfulfilled condition. Hence, the use of the perfect '…rapacious ape we … brought all the true …in with us'.
conditional (could not have) would make the option correct. Statement (e) has a spelling error i.e, 'ingenously'. The
Statement (C) is also incorrect due to use of past perfect correction is 'ingeniously' which means 'inventively'.
tense (had extended). Only statement (D) is free of errors. Hence, only (a) and (d) are correct. Choice (C)
Choice (D)

Triumphant Institute of Management Education Pvt. Ltd. (T.I.M.E.) HO: 95B, 2nd Floor, Siddamsetty Complex, Secunderabad – 500 003.
Tel : 040–27898195 Fax : 040–27847334 email : info@time4education.com website : www.time4education.com SM1001941/191

S-ar putea să vă placă și